You are on page 1of 143

Bifurcations: baby normal forms.

Rodolfo R. Rosales,

Department of Mathematics,

Massachusetts Inst. of Technology, Cambridge, Massachusetts, MA 02139


October 10, 2004
Abstract
The normal forms for the various bifurcations that can occur in a one dimensional dynamical
system (x = f (x, r)) are derived via local approximations to the governing equation, valid near
the critical values where the bifurcation occurs. The derivations are non-rigorous.

Contents
3

1 Introduction.
Necessary condition for a bifurcation. . . . . . . . . . . . . . . . . . . . . . . . . . .
2 Saddle Node bifurcations.

3
4

General remarks on structural stability. . . . . . . . . . . . . . . . . . . . . . . . . .

Saddle node bifurcations are structurally stable. . . . . . . . . . . . . . . . . . . . .

Structural stability and allowed perturbations. . . . . . . . . . . . . . . . . . . . . .

Normal form for a Saddle Node bifurcation. . . . . . . . . . . . . . . . . . . . . . .

Remark on the variable scalings near a bifurcation. . . . . . . . . . . . . . . . . . .

Theorem: reduction to normal form. . . . . . . . . . . . . . . . . . . . . . . . . . .

Problem 1: formal expansion to reduce to normal form. . . . . . . . . . . . . . . .

3 Transcritical bifurcations.

Normal form for a transcritical bifurcation. . . . . . . . . . . . . . . . . . . . . . . .

Theorem: reduction to normal form. . . . . . . . . . . . . . . . . . . . . . . . . . .

10

Structural stability for transcritical bifurcations. . . . . . . . . . . . . . . . . . . . .

11

Problem 2: formal expansion to reduce to normal form. . . . . . . . . . . . . . . .

11

Problem 3: What about problem 3.2.6 in the book by Strogatz? . . . . . . . . . .

11

Rosales

Bifurcations: baby normal forms.

4 Pitchfork bifurcations.

2
12

Introduction of the reflection symmetry. . . . . . . . . . . . . . . . . . . . . . . . .

12

Simplest symmetry: f (x, r) is an odd function of x. . . . . . . . . . . . . . . . . . .

12

Problem 4: normal form for a pitchfork bifurcation. . . . . . . . . . . . . . . . . .

13

Problem 5: formal expansion to reduce to normal form. . . . . . . . . . . . . . . .

13

Problem 6: proof of reduction to normal form. . . . . . . . . . . . . . . . . . . . .

13

5 Problem Answers.

14

Rosales

Bifurcations: baby normal forms.

Introduction.

Consider the simple one-dimensional dynamical system


dx
= f (x, r),
dt

(1.1)

where we will assume that f = f (x, r) is a smooth function, and r is a parameter. We wish to study
the possible bifurcations for this system, as the parameter r varies. Because the phase portrait for
a 1-D system is fully determined by its critical (equilibrium) points, we need only study what happens
to the critical points. Bifurcations will (only) occur as these points are created, destroyed, collide, or
change stability. For higher dimensional systems, the critical points alone do not determine the phase
portrait. However, the bifurcations we study here can still occur, and are important. Furthermore,
the normal forms we develop here still apply. Thus:
Consider some critical point x = x0 (occurring for a value r = r0 ) i.e.

f (x0 , r0 ) = 0.

Then ask:

When is (x0, r0) = 0 a bifurcation point? A necessary condition is:


fx (x0 , r0 ) = 0.

(1.2)

Why? Because otherwise the implicit function theorem would tells us that: In a neighborhood
of (x0 , r0 ), the critical point equation f (x, r) = 0 has a unique (smooth) solution x = X(r), which
satisfies X(r0 ) = x0 . Thus no critical points would be created, destroyed or collide at (x0 , r0 ).
Further, obviously: no change of stability can occur if fx (x0 , r0 ) 6= 0.

Without loss of generality, in what follows we will assume that x0 = r0 = 0.

Remark 1 From equation (1.2) we see that bifurcations and undecided (linearized) stability are
intimately linked. This is true not just for 1-D systems, and (in fact) applies even for bifurcations
that do not involve critical points.

Remark 2 For higher dimensional systems (where x and f are vectors of the same dimension),
fx is a square matrix, and the condition (1.2) gets replaced by fx is singular. The proof of this
is essentially the same as above, via the implicit function theorem (even in infinite dimensions, as
long as an appropriate version of the implicit function theorem applies,1 the result is true). We
1

In infinite dimensions the implicit function theorem may not apply.

Rosales

Bifurcations: baby normal forms.

point out that, as long as fx has a one-dimensional kernel (zero is a multiplicity one eigenvalue),
most of what follows next applies for higher dimensional systems as well.

Saddle Node bifurcations.

Given a critical point, say (x, r) = (0, 0), with f (0, 0) = fx (0, 0) = 0, the most generic situation is
that where fr (0, 0) =
6 0 and fxx (0, 0) =
6 0. By appropriately re-scaling x and r in (1.1) if needed,
we can thus assume that:
f (0, 0) = fx (0, 0) = 0,

fr (0, 0) = 1,

and fxx (0, 0) = 2.

(2.3)

Remark 3 For arbitrary dynamical systems (such as (1.1)), we have to be careful about assuming
that anything is exactly zero. Situations where something vanishes exactly are (generally)

structurally unstable, since arbitrarily small perturbations will destroy them. To (safely) make
such assumptions, we need extra information about the system: information that restricts the possible
perturbations in such a way that whatever vanishes, remains zero when the system is perturbed.

Remark 4 In view of the prior remark, the reader may very well wonder: How do we justify
the assumptions above, namely: f (0, 0) = fx (0, 0) = 0? The answer is that: It is the full
set of assumptions in (2.3) that is structurally stable, not just the first two. We prove
this next. Since (2.3) characterizes the saddle node bifurcations (we show this later), this will prove
that: Saddle Node bifurcations are structurally stable.
Proof: First, to show that the first two assumptions (when alone) are structurally unstable, consider
the example: f = r 2 + x2 , with the critical point (0, 0). Then change f to f = r 2 + x2 + 1030 ,
which causes the critical point to cease to exist. This example illustrates the fact that:
Isolated critical points2 are structurally unstable, thus not (generally) interesting.
Second: imagine now that f depends on some extra parameter f = f (x, r, h), such that the assumptions in (1.2) apply for (x, r, h) = (0, 0, 0) here h small and nonzero produces an arbitrary
(smooth) perturbation to the dynamical system in (1.1). Consider now the system of equations:
f (x, r, h) = 0,
2

and

fx (x, r, h) = 0.

These are points such that there is a neighborhood in (x, r) space where there is no other critical point.

(2.4)

Rosales

Bifurcations: baby normal forms.

Now (0, 0, 0) is a solution to this system, and the Jacobian matrix

fx (0, 0, 0)

J =

fr (0, 0, 0)

fxx (0, 0, 0) fxr (0, 0, 0)

2 fxr (0, 0, 0)

(2.5)

is non-singular there. Thus the implicit function theorem guarantees that there is a (unique) smooth
curve of solutions x = x(h) and r = r(h) to (2.4), with x(0) = 0 and r(0) = 0. Along this curve, for
h small enough, it is clear that: fr (x, r, h) 6= 0 and fxx (x, r, h) 6= 0. Thus, modulo normalization,
(2.3) is valid along the curve for h small enough. This finishes the proof.

Remark 5 In the proof of structural stability in the prior remark, we assumed that the perturbations to the dynamical system in (1.1) had the form
dx
= f (x, r, h),
dt

(2.6)

with the dependence in the extra parameter h being smooth. This sounds reasonable, but (clearly)
it does not cover all possible (imaginable or non-imaginable) perturbations. For example, we could
consider perturbations of the form
dx
d2 x
= f (x, r) + h 2 .
dt
dt

(2.7)

What small means in this case is not easy to state (and we will not even try here). However, this
example should make it clear that: when talking about structural stability, for the concept to

even make sense, the dynamical system must be thought as belonging to some class
within which the idea of close makes sense. Further, the answer to the question: is this
system structurally stable? will be a function of the class considered.

Let us now get back to the system in (1.1), with the assumptions in (2.3), and let us
study the bifurcation that occurs in this case: the Saddle Node bifurcation.
We proceed formally first, by expanding in Taylor series and writing the equation in the form
dx
= r x2 + O(r 2 , rx, x3 ),
dt

(2.8)

where all the information in (2.3) has been used. We now look at this equation in a small (rectangular) neighborhood of the origin, characterized by
|x| <  and |r| < 2 ,

(2.9)

Rosales

Bifurcations: baby normal forms.

where 0 <   1. Then the first two terms on the right in (2.8) are O(2 ), while the rest is O(3 ). We
thus argue that the behavior of the system in the neighborhood given by (2.9) is well approximated
by the equation
dx
= r x2 .
dt
This is the Normal form for a Saddle Node bifurcation

(2.10)
see Strogatz book for

a description of its behavior.

Remark 6 A natural question here is: Why the scaling in (2.9)? Such a question can
only be answered after the fact, with the answer being (basically) because it works. Namely,
after we have figured out what is going on, we can explain why the scaling in (2.9) is the right one
to do. As follows: at a Saddle Node bifurcation say, at (x, r) = (0, 0) a branch of critical
point solutions say x = X1 (r) turns back on itself.3 Thus, on one side of the value r = 0,
no critical point exist, while on the other side two are found, say at: x = X 1 (r) and x = X2 (r).
Locally, these two curves can be joined into a single one by writing r = R(x). Then r = R(x) has
either a maximum (or a minimum) at x = 0. Hence it can, locally, be approximated by a parabola.
Hence the scaling in (2.9) is the right one. Any other scaling would miss the fact that we have a
branch of critical points turning around.
Those with a mathematical mind will probably not be very satisfied with this explanation. For them,
the theorem below might do the trick. However, note that this theorem is just a proof that equation
(2.10) is the right answer, showing that (2.9) works. It does not give any reason (or method) that
would justify (2.9) a priori. Simply put: advance in science and mathematics requires places at
which insight is needed, and (2.9) is an example of this; perhaps a very simple example, but one
nonetheless.

Theorem 1 With the hypothesis in equation (2.9), there exists a neighborhood of the origin, and
there a smooth coordinate transformation (x, t) (X, T ) of the form
X = x (x)

and

dT
= (x, r),
dt

(2.11)

dX
= r X 2 that is, the normal form in equation (2.10).
dT
Furthermore: (0) = 1 and (0, 0) = 1 thus: X x and T t close to the origin.
such that (1.1) is transformed into

Note that this is the reason that this type of bifurcation is also known by the name of turning point bifurcation.

Rosales

Bifurcations: baby normal forms.

IMPORTANT: the definition for the transformed time is meant to be done along the solutions.
dT
That is: in the equation
= (x, r), x = x(t) is a solution of equation (1.1).
dt
Proof: Using the implicit function theorem, we see that f (x, r) = 0 has a unique (also smooth)
solution r = R(x) in a neighborhood of the origin: f (x, R(x)) 0,

which satisfies R(0) = 0. It

is easy to see that (dR/dx)(0) = 0 and (d2 R/dx2 )(0) = 2 also apply. Thus R = x2 (x)2 ,

where

is smooth and (0) = 1 this is the which appears in equation (2.11).


Because f (x, R(x)) 0, we can write f (x, r) = (x, r) (r R(x)) = (x, r) (r X 2 ),

where is

smooth and does not vanish near the origin in fact: (0, 0) = 1.
Define now = ( + x0 ) ,

where the prime indicates differentiation with respect to x. It is


dX
then easy to check that (0, 0) = 1, and that with this definition (2.11) yields
= r X 2.
dT
QED.

Problem 1 Implement a reduction to normal form, along lines similar to those used in theorem 1, by formally expanding the coordinate transformation up to O( 2 ) where  is as in equation
(2.9). To do so, write the dynamical system in the expanded form
dx
= |r
x2 + |a r x{z+ b x3} + O(4 ),
{z }
dt
O(3 )
O(2 )

(2.12)

where a and b are constants. Then expand the transformation


x = X + X 2 + O(3 ),
dt
= 1 + X + O(2 ),
dT

(2.13)
(2.14)

and find what values the coefficients and must take so that
dX
= r X 2 + O(4 ).
dT

(2.15)

This process can be continued so as to make the error term in (2.15) as high an order in  as
desired provided that f in (1.1) has enough derivatives. We point out here that: theorem 1

requires f to have only second order continuous derivatives to apply. By


contrast, the process here requires progressively higher derivatives to exist it, however, has the
advantage of giving explicit formulas for the transformation.

Rosales

Bifurcations: baby normal forms.

Transcritical bifurcations.

We now go back to the considerations in the introduction (section 1), and add one extra hypothesis
at the bifurcation point (x0 , r0 ) = (0, 0). Namely: we assume that there is a smooth

branch x = (r) of critical points that goes through the bifurcation point.
Taking successive derivatives of the identity f ((r), r) 0, and evaluating them at r = 0 (where
= f = fx = 0), we obtain:
fr (0, 0) = 0 and frr (0, 0) = 2 fxx (0, 0) 2 fxr (0, 0),

(3.16)

d
(0). As before, we assume that the coefficients for which we have no information are
dr
non-zero, and normalize (by scaling r and x in equation (1.1), if needed) so that: fxx (0, 0) = 2
where =

and frx (0, 0) = 1. Thus, at (x, r) = (0, 0), we have:


f = fx = fr = 0,

fxx = 2,

fxr = 1,

and frr = 2 a,

(3.17)

where a is a constant. In fact, (3.16) and (3.17), show that a = 2 = ( 1/2)2 1/4. Thus

1 + 4 a = (2 1)2 0. We do not know what the exact value of is, however, as usual (for
generality) we exclude the equal sign in this last inequality as too special. Thus:

Assume 1 + 4 a > 0.

(3.18)

As in the case of the Saddle Node bifurcation, the next step is to use (3.17) to expand the equation
(1.1). This yields:
dx
= r x x2 + a r 2 + O(x3 , r x2 , r 2 x, r 3 ).
dt

(3.19)

We now assume4 that both r and x are small, of size O(), where 0 <   1. Then,
keeping up to terms of O(2 ) on the right (leading order) in (3.19), we obtain the equation:

dx
= r x x2 + a r 2 = (x 1 r) (x 2 r),
(3.20)
dt



1
1
where 1 =
1 + 1 + 4 a and 2 =
1 1 + 4 a . In terms of the variables X = x 2 r
2
2
and R = 1 + 4 a r, this last equation takes the form:
dX
= R X X 2,
dt
which is the Normal form for a Transcritical bifurcation.
4

Compare this with (2.9).

(3.21)

Rosales

Bifurcations: baby normal forms.

Remark 7 The hypothesis 1 + 4 a > 0 in (3.18) is very important. For, write equation (3.19) in
the form:


dx
1
= x r
dt
2

2

1+4a 2
r + O(x3 , r x2 , r 2 x, r 3 ).
4

(3.22)

Then, if 1 + 4 a < 0, the leading order terms on the right in this equation would be a negative definite
quadratic form. This would imply that (x, r) = (0, 0) is the only critical point in a neighborhood of
the origin i.e. that (x, r) = (0, 0) is an isolated critical point. As explained in remark 4, such
points are (generally) of little interest.
On the other hand, 1 + 4 a = 0 would lead to a double root of the right hand side in (3.19) (at
leading order). In principle this can be interpreted as a limit case of the transcritical bifurcation,
where the two branches of critical points that cross at the origin, become tangent there. However:
this is an extremely structurally unstable situation, where the local details of what actually happens
are controlled by high order terms hence, again, this is a situation of little (general) interest.

Theorem 2 If the function f = f (x, r) is sufficiently smooth, the assumptions in equations


(3.17) and (3.18) guarantee that the f = f (x, r) = 0 has (exactly) two branches of solutions in
a neighborhood of the origin. Furthermore, let this branches be given by x = 1 (r) and x = 2 (r).
Then 1 (r) = 1 r + O(r 2 ) and 2 (r) = 2 r + O(r 2 ).
Sketch of the proof: The calculations leading to equations (3.19) and (3.20) show that:
f (x, r) = (x 1 r) (x 2 r) + O(x3 , r x2 , r 2 x, r 3 ).

(3.23)

f (x, r) = r 2 (X 1 ) (X 2 ) + r 3 O(X 3 , X 2 , X).

(3.24)

Let x = r X. Then

Thus, g = g(X, r) =

f
r2

satisfies:
g(X, r) = (X 1 ) (X 2 ) + r h(X, r),

(3.25)

where h is some non-singular function. We note now that:


6 0,
g(p , 0) = 0 and gX (p , 0) = (p q ) =

(3.26)

Rosales

Bifurcations: baby normal forms.

10

where {p, q} = {1, 2}. Then the implicit function theorem guarantees that there exist smooth
solutions X = Xn (r) to the equations:

g(Xn , r) = 0 and Xn (0) = n

where n = 1 or n = 2.

Then n = r Xn for n = 1, 2 are the two functions in the theorem statement.


Why are there no other solutions? Well, once we have 1 and 2 , we can write f = (x 1 )(x 2 ),
where = (x, r) does not vanish at the origin in fact: (0, 0) = fxx (0, 0) = 2.
QED.
The arguments made to obtain equations (3.17) and (3.18) depend on the existence of the smooth
branch of critical points x = (r). But the existence of this branch is not then used in the arguments
leading to the normal form (3.21). We explicitly exploit this existence in what follows below, and
use it to get a better handle on transcritical bifurcations. Thus, without loss of generality: 5

Assume that 0.
Then we can write f = x G(x, r),

(3.27)

where G(0, 0) = 0 since fx (0, 0) = 0. Other than this, we

assume that G is generic, so that its derivatives do not vanish. In particular, we normalize the
first order derivatives so that Gr (0, 0) = Gx (0, 0) = 1

this normalization is consistent with

the one used in (3.17), where we must take a = 0.


At this point we can invoke the implicit function theorem, that tells us that there is a function
x = z(r) such that G(z, r) = 0, with z(0) = 0 and dz/dr(0) = 1

note that, in this case 1 = 1

and 2 = 0. Again, we use this function to factor G in the form G = (x z(r)) H(x, r),
H(0, 0) = 1.

where

It follows then that we can write equation (1.1) in the form:


1 dx
= z(r) x + x2 .
H dt

Thus, if we introduce a new time T by dT /dt = H,

the equation is transformed into its Normal Form:

(3.28)

and change parameter6 r R = z(r),

dx
= R x x2 .
dT

(3.29)

The above is, clearly, the equivalent of theorem 1 for transcritical bifurcations:

a proof of the existence of a local transformation into normal form.


5

If needed, the change of variables x x will do the trick.

Note that, for r and x small, R r and T t. Thus both R and T are acceptable new variables.

Rosales

Bifurcations: baby normal forms.

11

Remark 8 Note that, because G above is generic, the situation is structurally stable. However, this depends on the assumption that there is a branch of solutions. Transcritical
bifurcations are not structurally stable without an assumption of this type.

Problem 2 Assume that equation (3.27), and the normalizations immediately below it, apply.
Then, for x and r both small and O() where 0 <   1 implement a reduction to normal
form, by formally expanding the coordinate transformation up to two orders in . To do so, write
the dynamical system in the expanded form
dx
= |r x {z
x2} + b0 x3 + b1 r x2 + b2 r 2 x + O(4 ),
|
{z
}
dt
2
3
O( )
O( )

(3.30)

where b0 , b1 , and b2 are constants. Then expand the transformation


dt
= 1 + 0 x + 1 R + O(2 ),
dT
r = R + R2 + O(3 ),

(3.31)
(3.32)

and find what values the coefficients 0 , 1 , and must take so that
dx
= R x x2 + O(4 ).
dT

(3.33)

This process can be continued so as to make the error term in (3.33) of arbitrarily high order in 
provided that f in (1.1) has enough derivatives. We point out here that: the derivation lead-

ing to equation (3.29) requires f to have only second order continuous


derivatives to apply. By contrast, the process here requires progressively higher derivatives
to exist it, however, has the advantage of giving explicit formulas for the transformation.

Problem 3 In problem 3.2.6 in the book by Strogatz, a process somewhat analogous to the one
in problem 2 is introduced. Basically, Strogatz tells you to do the following:
Consider the system
dx
= R x x2 + a x3 + O(x4 ),
dt

(3.34)

where R 6= 0 and a are constants. Introduce now a transformation (expanded) of the form
x = X + b X 3 + O(X 4 ),

(3.35)

Rosales

Bifurcations: baby normal forms.

12

where b is a constant. Then show that b can be selected so that the equation for X has the form
dX
= R X X 2 + O(X 4 ).
dt

(3.36)

Thus the third order power is removed. The process can be generalized to remove arbitrarily high
powers of X from the equation.

Question: This process is simpler than the one employed in problem 2: it involves neither transforming the independent variable t, nor the parameter R. Why is it not appropriate for

reducing an equation to normal form near a transcritical bifurcation?

Pitchfork bifurcations.

We now go back to the considerations in the introduction (section 1), and add two extra hypotheses
at the bifurcation point (x0 , r0 ) = (0, 0), one of them being the same one that was introduced in
section 3 for the transcritical bifurcations. Namely, we assume that:

A. There is a smooth branch x = (r) of critical points that goes through


the bifurcation point (0, 0).
B. The problem has right-left symmetry across the branch of critical
points x = (r). Specifically, there is smooth bijection x X = X(x, r),

valid in a neighborhood of the branch x = , such that:

Equation (1.1) is invariant under the transformation: X = f (X, r).


is a fixed curve for the transformation: X((r), r) = (r).
x < (r) = X > (r)

and

x > (r) = X < (r).

Without any real loss of generality, assume that f (x, r) is an odd function
of x. Then = 0, X = x, and (1.1) becomes:
dx
= x g(, r),
dt

where = x2.

(4.37)

Rosales

Bifurcations: baby normal forms.

13

The bifurcation condition (1.2) yields g(0, 0) = 0. Other than this, we assume that
g is generic. After appropriate re-scaling of the variables, we thus have
g(0, 0) = 0,

gr (0, 0) = 1,

and g (0, 0) = = 1.

(4.38)

Note that the sign of g (0, 0) cannot be changed by scalings!


Problem 4 Expand g in (4.37) in powers of and r. Show that, in a small neighborhood of the
origin (of appropriate shape see (2.9) and (3.19 3.20)), the leading order terms in the equation
dx
reduce to the normal form for a pitchfork bifurcation:
= r x + x3 .
dt

Problem 5 In a manner analogous to the ones in problems 1 (saddle-node bifurcations) and


2 (transcritical bifurcations) introduce new variables (via formal expansions) x X, t T , and
dX
r R, that reduce equation (4.37) to normal form:
= R X + X 3.
dT
HINT: First Expand g in a Taylor series g = r + + a2 r 2 + a1 r + a0 2 + . . . and substitute
this expansion into the equation. Second Assume an appropriate size scaling for the variables x
and r in terms of a small parameter 0 <   1. This scaling should be consistent with the normal
form for the equation.7 It is very important since it assures that the ordering in the expansions is kept
straight, without higher order terms being mixed with lower order ones. Third Introduce expansions

for R = R(r) = r + o(r) and dT /dt = H(x2 , r) = 1 + o(1). IMPORTANT: Notice that, because of
the symmetry in the equation, it must be that x = X and the expansion for dT /dt must involve even
powers of x only. Fourth Substitute the expansions in the equation, and select the coefficients to
eliminate the higher orders beyond the normal form. Carry this computation to ONE ORDER
ONLY: What are the dominant terms in the expansions, beyond R r and dT /dt 1.

Problem 6 Prove that a transformation with the properties stated in problem 5 actually exists
this in a way similar to the one used in theorem 1 for saddle-node bifurcations, and above equation
(3.29) for transcritical bifurcations.
HINT: Show that g(, r) = 0 has a solution of the form = R(r). Use this solution to factor g
as a product, and substitute the result into the equation. It should then be obvious how to proceed.
7

It is the same scaling required by problem 4.

Rosales

Bifurcations: baby normal forms.

14

Problem Answers.

The problem answers will be handed out with the answers to the problem
sets.

MIT OpenCourseWare
http://ocw.mit.edu

18.385J / 2.036J Nonlinear Dynamics and Chaos


Fall 2014

For information about citing these materials or our Terms of Use, visit: http://ocw.mit.edu/terms.

Bead moving along a thin, rigid, wire.


Rodolfo R. Rosales,

Department of Mathematics,

Massachusetts Inst. of Technology, Cambridge, Massachusetts, MA 02139


October 17, 2004
Abstract
An equation describing the motion of a bead along a rigid wire is derived. First the case
with no friction is considered, and a Lagrangian formulation is used to derive the equation.
Next a simple correction for the effect of friction is added to the equation. Finally, we consider
the case where friction dominates over inertia, and use this to reduce the order of the system.

Contents
1 Equations with no friction.

Principle of least action. . . . . . . . . . . . . . . . . . . . . . . . . . . . . . . . . .

Equation for the motion of the bead. . . . . . . . . . . . . . . . . . . . . . . . . . .

Bead on a vertical rotating hoop. . . . . . . . . . . . . . . . . . . . . . . . . . . . .

Problem 1.1: Bead on an horizontal rotating hoop. . . . . . . . . . . . . . . . . .

Wire on vertical plane, rotating around a vertical plane.

. . . . . . . . . . . . . . .

Problem 1.2: Wires rotating with variable rates. . . . . . . . . . . . . . . . . . . .

Parametric instabilities. . . . . . . . . . . . . . . . . . . . . . . . . . . . . . . . . .

The Euler Lagrange equation: derivation. . . . . . . . . . . . . . . . . . . . . . . . .

Problem 1.3: Hoop moving up and down in a vertical plane. . . . . . . . . . . . .

Parametric stabilization. . . . . . . . . . . . . . . . . . . . . . . . . . . . . . . . . .

Wire restricted to a fixed vertical plane. . . . . . . . . . . . . . . . . . . . . . . . .

2 Add friction to the equations.

Equation of motion with friction included. . . . . . . . . . . . . . . . . . . . . . . .

Add friction to the vertical rotating hoop. . . . . . . . . . . . . . . . . . . . . . . .

Wire restricted to a fixed vertical plane. . . . . . . . . . . . . . . . . . . . . . . . .

Wire moving rigidly up and down. . . . . . . . . . . . . . . . . . . . . . . . . . . . .

Sliding wires and friction. . . . . . . . . . . . . . . . . . . . . . . . . . . . . . . . .

Rosales

Bead moving along a thin, rigid, wire.

3 Friction dominates inertia.

10

3.1

Nondimensional equations. . . . . . . . . . . . . . . . . . . . . . . . . . . . . . . . .

10

3.2

Limit of large friction. . . . . . . . . . . . . . . . . . . . . . . . . . . . . . . . . . .

10

4 Problem Answers.

11

Equations with no friction.

Consider the motion of a bead of mass M moving along a thin rigid wire, under the influence of
gravity. Let (x, y, z) be a system of Cartesian coordinates, with z the vertical direction and z
increasing with height. Let g be the acceleration of gravity. Describe the wire in parametric form,
as follows:
x = X(s, t) ,

y = Y (s, t) ,

and z = Z(s, t) ,

(1.1)

where s is the arclength along the wire. Note that:


Because the wire is thin, we have approximated it as just a curve in space.
The wire is allowed to move and change shape as a function of time.
The bead will be approximated as a mass point.
We use the same length units for x , y , z, and s, so that: Xs2 + Ys2 + Zs2 = 1.
Furthermore: we will assume that the wire does not stretch as it moves. In particular, s is not only
arclength, but a material coordinate along the wire (i.e. s constant denotes always the same material
point on the wire). This will be important when we consider the effects of friction.
It is well known that, in the presence of rigid body constraints, the best way to formulate equations
in mechanics is by the use of Lagrangian mechanics that is: by using the principle of least action.

Remark 1.1 Of course, one can get the equations using the classical newtonian formulation.
What complicates things is that one must consider the unknown forces that arise to keep the
constraints valid. For example, in this case the bead experiences the force of gravity, and an unknown
force from the wire which must be exactly right to keep the bead moving along the wire (this is what
determines this force). When the wire is static, calculating this force is relatively easy. However,
when the wire moves, things get complicated.

Rosales

Bead moving along a thin, rigid, wire.

Remark 1.2 (Principle of least action). In its simplest form, the principle of least
action for a conservative system (no energy is dissipated) can be stated as follows: Let be a
mechanical trajectory, joining the points P1 and P2 in phase space. Parameterize this trajectory by
time t, with t = t1 the starting time (when the trajectory is at P1 ), and t = t2 the final time (when
P2 is reached). Consider now all the possible trajectories (curves) in phase space, starting at P 1 for
t = t1 , and ending at P2 for t = t2 is, of course, one member of this set. For any each such
curve the action L can be computed:
L=

t2

(Kinetic Energy Potential Energy) dt.

t1

Then the principle of least action says that is a stationary point for L within the set of
all possible trajectories described above.
We now apply this principle to our problem. Let the position of the bead along the wire be
given by s = S(t) .

Then:



1
M x 2 + y 2 + z 2
2
1
1
=
M S 2 + M (Xs Xt + Ys Yt + Zs Zt ) S + M (Xt2 + Yt2 + Zt2 )
2
2
1
=
M S 2 + S + ,
2
Potential Energy = g M Z ,

Kinetic Energy =

where: (A) = (S, t) = M (Xs Xt + Ys Yt + Zs Zt ) and = (S, t) =

(1.2)
(1.3)

1
M (Xt2 + Yt2 + Zt2 ) .
2

(B) X, Y , Z, and all their derivatives are evaluated at (S, t).


(C) The dots indicate (total) derivatives with respect to time.
(D) Since x = X(S, t), we have x = S Xs (S, t) + Xt (S, t)
with similar formulas for y and z.
(E) We have used that s is arclength, so that Xs2 + Ys2 + Zs2 = 1.
The action is then given by
L = L[S] =
where L =

1
M S 2 + S V ,
2

t2

L(S , S, t) dt ,

t1

with V = V (S, t) = g M Z .

(1.4)

Rosales

Bead moving along a thin, rigid, wire.

Because of the principle of least action, the Euler-Lagrange equation


d
dt

L
L

= 0,

S
S

(1.5)

must apply for the mechanical trajectory. Using the expression for L above, this yields the equation:
M

d2 S
V

=
(S , t)
(S , t) .
2
dt
S
t

(1.6)

This is the equation for the motion of a bead along a rigid moving wire,

under the action of gravity, when friction is neglected.


Example 1.1 When the wire moves normal to itself, 0. An example is a hoop
rotating around a diameter. Consider now the case with vertical rotation axis:
s
cos( t) ,
R
 
s
Y = +R sin
sin( t) ,
R
 
s
Z = R cos
,
R

X = +R sin

where R is the radius of the hoop and is its angular velocity. Then
=0

S
1
S
and V = gMR cos
MR2 2 sin2
R
2
R


(1.7)

where we notice that V is independent of time. The equation of motion is then:


d2 S
S
1
2S
= g sin
+ R2 sin
2
dt
R
2
R


Rewrite the equation using the angle =

S
that the bead makes with the vertical down-radius:
R

d2
1
= 2 sin + 2 sin 2 = (2 cos 2 ) sin ,
2
dt
2
where =

(1.8)

(1.9)

g/R is the angular frequency for a pendulum of length R. The two terms on the right

in equations (1.8 1.9) correspond to the effects of gravity and the centrifugal force.

Problem 1.1 Consider the case of a hoop rotating around an horizontal diameter.

Rosales

Bead moving along a thin, rigid, wire.

Example 1.2 A generalization of example 1.1 is that of a wire on a vertical plane, rotating
around a vertical axis. Then:
X = R(s) cos( t) ,
Y

= R(s) sin( t) ,

Z = Z(s) ,
where (R0 )2 + (Z 0 )2 = 1, and is the rotation angular velocity. Then
1
= 0 and V = gMZ(S) M 2 R2 (S),
2

(1.10)

where, again, V is independent of time. The equation of motion is then


d2 S
+ gZ 0 (S) 2 R(S )R0 (S) = 0.
dt2

(1.11)

Problem 1.2 Generalize the derivations in examples 1.1 and 1.2 to consider variable
rotation rates. That is to say: in the formulas describing the wires, replace t , where
= (t) is some function of time. Define = (t) = d/dt, then show that the ONLY

change in the governing equations (1.8), (1.9), and (1.11) is that is no longer
a constant, and is replaced by = (t), as defined above.
Example 1.3 Consider equation (1.9) for the case of a variable, and periodic, rotation rate
= (t). Assume that the amplitude of remains below for all times: 0 2 < 2 . In this
case, intuition would seem to indicate that 0 is a stable solution to the equation, with small
perturbations near it obeying the linear equation:
d2
+ ( 2 2 ) = 0.
dt2

(1.12)

However, it turns out that, when the period of the forcing is appropriately selected, it is possible
to make the solutions of (1.12) grow in time thus turning the solution 0 unstable. 1 This
phenomena is known as a parametric instability, and we will study it later when we cover

Floquet theory. When nonlinearity and dissipation are added to the system, chaotic behavior can
result from it.
1

This can happen even if the amplitude of is small.

Rosales

Bead moving along a thin, rigid, wire.

Remark 1.3 Derivation of the Euler-Lagrange equation (1.5).


Let S = S(t) be a small (but arbitrary) perturbation to the mechanical trajectory S, vanishing
at t1 and t2 . Let L be the corresponding perturbation to the action:
L = L[S + S] L[S] =

t2
t1
t2

t1
t2




L(S + S , S + S , t) L(S , S , t) dt


L2 (S , S , t) S + L1 (S , S , t) S dt + O( S 2 )

t1


d 
L2 (S , S , t)
L1 (S , S , t) S dt + O(S 2 ) ,
dt

L
L
, L2 =
, and we have integrated by parts in the third line. Because S is a

S
S
stationary point, the O(S) terms in L must vanish for all possible choices of S. It follows that
where L1 =


d 
L2 (S , S , t)
L1 (S , S , t) = 0 ,
dt

which is exactly the Euler-Lagrange equation (1.5).

Problem 1.3 Consider the case of a hoop that is restricted to move up and down on
a vertical plane. Derive the equation for the bead motion in this case. Parameterize the
wire in the form
X = R sin

s
,
R


Y 0,

and Z = R cos

s
+ A sin(t),
R


(1.13)

where R is the radius of the hoop (constant), is the (constant) angular frequency of the up-down
motions of the hoop, and A is the (constant) amplitude of the hoop oscillations. Then show that
the equation for the bead position, written in terms of the angle = S/R that the bead makes with
the bottom point on the hoop, is given by:

d2  2
2
+

sin(t)
sin = 0,
dt2

where 2 = g/R, and  = A/R

(1.14)

For small values of , the linearization of this equation is of the

same form as equation (1.12). Thus this is another example where parametric instability

can occur. Furthermore, consider the linearization of this equation near the equilibrium solution
bead on the top end of the hoop. Then, for = + and small, we can write:

d2  2
2

A
sin(t)
= 0.
dt2

(1.15)

Rosales

Bead moving along a thin, rigid, wire.

Intuitively we expect the solution to be unstable. However, it is possible to select the frequency
and amplitude of the up-down oscillations so that this solution becomes stable. This is an example
of parametric stabilization to be covered later in the course.

Example 1.4 Consider now2 a wire restricted to move on a fixed vertical plane so
that Y 0 in equation (1.1). Furthermore, assume that the wire position is given in the form
z = F (x, t) .

(1.16)

What is then the equation of motion, with the bead position given as x = (t)? To
find this equation, we could start from equation (1.6), and transform variables. This, however, is
quite messy. It is much easier to re-write the Lagrangian L in (1.4) in terms of the new variables,
and then use again the Euler-Lagrange equation. Clearly, we have:




1
1
1
2
2
2
L = M 1 + Fx + M Fx Ft gMF MFt = 2 + V ,
2
2
2

where F and its derivatives are evaluated at ( , t). Furthermore:


= ( , t) = M Fx Ft ,

and V = V ( , t) = gMF (1/2)MFt .


d
dt

(1.17)

= (, t) = M (1 + F x2 ),

The Euler-Lagrange equation

L
L

=0

then yields:
d
d

dt
dt

1
=
2

d
dt

!2

V t .

(1.18)

Add friction to the equations.

We want now to add the effects of friction to the equation of motion for the bead. The friction
laws for solid-to-solid contact are quite complicated, thus we will simplify the problem by assuming
that the wire is covered by a thin layer of lubricant liquid. In this case, an adequate model
is provided by: friction produces a force opposing the motion, of magnitude proportional

to the velocity of the bead relative to the wire. Since s is not just arclength, but a material
2

The example in problem 1.3 is of this type.

Rosales

Bead moving along a thin, rigid, wire.

Thus we
coordinate along the wire, the sliding velocity of the bead along the wire is simply S.
modify equation (1.6) to:
M

d2 S
dS V

(S , t)
(S , t) ,
2
dt
dt
S
t

(2.1)

where is the friction coefficient which we assume is a constant. This is the equation for

a bead moving along a lubricated rigid moving wire, under the influence
of gravity, with the friction force included.
Example 2.1 Add friction to the rotating hoop in example 1.1. It is easy to see that
equation (1.9) is modified to:
d2
d
+
+ ( 2 2 cos ) sin ,
2
dt
M dt

(2.2)

by the addition of friction. When the rotation rate is variable, = (t) see problem 1.2. More
generally, equation (1.11) for a rotating wire is modified to:
d2 S
dS
+
+ gZ 0 (S) 2 R(S)R0 (S) = 0.
2
dt
M dt

(2.3)

Example 2.2 Consider the situation in example 1.4 above. To add the effects of friction
when the equation of motion is written as in (1.18), we need to compute first what the velocity of
the bead is relative to the wire. This is not as simple as it may appear, for equation (1.16) does
not provide enough information as to the actual motion of the wire mass points for example:
the wire may have a fixed shape, with the points in the wire sliding along this shape. 3 Thus extra
information is needed. For example, let us assume that there is some material point in the

wire for which x = x0 is constant as the wire deforms and moves. In this case we can
proceed as follows:
ds2 = dx2 + dz 2 = (1 + Fx2 ) dx2 = 2 dx2 ,
S =
dS
dt

(x , t) dx ,

x0

d
= ( , t)
+
t (x , t) dx ,
dt
x0
!
d Z
F = ( , t)
+
t (x , t) dx ,
dt
x0

Think of a circular wire rotating around its center. See example 2.3.

=
(2.4)

Rosales

Bead moving along a thin, rigid, wire.

where = (x , t) =

1 + Fx2 (thus = M2 , and t = Fx Ft ) ,

and F is the magnitude

of the friction force. The magnitude of the component of the friction force along the x-axis is
thus given by F /. This component must oppose the inertial force along the x-axis, which is given
by M . Since = M 2 , we conclude that equation (1.18) must be modified as follows:
!

d
d
1

+ F =
dt
dt
2

d
dt

!2

V t ,

(2.5)

where F = F (,
, t) is given by equation (2.4). A particularly simple instance of this occurs in the

case of a wire moving rigidly up and down. In this case we can write, for F in
(1.16),
z = F (x , t) = f (x) + h(t) .

(2.6)
q

= (, t) = Mf 0 ()h(t),
= () = 1 + (f 0 ())2 ,
= () = M 1 + (f 0 ())2 ,


1
2

V = V (, t) = gM f () gM h(t) + M (h(t))
, and F = . Thus, the equation of motion
2
can be written in the form:
Then



d
d
d 1 0

+
+ f () g + h(t)
.
dt
dt
M dt

(2.7)

Example 2.3 Sliding wires and friction. We consider two examples. First a horizontal
straight wire sliding at a variable speed. Thus:
X = s + u(t),

and Y = Z = 0.

(2.8)

du
dv
be the wire speed, and a =
be the wire acceleration. Then the formulas above equadt
dt
1
1
tion (1.6) yield: = M v, = M v 2 , and V = Mv 2 . Thus, from equation (2.1)
2
2
Let v =

d2 S
dS
+
+a=0
2
dt
M dt

d2

+
2
dt
M

d
v = 0,
dt

(2.9)

where = S + u. The second example is that of a horizontal circular wire rotating around
its center. Thus
X = R cos ,

Y = R sin ,

and Z = 0,

(2.10)

d
where R is the radius of the hoop, = s/R + , and = (t) indicates the wire motion. Let =
dt
S
be the wire angular velocity. Then equation (2.1) yields, for = + ,
R
d2

+
dt2
M

d
= 0.
dt

(2.11)

Rosales

Bead moving along a thin, rigid, wire.

10

Friction dominates inertia.

3.1

Nondimensional equations.

We consider equation (2.1), for a simple situation where the equation is autonomous (no time
dependence). Thus, the equation takes the form
M

d2 S
dS
+

= F (S),
dt2
dt

(3.1)

where F is the force driving the bead along the wire (produced by gravity and the wire motion).
Assume now that F varies on some characteristic length scale, and has a typical size. Thus we can
write
S
F = F0
L


(3.2)

where F0 is a constant with force dimensions, L is a constant with dimensions of length, and is
nondimensional and has O(1) size, with an O(1) derivative. We introduce nondimensional variables
as follows:
=

S
L

and =

F0
t,
L

(3.3)

L
is the time scale that results from the balance of the applied forces and the viscous forces.
F0
In terms of these variables, the equation becomes:
where

where  =

3.2

MF0
L 2

d2 d
+
= (),
d 2
d

(3.4)

is a nondimensional parameter.

Limit of large friction.

If the inertial forces are negligible relative to the viscous forces, to be precise, if 0 <   1

in

(3.4), the inertial terms in the equation can be neglected. This leads to:
d
= ().
d

(3.5)

We point out that it is, generally, dangerous to neglect terms that involve the highest
derivative in an equation even if the term is multiplied by a very small parameter.

Rosales

Bead moving along a thin, rigid, wire.

11

In the particular case here, this can be entirely justified, 4 but (in general) one must be
very careful.

Problem Answers.

The problem answers will be handed out separately, with the solutions to problem sets
and exams.

This will be done later in the course.

MIT OpenCourseWare
http://ocw.mit.edu

18.385J / 2.036J Nonlinear Dynamics and Chaos


Fall 2014

For information about citing these materials or our Terms of Use, visit: http://ocw.mit.edu/terms.

18.385 MIT
Hopf Bifurcations.

Rodolfo R. Rosales
Department of Mathematics
Massachusetts Institute of Technology
Cambridge, Massachusetts MA 02139
September 25, 1999
Abstract

In two dimensions a Hopf bifurcation occurs as a Spiral Point switches from stable
to unstable (or vice versa) and a periodic solution appears. There are, however, more
details to the story than this: The
to unstable spiral

fact that a critical point switches from stable

(or vice versa)

solution will arise,

alone does not guarantee that a periodic

though one almost always does. Here we will explore these

questions in some detail, using the method of multiple scales to nd precise conditions
for a limit cycle to occur and to calculate its size. We will use a second order scalar
equation to illustrate the situation, but the results and methods are quite general and
easy to generalize to any number of dimensions and general dynamical systems.

1 Extra

conditions have to be satis ed. For example, in the damped pendulum equation: x + x_ +sin x = 0,

6 0!
there are no periodic solutions for  =

18.385 MIT (Rosales) Hopf Bifurcations.


Contents

1 Hopf bifurcation for second order scalar equations.

1.1
1.2
1.3
1.4
1.5
1.6
1.7

Reduction of general phase plane case to second order scalar. . . . . . . .


Equilibrium solution and linearization. . . . . . . . . . . . . . . . . . . .
Assumptions on the linear eigenvalues needed for a Hopf bifurcation. . .
Weakly Nonlinear things and expansion of the equation near equilibrium.
Explanation of the idea behind the calculation. . . . . . . . . . . . . . .
Calculation of the limit cycle size. . . . . . . . . . . . . . . . . . . . . . .
The Two Timing expansion up to O(3). . . . . . . . . . . . . . . . . . .
Calculation of the proper scaling for the slow time. . . . . . . . .
Resonances occur rst at third order. Non-degeneracy. . . . . . .
Asymptotic equations at third order. . . . . . . . . . . . . . . . .
Supercritical and subcritical Hopf bifurcations. . . . . . . . . . . .
1.7.1 Remark on the situation at the critical bifurcation value. . . . . .
1.7.2 Remark on higher orders and two timing validity limits. . . . . . .
1.7.3 Remark on the problem when the nonlinearity is degenerate. . . .

.
.
.
.
.
.
.
.
.
.
.
.
.
.

. 3
. 3
. 4
. 5
. 5
. 6
. 7
. 7
. 8
. 8
. 9
. 9
. 10
. 10

18.385 MIT (Rosales) Hopf Bifurcations.


1

Hopf bifurcation for second order scalar equations.

1.1

Reduction of general phase plane case to second order scalar.

We will consider here equations of the form


x + h(x;
_ x; ) = 0 ;

(1.1)

where h is a smooth and  is a parameter.


Note 1

There is not much loss of generality in studying an equation like (1.1), as

opposed to a phase plane general system. For let:

x_ = f (x; y; ) and y_ = g (x; y; ) :

(1.2)

x = fx x_ + fy y_ = fx f + fy g = F (x; y; ) :

(1.3)

Then we have
Now, from x_ = f (x; y; ) we can, at least in principle,2 write

y = G(x;
_ x; ) :

(1.4)

Substituting then (1.4) into (1.3) we get an equation of the form (1.1).3
1.2

Equilibrium solution and linearization.

Consider now an equilibrium solution4 for (1.1), that is:


x = X () such that h(0; X; ) = 0 ;
2 We

can do this in a neighborhood of any point (x ; y )

(say,a critical point)

(1.5)
such that fy (x ; y ; ) 6= 0,

as follows from the Implicit Function theorem. If fy = 0, but gx 6= 0, then the same ideas yield an equation
of the form y + eh(y;
_ y; ) = 0 for some eh. The approach will fail only if both fy = gx = 0. But, for a
critical point this last situation implies that the eigenvalues are fx and gy , that is:

both real

! Since we are

interested in studying the behavior of phase plane systems near a non{degenerate critical point switching
from stable to unstable spiral behavior, this cannot happen.
3 Vice versa, if we have an equation of the form (1.1), then de ning y by y = G(x;
_ x; ), for any G such
that the equation can be solved to yield x_ = f (x; y; ) (for example: G = x_ ), then y_ = Gx_ x + Gx x_ = g (x; y )
upon replacing x_ = f and x = h.
4 i.e.:

a critical point.

18.385 MIT (Rosales) Hopf Bifurcations.

so that x  X is a solution for any xed . There is no loss of generality in assuming


X ()  0 for all values of  ;

(1.6)

since we can always change variables as follows: xold = X () + xnew.


The linearized equation near the equilibrium solution x  0 (that is, the equation for x
in nitesimal) is now:
x 2 x_ + x = 0 ;
(1.7)
where = () =

1
2

hx_ (0; 0; ) and = () = hx (0; 0; ) :

The critical point is a spiral point if > 2. The eigenvalues and linearized solution
are then
 =  i!
(1.8)
e

(where ! = 2) and
e

x = ae t cos (!e (t t0 )) ;

(1.9)

where a and t0 are constants.


1.3

Assumptions on the linear eigenvalues needed for a Hopf bifurcation.

Assume now: At  = 0 the critical point changes from a stable to an unstable spiral
point (if the change occurs for some other  = c, one can always rede ne old = c + new).

Thus

< 0 for  < 0 and > 0 for  > 0, with > 0 for  small.

In fact, assume:

 I. h is smooth.
d
(0) > 0 :5
 II. (0) = 0; (0) > 0 and d

9
>
=
>
;

(1.10)

We point out that, in addition, there are some restrictions on the behavior of
the nonlinear terms near the critical point that are needed for a Hopf bifurcation
to occur. See equation (1.22).
5 This

last is known as the Transversality condition. It guarantees that the eigenvalues cross the imaginary

axis as  varies.

18.385 MIT (Rosales) Hopf Bifurcations.


1.4

Weakly Nonlinear things and expansion of the equation near


equilibrium.

Our objective is to study what happens

near the critical point, for  small. Since for  = 0 the

critical point is a linear center, the nonlinear terms will be important in this study. Since
we will be considering the region near the critical point, the nonlinearity will be weak.
Thus we will use the methods introduced in the Weakly Nonlinear Things notes.
For x; x_ , and  small we can expand h in (1.1). This yields

 + !02x +
+

_ + Bxx
_ + 12 Cx2 +
_ + 3Ex_ 2x + 3F x_ x2 + Gx3
2 _ +
x + O(4; 2; 2)
= 0;

1 Ax2
2
n
1 Dx3
6
p2 x

(1.11)

where we have used that h(0; 0; )  0 and (0) = 0. In this equation we have:
@
h(0; 0; 0) = (0) > 0, with !0 > 0,
@x
@2
@2
A = 2 h(0; 0; 0); B =
@x_
@x@
_ x h(0; 0; 0) ; : : :,
1 @ 2 h(0; 0; 0) = d (0) > 0, with p > 0,
p2 =
2 @x@
_ 
d
@2
d

= @x@
h(0; 0; 0) = (0),
d
 is a measure of the size of (x; x_). Further: both  and  are small.

A. !02 =
B.
C.
D.
E.
1.5

Explanation of the idea behind the calculation.

of (1.11). The idea is, again: for  and  small the


solutions are going to be dominated by the center in the linearized equation x + !02x = 0,
with a slow drift in the amplitude and small changes to the period6 caused by the higher
order terms. Thus we will use an approximation for the solution like the ones in section 2.1
of the Weakly Nonlinear Things notes.
We now want to study the solutions

6 We

will not model these period changes here. See section 2.3 of the

how to do so.

Weakly Nonlinear Things notes for

18.385 MIT (Rosales) Hopf Bifurcations.


1.6

Calculation of the limit cycle size.

An important point to be answered is:

What is epsilon?

(1.12)

This is a parameter that does not appear in (1.1) or, equivalently, (1.11). In fact, the only
parameter in the equation is  (assumed small as we are close to the bifurcation point  = 0).
Thus:
 must be related to .
(1.13)
In fact,  will be a measure of the size of the limit cycle, which is a property of the
equation (and thus a function of  and not arbitrary all).
However: We do not know  a priori! How do we go about determining it?
The idea is: If we choose  \too small" in our scaling of (x; x_), then we will be looking

\too close" to the critical point and thus will nd only spiral-like behavior, with no limit
cycle at all. Thus, we must choose  just large enough so that the terms involving
 in (1.11) (speci cally 2p2 x_ , which is the leading order term in producing the stable/unstable spiral behavior) are \balanced" by the nonlinearity in such a fashion that
a limit cycle is allowed. In the context of Two{Timing this means we want  to \kick
in" the damping/ampli cation term 2p2 x_ at \just the right level" in the sequence of
solvability conditions the method produces. Thus, going back to (1.11), we see that7

 The linear leading order terms x + !02x appear at O().


 The rst nonlinear terms (quadratic) appear at O(2).

However: Quadratic terms produce no resonances, since sin2  = (1 cos 2) and
2
there are no sine or cosine terms. The same applies to cos2  and to
sin  cos .
Thus, the rst resonances will occur when the cubic terms in x play a role ) we must
have the balance
O(x3 ) = O(x_) ;
(1.14)

)  = O(2).
7 This

is a crucial argument that must be well understood. Else things look like a bunch of miracles!

18.385 MIT (Rosales) Hopf Bifurcations.


1.7

( )

The Two Timing expansion up to O 3 .

We are now ready to start. The expansion to use in (1.11) is


x = x1 (; T ) + 2 x2 (; T ) + 3 x3 (; T ) + : : : ;

(1.15)

where 0 <   1, 2{periodicity in T is required, T = !0t, !0 is as in (1.11)8,  is a slow


time variable and  is related to  by  = 2 , where  = 1 (which  we take depends
on which \side" of  = 0 we want to investigate).
What exactly is  ? Well, we need  to resolve resonances, which will not occur until the cubic
terms kick in into the expansion )  = 2t: (This is exactly the same argument used to
get (1.14)).
Then, with 0 = @T@ , (1.11) becomes:
n

!02x00 + !02x + 12 A!02 (x0 )2 + B!0 xx0 + 12 Cx2 +


1
3 0 3
2 0 2
0 2
3
6 fD!0 (x ) + 3E!0 (x ) x + 3F !0 x x + Gx g +
22!0x0 22p2 !0x0 + 2 
x + O(4) = 0 :

(1.16)

The rest is now a computational nightmare, but it is fairly straightforward. Without


getting into any of the messy algebra, this is what will happen:
At O()

!02 fx001 + x1 g = 0 : Thus


x1 = a1 ( )eiT + c:c:

(1.17)

for some complex valued function a1( ). We use complex notation, as in the Weakly Nonlinear Things notes.
At O(2 )

!02 fx002 + x2 g + |fquadratic terms


in x1 and x01 g} = 0 :
{z

(1.18)

From the rst bracket in (1.16), the quadratic terms here have the form:
C1 a21 ei2T + C2 a12 + C1 (a1 )2 e 2iT ;


where C1 and C2 are constants that can be computed in terms of !0, A, B and C .
Since the solution and equation are real valued, C2 is real. Here, as usual,  indicates the
complex conjugate.
8 Same

as the linear (at  = 0) frequency. No attempt is made in this expansion to include higher order

nonlinear corrections to the frequency.

18.385 MIT (Rosales) Hopf Bifurcations.


No resonances occur and we have
x2 =
At O(3 )



a2

( )eiT

+ 13 !0 2C1a12 ei2T + c:c:




!02 (x003 + x3 ) + 2!0 x01

!0 2C2 a21 :

(1.19)

2p2 !0x01 + 
x1 + CNLT = 0 ;

(1.20)

where CNLT stands for Cubic Non Linear Terms, involving products of the form x2 x1 ,
x02 x1 , x2 x01 , x02 x01 , (x10 )3 , (x10 )2 x1 , x10 x21 and x13 . These will produce a term of the form
da21 a1 eiT + c:c: plus other terms whose T dependencies are: 1, e2iT and e3iT , none of
which is resonant (forces a non periodic response in x3). Here
d is a constant that can be computed in terms of !0 ; A; B; C; D; E; F and G :

(1.21)

This is a big and messy calculation, but it involves only sweat. In general, of course,
Im(d) 6= 0. The case Im(d) = 0 is very particular, as it requires h in equation (1.1)to be just
right, so that the particular combination of its derivatives at x = 0, x_ = 0 and  = 0 that
yields Im(d) just happens to vanish. Thus
Assume a nondegenerate case: Im(d) 6= 0 :

(1.22)

For equation (1.20) to have solutions x3 periodic in T , the forcing terms proportional to eiT
must vanish. This leads to the equation:
2!0i dd a1 2p2!0 ia1 + 
a1 + d a21 a1 = 0:


Then write

(1.23)

a1 = ei ; with  and  real ;  > 0 :

This yields
and
where q = 12 !0 1p 2Im(d).

1
d
1
 = !0 1
+ !0 1 Re(d)3
2
d
2

(1.24)

d
 = p2 (1 q2 ) ;
d

(1.25)

18.385 MIT (Rosales) Hopf Bifurcations.

Equation(1.24) provides a correction to the phase of x1 , since x1 = 2 cos(T + ). The


rst term on the right of (1.24) corresponds to the changes in the linear part of the phase
due to  6= 0, away from the phase T = !0t at  = 0. The second term accounts for the
nonlinear e ects.
The second equation (1.25) above is more interesting. First of all, it recon rms that for
 < 0 (that is,  = 1) the critical point ( = 0) is a stable spiral, and that for  > 0 (that
is,  = 1) it is an unstable spiral. Further
If Im(d) > 0: Then a stable limit cycle exists for
 > 0 (i.e.  = 1) with  = 2!0 p2 (Im(d)) 1 :
q

Supercritical (Soft) Hopf Bifurcation.

If Im(d) < 0: Then an unstable limit cycle exists for


 < 0 (i.e.  = 1) with  =
2!0p2 (Im(d)) 1 :
q

Subcritical (Hard) Hopf Bifurcation.

9
>
>
>
>
>
>
>
>
>
>
>
>
>
>
>
>
>
>
=
>
>
>
>
>
>
>
>
>
>
>
>
>
>
>
>
>
>
;

(1.26)

Notice that  here is equal to 21 the radius of the limit cycle.
1.7.1 Remark on the situation at the critical bifurcation value.

Notice that, for  = 0 (critical value of the bifurcation parameter)9 we can do a two timing
analysis as above to verify what the nonlinear terms do to the center.10 The calculations are
exactly as the ones leading to equations (1.23){(1.25), except that  = 0 and  is now a small
parameter (unrelated to , as  = 0 now) simply measuring the strength of the nonlinearity
near the critical point. Then we get for  = 21 radius of orbit around the critical point
d
1
 = !0 1 Im(d)3 :
d
2

(1.27)

From this the behavior near the critical point follows.


9 Then the critical point is a center in the linearized regime.
10 This

is the way one would normally go about deciding if a linear center is actually a spiral point and

what stability it has.

18.385 MIT (Rosales) Hopf Bifurcations.

Clearly

8
>
>
>
>
>
>
<
>
>
>
>
>
>
:

10

 Im(d) > 0 () Soft bifurcation () Nonlinear terms stabilize.


For  = 0 critical point is a stable spiral.

 Im(d) < 0 () Hard bifurcation () Nonlinear terms de-stabilize.


For  = 0 critical point is an unstable stable spiral.

1.7.2 Remark on higher orders and two timing validity limits.

As pointed out in the Weakly Nonlinear Things notes, Two Timing is generally valid for some
\limited" range in time, here probably j j   1. This is because we have no mechanism
for incorporating the higher order corrections to the period the nonlinearity produces. If
we are only interested in calculating the limit cycle in a Hopf bifurcation (not it's stability
characteristics), we can always do so using the Poincare{Lindsteadt Method. In particular,
then we can get the period to as high an order as wanted.
1.7.3 Remark on the problem when the nonlinearity is degenerate.

What about the degenerate case Im(d) = 0 ?


In this case there may be a limit cycle, or there may not be one. To decide the question
one must look at the e ects of nonlinearities higher than cubic (going beyond O(3) in the
expansion) and see if they stabilize or destabilize. If a limit cycle exists, then its size will
not be given by jj, but something else entirely di erent (given by the appropriate balance
between nonlinearity and the linear damping/ampli cation produced by 6= 0 when  6= 0
in equation (1.7)). The details of the calculation needed in a case like this can be quite hairy.
One must use methods like the ones in Section 2.3 of the Weakly Nonlinear Things notes
because: even though the nonlinearity may require a high order before it decides the issue of
stability, modi cations to the frequency of oscillation will occur at lower orders.11 We will
not get into this sort of stu here.
q

11 Note

that Re(d) 6= 0 in (1.24) produces such a change, even if Im(d) = 0 and there are no nonlinear

e ects in (1.25).

MIT OpenCourseWare
http://ocw.mit.edu

18.385J / 2.036J Nonlinear Dynamics and Chaos


Fall 2014

For information about citing these materials or our Terms of Use, visit: http://ocw.mit.edu/terms.

18.385 MIT
Weakly Nonlinear Things: Oscillators.

Rodolfo R. Rosales
Department of Mathematics
Massachusetts Institute of Technology
Cambridge, Massachusetts MA 02139
October 24, 1995

Abstract

When nonlinearities are \small" there are various ways one can exploit this
fact | and the fact that the linearized problem can be solved exactly1 | to produce

useful approximations to the solutions.


We illustrate two of these techniques here, with examples from phase plane
analysis: The Poincare{Lindstedt method and the (more exible) Two Timing
method. This second method is a particular case of the Multiple Scales approxima-

tion technique, which is useful whenever the solution of a problem involves e ects that
occur on very di erent scales. In the particular examples we consider, the di erent
scales arise from the basic vibration frequency induced by the linear terms (fast scale)
and from the (slow) scale over which the small nonlinear e ects accumulate.
The material in these notes is intended to amplify the topics covered in
section 7.6 and problems 7.6.13{7.6.22 of the book \Nonlinear Dynamics
and Chaos" by S. Strogatz.
1 Actually,

one can also use these ideas when one has a

nonlinear

problem with known solution, and

wishes to solve a slightly di erent one. But we will not talk about this here.

18.385 MIT (Rosales) Weakly Nonlinear Things: Oscillators.

Contents
1 Poincare-Lindstedt Method (PLM).
General ideas behind the method.

1.1 Dung Equation. . . . . . . . . . . . . . . . . . . . . . . . . . . . . . . . . .

1.2 van der Pol equation. . . . . . . . . . . . . . . . . . . . . . . . . . . . . . . .

Periodic solutions and amplitude dependence of their periods.

Calculation of the limit cycle.

2 Two Timing, Multiple Scales method (TTMS)


for the van der Pol equation.

2.1 Calculation of the limit cycle and stability. . . . . . . . . . . . . . . . . . . .

2.2 Higher orders and limitations of TTMS. . . . . . . . . . . . . . . . . . . . .


This topic is fairly technical.y

11

2.3 Generalization of TTMS to extend the range of validity. . . . . . . . . . . . .


This topic is fairly technical.y

14

A Appendix.

16

A.1 Some details regarding section 1.1. . . . . . . . . . . . . . . . . . . . . . . .

16

A.2 More details regarding section 1.1. . . . . . . . . . . . . . . . . . . . . . . . .


This topic is fairly technical.y

17

A.3 Some details regarding section 1.2. . . . . . . . . . . . . . . . . . . . . . . .

17

y The material here is for completeness, but not actually needed to get a "basic" understanding.

18.385 MIT (Rosales) Weakly Nonlinear Things: Oscillators.

1 Poincare-Lindstedt Method (PLM).


PLM is a technique for calculating periodic solutions. The idea is that, if the linearized

equations have periodic solutions and 0 <   1 is a measure of the size of the nonlinear terms
then:

I. For any nite time period t0  t  t0 + Tf (Tf > 0), the trajectories for the full
system will remain pretty close to those of the linearized system (errors no worse than

O(Tf ), typically).

II. On the other hand, even a small error is enough to destroy periodicity. An orbit that
\closes on itself" after some time period, will generally fail to do so if slightly perturbed.
Thus, typically, nonlinearity will destroy most periodic orbits the linearized system might
have. Some, however, may survive2

! PLM is designed to pick those up.

Even if a periodic orbit of the linearized system survives:

III. The nonlinearity will change (slightly) the shape of the orbit.
IV. The speed of \travel" along the orbit will be a ected by the nonlinearity. In
particular the period will change (slightly.)
PLM takes care of these e ects as follows:
A. The solution is approximated at leading order by the linear solution, but small corrections at higher orders are introduced to take care of the (small) shape changes.

B. The linear solution is evaluated at a stretched time, to account for the change in period.
The two examples that follow illustrate the ideas.

1.1 Dung Equation.


The equation can be written in the form

x + x + x3 = 0 ;
2 That

(1.1)

is, if ~u = ~u(t) is a periodic solution of the linearized system, then so is a~u, for any scalar constant

a. But for only a few values of a will periodicity \survive" the e ect of the nonlinearity.

18.385 MIT (Rosales) Weakly Nonlinear Things: Oscillators.


where 0 < 

1 and  =

1.

This equation is actually a conservative system, with

(conserved) energy

1
1
1
(1.2)
2
2
4
Thus all orbits for x bounded will be periodic.3 PLM will allow us to calculate corrections

E = x_ 2 + x2 + x4 :

to the linear period of 2 and sinusoidal orbit shape (for the bounded orbits).
The PLM expansion is given by:

x(t) = x0 (T ) + x1 (T ) + 2 x2 (T ) +    ;

(1.3)

where xj = xj (T ) is periodic of period 2 in T and does not depend on . T = !t is the

stretched time variable, where


! = 1 + !1 + 2 !2 +    ;

(1.4)

is a (real, positive) constant to be computed. The nonlinear period is then 2=! .

Note 1 x (T ) will be the solution to the linearized problem, so (1.3) will reduce to the
0

right answer when  = 0.

We now proceed as follows:

 First:
0=

d
dT

Rewrite (1.1) in terms of the new independent variable T , replacing  =

via

d
dt

by

d
dt

= ! dTd . Thus:

! 2 x00 + x +   x3 = 0 :

 Second: Substitute (1.3) and (1.4) into (1.5) and collect equal powers

(1.5)
4

of . Then require

that the equation be satis ed at each level in . Thus we get an equation for each order p ,
which determine higher and higher orders of approximation in the expansion (1.3), as follows.
3 Notice

that,

for

=1

ALL orbits are periodic.

However, for  = 1, orbits where jxj > 

1
2

are

not periodic. This follows from looking at the level curves for E in the (x; x_ ) phase plane. Of course, when

jx j = O (

1
2

), the nonlinear term in equation (1.1) has the same size as the linear terms: the problem is no

longer \weakly nonlinear". Thus, we should not be surprised if the solution exhibits behavior not close to
the linearized one.
4 This is the messy

part.

It means you have to plug (1.3) and (1.4) into (1.5), then do all the products,

etc. . . . so as to end with the equation written as:

f  g +  f  g + 2 f  g +    = 0.

18.385 MIT (Rosales) Weakly Nonlinear Things: Oscillators.


O(1) equation:

x000 + x0 = 0 :

(1.6)

x0 = a cos T ;

(1.7)

Clearly then
where a is, at this stage, an arbitrary constant.5
O() equation:
x00 + 2!1 x00 + x1 + x3 = 0,
1

that is:

x001 + x1 = 2!1 a cos T a3 cos3 T =


o
n
= 2!1 a 34 a3 cos T 14 a3 cos 3T :

(1.8)

The form of equation (1.8) is typical of all the higher order equations.
Namely, we get the linear equation for the new term in x at that order | x1
here | forced by terms involving the lower orders already solved for.
The solution x1 to (1.8) will be 2 -periodic in T only if the coecient of the cos T term on
the right hand side (terms between the brackets) vanishes. This is because this term will
produce a response in x1 proportional to T sin T , which is clearly not periodic. Since we

are interested in a nontrivial solution (that is a 6= 0) we conclude that:

!1 =
x1 =

a2 ;
1
a3 cos 3T + A
cos T {z
+ B sin T} ;
32
|
3
8

(1.9)

where the term marked by the brace in the second equation is the arbitrary homogeneous
solution, with A and B arbitrary constants. The rst equation here determines the rst
frequency correction, in terms of the amplitude6 of the oscillations a, which remains arbitrary
at this level.7 We note also that the homogeneous solution in the second equation above
5 In

fact, in this case, a will remain arbitrary. There is also a

phase shift we could include in (1.7).

But

this is just a matter of where we put the time origin (see appendix A.1).
6 This

7 That

is typical of nonlinear oscillators: the frequency depends on the amplitude.


is, no restrictions have been imposed by the expansion on it. In fact, it can be shown that no

restrictions on a will appear at any level of the expansion. This is because there is in fact a whole one
parameter set of periodic solutions, which can be parameterized by the amplitude a.

18.385 MIT (Rosales) Weakly Nonlinear Things: Oscillators.

amounts to no more than a small change in the amplitude and phase of the leading order
solution. That is:

! (a + A) cos T + B sin T = a cos(T T ) ;

a cos T

for some ae and Te . Thus (see appendix A.1)

Without Loss of Generality: we can set A = B = 0 in (1.9).


x002 + 2!1 x001 + (2!2 + !12 ) x000 + x2 + 3x02 x1 = 0,

O(2 ) equation:


x002 + x2 = 2!2 + !12 a cos T +

9
! a3 cos 3T
16 1

(1.10)
that is:

3 5 2
a cos T cos 3T ;
32

(1.11)

where cos2 T cos 3T = 41 cos T + 21 cos 3T + 41 cos 5T . Again: x2 will be periodic only if the
coecient of the cos T forcing term on the right hand side here vanishes. This yields

!2 =

1 2
3 4
!1 +
a =
2
256

15 4
a
256

(1.12)

and an explicit formula for x2 , which we do not display here. Clearly, this process can be

carried to any desired order (see appendix A.2).


In summary, we have found for the solutions8 of the Dung equation:
x

= !t ;

1
a cos T + 32 a3 cos 3T + O (2 ) ;
3
1 + 8 a2

(1.13)
15 2 4
3
256  a + O ( ) :

1.2 van der Pol equation.


The equation has the form

x  (1 x2 )x_ + x = 0 ;

(1.14)

where 0 <   1 and  = 1. We use now the same ideas of section 1.1, so that (1.3) and
(1.4) still apply. Instead of (1.5) we get now

! 2x00 + x ! (1 x2 )x0 = 0 :


8 Notice that this is valid only as long as 0  a2   1 . When jaj = O (

(1.15)
1
2

), the \corrections" cease to

be smaller than the leading order and the expansion fails. This agrees with our observations in footnote 3.

18.385 MIT (Rosales) Weakly Nonlinear Things: Oscillators.

We proceed now to look at the expansion order by order.

At O(1) we get, as before (see appendix A.3):


x0 = a cos T:
O() equation:

x001 + 2!1 x000 + x1

(1.16)

 (1 x20 )x00 = 0,

that is:

x00 + x1 = 2!1 a cos T a sin T + a3 cos2 T sin T




= 2!1 a cos T + a 14 a2 1 sin T + 41 a3 sin 3T :

(1.17)

To get a periodic solution x1 , both the coecients of cos T and sin T must vanish on the
right hand side =) For a nontrivial solution (a 6= 0) we must have9 :

a = 2 ; !1 = 0 and x1 =

1 3
a sin 3T + A
cos T {z
+ B sin T} :
|
32

(1.18)

Note 2 There is an important di erence here with the situation in the analog equations

(1.8) and (1.9). Now both sines and cosines appear on the right hand side of equation (1.17).

Thus we end up with TWO conditions that must be satis ed if equation (1.17) is to have
a periodic solution for x1 . These conditions are generally called Solvability Conditions.
Thus now BOTH a and !1 are determined. There is NO FREE PARAMETER left and
there is just one periodic orbit: the LIMIT CYCLE.
Since now a is xed to be a = 2, we can no longer argue that by a slight change in
the amplitude and phase of x0 , we can set A = B = 0 (homogeneous part of the solution,
marked by the brace above), as we did in (1.10). It is still true, however, that the phase of
the leading order x0 can be changed slightly. We can then use this to conclude (see appendix
A.3)

Without Loss of Generality: we can set B = 0 in (1.18).

(1.19)

On the other hand, we point out that A remains to be determined. That is, the circular
part of the limit cycle orbit does not have a radius exactly equal to 2, but rather equal to
2 + A + : : :
9 We

could take a = 2 also. This, however, is just a phase change T

assume a > 0.

! T + .

Thus, we may as well

18.385 MIT (Rosales) Weakly Nonlinear Things: Oscillators.

At the next order (that is, O(2 )) we will get an equation of the form:

x002 + x2 = Forcing :

(1.20)

Again (see note 3) sine and cosine forcing terms on the right will have to be eliminated.
This will produce two conditions, that will determine both A and !2 uniquely. In x2 an
homogeneous term of the form cos T will appear,10 with and !3 determined at O(3 ).
And so on to higher and higher orders.
A unique solution, up to a phase shift, is produced
this way to all orders:

The LIMIT CYCLE.

(1.21)

Note 3 In fact, after some calculation | using (1.16), (1.18) and (1.19) | we can see

that (1.20) is:

x002 + x2 =

1
2!2 + 128
a4 a cos T +

a2 1 A sin T
3 3
5
a (2 a2 ) cos 3T + 43 Aa2 sin 3T + 128
a5 cos 5T :
64
3
4

(1.22)

Thus we conclude

!2 =

1 4
3
a ; A = 0 and x2 = cos T + a3 (2 a2 ) cos 3T
256
512

5 5
a cos 5T ; (1.23)
3072

where we recall that a = 2.

2 Two Timing, Multiple Scales method (TTMS)


for the van der Pol equation.

2.1 Calculation of the limit cycle and stability.


In section 1.1 we basically obtained all the solutions to the Dung equation (1.1) |
since we ended up with two free parameters: the amplitude a and an arbitrary phase shift

!T T.
0

On the other hand, in section 1.2 we only obtained the limit cycle solution.

Now, suppose we want all the solutions to the van der Pol equation (1.14) | this will
10 With

a \ sinT " homogeneous part of the solution eliminated just as above in (1.19)

18.385 MIT (Rosales) Weakly Nonlinear Things: Oscillators.

allow us to determine, in particular, the stability of the limit cycle. The method we introduce
in this section (TTMS) will allow us to do this.
The main idea is that, if the solution is not periodic, then we cannot represent it

with a single solution of the linearized equation (as we did in section 1, with its time
dependence stretched by ! from t to T = !t | to allow for nonlinear corrections to the
period.11 ) For any \short" time period this will be O.K., but over long periods large errors
may result because they accumulate. To resolve this diculty we will allow ALL the

parameters of the linear solution to change SLOWLY in time, so as to track the


true evolution of the solution. Thus, for equation (1.14), we expand12 :
x  x0 (; t) + x1 (; t) + 2 x2 (; t) +    ;

(2.1)

where t takes care of the \normal" 2 -periodic dependence induced by the linear solution
and  = t is the slow time (that will allow the linear solution being used to drift (slowly)
as time evolves, from one linear orbit to the next.13 )

Remark 1 Note that now the solution depends explicitly on two times, thus the name
for the method. In this case the \slow" time is  = t, but in other problems it may be

 = 2 t | or something else. Figuring out what the exact dependence should be need not be
trivial and usually requires some thinking: it is related to the rate at which the nonlinearity
causes drift in the orbits | as opposed to just shape changes. We will talk about this later.
We now rewrite equation (1.14) in terms of the increased set of \independent" variables

 and t to obtain (here a dot indicates di erentiation with respect to t ):


x + 2x_  + 2 x  + x  (1 x2 )x_ 2  (1 x2 )x = 0 :

(2.2)

Note that the equation is now a P. D. E. ! This method appears to complicate things! How-

ever, the extra terms are multiplied by  and 2 and so at leading order we only get the linear
O. D. E. In fact: we will only have to solve linear O. D. E.'s at each order in the approximation!
11 Namely:

the orbits in phase space are quite close to the linear ones, but the speed at which they are

tracked is slightly di erent =) Over long times a big error will accumulate, unless we correct for it.
12 This

13 This

is only a rst, very simple, implementation. We will introduce a more re ned one in section 2.3.
description, strictly, only applies to x0 above. The higher order terms x1 : : : are there to account

for the fact that the nonlinear orbits will have slightly di erent shapes than the linear ones.

18.385 MIT (Rosales) Weakly Nonlinear Things: Oscillators.

10

As usual, we now substitute the expansion (2.1) into equation (2.2) and collect equal

powers of  to obtain

O(1) equation:

x000 + x0 = 0 :

(2.3)

This is the same as in section 1.2, except that now the arbitrary \constants" in the solution
of (2.3) will depend on  . We thus have

x0 = A0 ( )eit + c:c: ;

(2.4)

where c:c: denotes complex conjugate and A0 is complex valued.

Remark 2 Alternatively, we could write x

ib).
We cannot now argue, as we did before, that it is O.K. to set b = 0 using the fact that a
change of time origin t ! t + t0 is allowed. This is because t0 has to be constant, while
setting an arbitrary b( ) to zero would require t0 = t0 ( ), at least in principle.14
0

= a( ) cos t + b( ) sin t, where A = 12 (a

Remark 3 The use of complex notation in (2.4) makes life simpler. The kind of expansions we are doing require at each level of approximation that one expand things like x30 in

Fourier modes. This is much easier to do with exponentials than with sine and cosines!

At O() we obtain:
x1 + x1 =
=

2x_ 0  +  (1
2i

d
d

A0

x02 )x_ 0

1
A0 1
2

jA j
0



eit

iA30 e3it + c:c: :

(2.5)

This equation is very similar to (1.17), except that now: (i) We are using complex notation, (ii) There is no !1 term and (iii) A new term in

A0 appears because of the allowed


 dependence. The solution x1 will be periodic in t provided the coecient of the eit forcing
d
d

on the right hand side of (2.5) vanishes. This yields the equation
d
d

14 Actually,

A0 = 12  1

jA j A ,
0

(2.6)

an argument to set b = 0 can be made, namely: we expect the solutions of equation (1.14) to

be basically oscillatory. Thus, they will have maximums and minimums. If we set t = 0 to occur at a local
maximum, then x_ = 0 at t = 0, which yields b = 0. But this argument will not work at higher orders.

18.385 MIT (Rosales) Weakly Nonlinear Things: Oscillators.

11

which governs the evolution15 of the amplitude A0 for the linear (circular) orbits under the
e ect of the weak nonlinearity.
If we let A0 = 21 aei' , where a and ' are a real amplitude and phase, respectively, then16

d
d
1
' = 0 and
a =  (4 a2 )a :
d
d
8

(2.7)

These formulas show that the orbits in the phase plane are nearly circular, with a slowly changing
radius a that evolves following the second equation in (2.7) and a limit cycle for a = 2. In
particular:

For  = 1 the limit cycle is stable and it is unstable for  = 1:

(2.8)

If we let  =  in (1.14) and write the equation as

x (1 x2 )x_ + x = 0 ;

(2.9)

then we see that our calculations here show that at  = 0 we have a bifurcation, with an
exchange of stability between the limit cycle and the critical point at the origin.

 < 0. Unstable limit cycle and stable spiral point.


 > 0. Stable limit cycle and unstable spiral point.
 = 0. Center with continuoum of periodic orbits. (There is no limit cycle.)

(2.10)

2.2 Higher orders and limitations of TTMS.


We us now nish the O() calculation and solve equation (2.5) using (2.6). We have

x1 =


1 3 3it
iA0 e + A1 ( )eit + c:c: ;
8

(2.11)

where A1 is complex valued.


Let us now continue the expansion to one more order, as there is an important detail to

be learned from doing this.


15

Drift in phase space

16 Since

this shows that ' is a constant, we could have taken b = 0 in remark 2 !

18.385 MIT (Rosales) Weakly Nonlinear Things: Oscillators.

12

The O(2 ) equation is:


x2 + x2 =
=
where 0 =

d
d

x0  + x_ 1 + x0 x20 x_ 1 2x0 x1 x_ 0 x20 x0



2i A01 12 A1 +  jA20 j A1 + 12 A20 A1 + 12 iA00 21 iA000
0 + 1 i jA4 j A  eit + (: : :)e3it + (: : :)e5it o + c:c: ;
1
2
i
(
j
A
j
A
)
0
0
0
0
2
16

2x_ 1

(2.12)

and A1 denotes the complex conjugate of A1 . Thus, to avoid secular terms

in x2 (namely: terms proportional to t eit , that destroy the periodicity in t) the coecient
of eit on the right hand side of this last equation must vanish. Thus

A01



1
1
A1 +  A20 A1 + A20 A1 =
2
2

1 0 1 00 1  2 0
iA + iA + i A0 A0
2 0 2 0 2

1 4
i A A : (2.13)
16 0 0

This is a rather messy equation. We do not aim to solve it here; but only to analyze its behavior
for  large.

Assume  = 1: In this case the limit cycle is stable and, for  large | see equation
(2.7) | A0  ei' , for some constant '. Then equation (2.13) reduces to
1
2

1
2

1 i'
ie :
16

(2.14)

1  i'
i e ;
16

(2.15)

A01 + A1 + e2i' A1 =


This is much simpler and can be solved explicitly17


A1 = C1 e + iC2


where C1 and C2 are real constants. This means that the solution of equation (2.13)

will behave, for large  , like

1 i'
ie :
(2.16)
16
This is \bad". Notice that the expansion (2.1) for the solution of (1.14) | use equations

A1 

(2.4) and (2.11) | is




x  2 Re A0 ( )ei





1
 Im A30 ( )e3it + 2 Re A1 ( )eit +    :
4

But, when  = O(1) the second term in the expansion will not be small at all (as

A1 

1
16

iei' ) ! Thus
The two timing expansion (2.1) is only valid as long as j j   1 .

17 Write

A1 = zei' . Then z 0 + Re(z ) =

1
16 i.

(2.17)

18.385 MIT (Rosales) Weakly Nonlinear Things: Oscillators.

13

This is pretty typical for TTMS expansions: Usually they are valid for a time range
where the \slow" time can be taken large | but not arbitrary large. Beyond some  p , for
some p, they fail.
In the current situation (2.17) is not terribly upsetting. It still allows us to take  fairly

large. Once  is large and the limit cycle is reached =) can switch to the expansion in section
1.2 !!

However: suppose that (2.17) makes us terribly unhappy, for whatever reasons. Then
Can we x the problem posed by (2.17)?

(2.18)

The answer to this question is YES, but rst we must understand why (2.17)

occurs! This is clari ed next; for simplicity we CONSIDER ONLY the STABLE
LIMIT CYCLE case, when  = 1.

Note 4 Equations (2.1){(2.7) lead to an approximation of the limit cycle (for large  , so
that A0  ei' ) given by

x  2 Re(ei(t+') ) = 2 cos(t + ') :

(2.19)

On the other hand, the PLM calculation of section 1.2 tells us that we should use

x  2 cos(!t + ') = 2 Re(ei(!t+') ) ;


where ! = 1

 +   . Now, since (expand in Taylor series)

1 2
16

ei(!t+') = ei(t+') e

1 2 t+
i 16

 = ei(t+')

1 2 i(t+ ' )
i te
+ ;
16

(2.20)

i2 tei(t+') +    ; which is precisely the \bad" behavior


arising in A1 earlier in equation (2.16). Thus
we see that the error in (2.19) is

1
16

The TTMS expansion goes bad because it does not properly take into account
the fact that the nonlinearity a ects the phase | i.e. the position along the

(2.21)

linear orbit of the solution.

 It follows that, to achieve (2.18) we must x the problem pointed out by (2.21).
THIS WE DO NEXT.

18.385 MIT (Rosales) Weakly Nonlinear Things: Oscillators.

2.3 Generalization of TTMS to extend the range of validity.

14

Let  be the phase of the solution | namely: its position along the orbit | and ! = dtd 
its angular velocity. The phase increases with time and, for the linearized equation, we

have

d
 = ! = 1:
(2.22)
dt
However, once nonlinear e ects kick in, there is no reason for ! to remain equal to 1,
or in fact even constant!
Now, when considering a periodic orbit, as long as ! is approximated by its correct
average value things will be O.K. (as then errors will not accumulate over time). This is
what PLM does, by taking  = T = !t with ! = 1 + !1 +    . We cannot use this
idea of PLM in TTMS, because now the orbit (thus the average value of ! ) varies slowly
as time changes. We must then allow ! to be a function of  . Thus
To x the type of problem discussed in the previous section 2.2 we must replace
the expansion (2.1) by a subtler type, where the phase (fast time) itself is to be determined.
Generally we must deal then with expansions of the form

x  X0 (; ) +  X1 (; ) + 2 X2 (; ) +    ;

(2.23)

where 2 {periodic dependence on the phase  is required,  = t and

d
 = ! = 1 +  ! 1 ( ) +  2 ! 2 (  ) +    :
dt
This amounts to writing:  = 1 ( +  1 ( ) + 2 2 ( ) +   ), where

d
d

j = !j .

When no  dependence is allowed, this reduces to PLM. We will not carry out the details of
this expansion here | they are quite messy and some technicalities are involved in selecting
the !j 's so that the Xj 's behave \properly" as functions of  (that is, no secular growth in 
occurs). On the other hand, in the particular case of the van der Pol equation (1.14), when
the limit cycle is stable18 : all solutions eventually approach the limit cycle, and they do so
on time scales where 



(as follows from our results in section 1.2). Thus, as long as

no cumulative errors occur in tracking the limit cycle, there should be no problems. We can
conclude thus, without doing any calculations, that:
18 That

is,  = 1.

18.385 MIT (Rosales) Weakly Nonlinear Things: Oscillators.

15

For equation (1.14), in the case  = 1:

 The ! 's in equation (2.23) are constant and equal to the values calculated for the
j

expansion in section 1.2.

 The functional form of X (; ) in equation (2.23) is the same as that we obtained
0

for x0 (; t) in equation (2.1), with t replaced by . That is: X0 (; ) = x0 (; ).
In particular, note that from this we learn that the TTMS approximation for the behavior

of the van der Pol equation is quite good. The secular growth displayed by A1 in equation
(2.16) for very long times is nothing to worry about. It is simply a manifestation of the
fact that we have some small (very small, O(2 )) errors on the velocity at which the solution
moves along the limit cycle, but of nothing else. No important qualitative or quantitative

e ect is missing.

Note 5 Other ways to x the problem in (2.17) can be devised. For example, some people
advocate introducing ever slower time scales, such as 2 t, 3 t and so on | in addition to
the t of equation (2.1). This is not a good idea, unless the problem truly depends

on that many scales! For example: if the diculty arises because the true slow time
dependence19 is on something like (say) 1+2 t and not t, then this \lots of scales" approach
will just complicate things for no real gain at all. For an expansion to be useful, it has to
zero into the real behavior of the solution. The aim of doing an asymptotic expansion should
be to learn something useful about the solution, not to produce a massive amount of algebra
(even if this is, sometimes, an unfortunate byproduct, it is not the aim). In particular,
producing an \approximation" that fools us into believing that the solution depends on very
many di erent time scales (when in fact it does not), is exactly opposite to this objective.
19 Notice

that the van der Pol equation is exactly an example of this type.

18.385 MIT (Rosales) Weakly Nonlinear Things: Oscillators.

16

A Appendix.

A.1 Some details regarding section 1.1.


Generally, asymptotic expansions | like the ones in these notes | require at each level
the solution of a linear equation with some forcing made up from the prior terms. The
solution of this linear equation is then required to satisfy some condition (periodicity in the
examples here) and this imposes restrictions on the forcing terms. These restrictions are
then used to determine free parameters, slow time evolutions, etc.
When solving the linear equations in the expansion, it is very important to include in the
solution ALL the free parameters consistent with the conditions imposed on the solution.
This is because parameters that are \arbitrary" at some level, may later be needed to satisfy
the restrictions at a higher order.20 Failure to include a particular parameter | which boils
down to setting it to some arbitrary xed value | will typically cause trouble at higher
order, when a restriction on a forcing term will be found impossible to satisfy.
On the other hand, practical considerations dictate that we carry as few free parameters
in a calculation as feasible. Thus, one must always look at the equations involved and ask
if there is some argument that would allow for the elimination of a parameter | but never
must one eliminate a parameter without a good reason.21

Consider now equation (1.1) | or (1.5). This equation is invariant under time translation: if x = X (t) is a solution, then so it is x = X (t t0 ). Thus, we can always pick the
origin of the time coordinate to simplify the solution and eliminate parameters.

For example: The general solution of (1.6) is: a cos(T T0 ), where a and T0 are constants.
But the invariance under time translation shows that we can set T0 = 0.
Furthermore: At the level of (1.9) we know that in fact a is arbitrary. Then, since A and
B in (1.9) amount to making small O() changes to a and T0 at the O(1) level | thus they
are not true \new" free parameters | we can again set A = B = 0, as in (1.10), without
any fear.
20 For

example, in section 1.2, the amplitude a in (1.16) is eventually set to a = 2 in (1.18).


if an expansion fails at some level, one should always check to see if somehow an important

21 Conversely:

degree of freedom (some parameter) was ignored!

18.385 MIT (Rosales) Weakly Nonlinear Things: Oscillators.

17

In fact, the same argument shows that we can conclude:


At any level O(n) in the expansion, for n > 1, we can

(A.1)

take xn in (1.3) with NO cos T or sin T components.

A.2 More details regarding section 1.1.


It is clear that, in the expansion of section 1.1, the O(n ) equations | for n > 1 | have the
form

x00n + xn = Pn (x0 ; : : : ; xn 1 )

n
X

` x00n ` ;

(A.2)

`=1

where Pn is a cubic polynomial and the ` 's are constants de ned by ! 2 =

1 ` . Thus
`=0 `

0 = 1, 1 = 2!1 , 2 = 2!2 + !12 , 3 = 2!3 + 2!1!2 , . . . . In general we can see that


n = 2!n + fn (!1 ; : : : ; !n 1 ), where fn is a quadratic polynomial.
Because x0 is even, the forcing on the right hand side of (1.8) is also even. Then (1.10)
gives x1 even. The same type of argument shows then that x2 is also even. More generally,
one can show using (A.1) that all the xn 's are even.
Now, the condition on (A.2) to get xn periodic in T is that the right hand side should
not have any forcing proportional to either sin T or cos T . But the right hand side is even,
thus there is NO sin T forcing ever. On the other hand, the coecient of the cos T forcing
has the form: 2a!n + Gn (a; !1 ; : : : ; !n 1), where Gn is some polynomial function. Thus, one
can always choose !n so as to make the coecient of cos T vanish. We have thus
shown that
The expansion in equation (1.3) works up to any order.

(A.3)

A.3 Some details regarding section 1.2.


Equation (1.14) is invariant under time translation. Thus, just as we did in appendix A.1,
we have a phase to play with and can use to eliminate parameters.
We used this fact in (1.16) to eliminate the sine component in x0 (T ). But now a is no
longer a free parameter in the solution, as equation (1.18) shows that a = 2. Thus, in order
to eliminate spurious parameters in x1 (T ) (from the two { A and B { that appear in (1.18)),
we only have a phase to play with.

18.385 MIT (Rosales) Weakly Nonlinear Things: Oscillators.


Since 2 cos(T

18

B ) = 2 cos T + B sin T + : : :, it follows that a small phase change can be


used to eliminate B in x1 (T ) as given in (1.18). But A cannot and should not be eliminated
from the formula. In fact, at O(2 ) the solvability requirement on the equations (periodicity
of x2 (T )) will determine A in the same fashion that a = 2 followed from the O() equation.
At this level it will be possible to argue that no term in sin T is needed in x2 (T ), but a
term cos T must be kept (with determined at O(3 )). Clearly the same pattern will be
repeated over and over. In this fashion the expansion can be continued to any desired
order.
1
2

MIT OpenCourseWare
http://ocw.mit.edu

18.385J / 2.036J Nonlinear Dynamics and Chaos


Fall 2014

For information about citing these materials or our Terms of Use, visit: http://ocw.mit.edu/terms.

Various lecture notes for 18385.


R. R. Rosales.
Department of Mathematics,
Massachusetts Institute of Technology,
Cambridge, Massachusetts, MA 02139.
September 17, 2012
Abstract
Notes, both complete and/or incomplete, for MITs 18.385 Nonlinear Dynamics and Chaos.

Contents
1 Lecture Notes Fall 2012.
1.1 Lecture # 01, Thu. Sep. 06.

. . . . . . . . . . .
Introduce the notion of phase space. . . . . .
Introduce the notion of a well posed problem. .
Well posed versus chaotic. . . . . . . . . . .
Classifications of models. . . . . . . . . .
Notion of phase portrait. . . . . . . . . . . .
Canonical forms. . . . . . . . . . . . . . .

1.2

.
.
.
.
1.1.1
.
.
1.1.2
.
Delay-difference and integro-differential equations. .
Lecture # 02, Tue. Sep. 11. . . . . . . . . . . . . . . .
Inverse function theorem. . . . . . . . . . . . . . .
Implicit function theorem. . . . . . . . . . . . . .

1.2.1

.
.
.
.
.
.
.
.
.
.
.
Existence and uniqueness for the I.V. problem for ode. .
Lipschitz continuity. . . . . . . . . . . . . . . . . . .
Example of an ode with a discontinuity. . . . . . . . . .
The linearization metatheorem. . . . . . . . . . . .

List of Figures

.
.
.
.
.
.
.

.
.
.
.
.
.
.

.
.
.
.
.
.
.
.
.
.
.

.
.
.
.
.
.
.
.
.
.
.
.
.
.
.

.
.
.
.
.
.
.
.
.
.
.
.
.
.
.

.
.
.
.
.
.
.
.
.
.
.
.
.
.
.

.
.
.
.
.
.
.
.
.
.
.
.
.
.
.

.
.
.
.
.
.
.
.
.
.
.
.
.
.
.

.
.
.
.
.
.
.
.
.
.
.
.
.
.
.

.
.
.
.
.
.
.
.
.
.
.
.
.
.
.

.
.
.
.
.
.
.
.
.
.
.
.
.
.
.

.
.
.
.
.
.
.
.
.
.
.
.
.
.
.

.
.
.
.
.
.
.
.
.
.
.
.
.
.
.

.
.
.
.
.
.
.
.
.
.
.
.
.
.
.

.
.
.
.
.
.
.
.
.
.
.
.
.
.
.

.
.
.
.
.
.
.
.
.
.
.
.
.
.
.

.
.
.
.
.
.
.
.
.
.
.
.
.
.
.

2
2
2
3
4
5
5
6
6
7
7
7
7
8
8
8

Rosales, MIT (Math.).

Various lecture notes for 18385.

Lecture Notes Fall 2012.


[vln385:LN12]

This is a short summary of (some of) the lectures from the fall of 2012.

1.1

Lecture # 01, Thu. Sep. 06.


[vln385:LN12:L01]

Students are expected to read, and be familiar, with the contents of chapter #1 in the
textbook (by Strogatz). The material here is meant as a supplement to that in the book.
The notion of a phase space P is a crucial one in mathematical modeling in science. To make a
mathematical model of some physical system, one needs to identify parameter/quantities that can
be used to fully describe the system.1 The set of all possible values that can be taken by these
parameters is the phase space P this set can often be given some geometrical structure, hence
the space in the name. Some examples are:
1. Coin tossing. P = {u, d}, where u = coin up and d = coin down.
2. Pendulum. P = cylinder.
3. Classical Mechanical system of unconstrained point particles in a d-dimensional Euclidean
space. P = set of all the possible positions and velocities = R2d .
4. Quantum Mechanical system of unconstrained point particles in a d-dimensional Euclidean
space. P = set of all the complex valued functions defined on Rd , with unit square integral =
unit sphere in L2 (Rd ).
When fully describing a system the aim is not to necessarily describe the full system, but
whatever is relevant (e.g.: in item 2 we ignore the color of the pendulum, the air motion around
it, etc). We also may ignore effects that are not important to the level of accuracy to which we
care to describe the system (e.g.: in item 2 we ignore the fact that the pendulum bar is not rigid
hence its length is not a constant). In fact, the same system may be associated with more than
one P, depending on how much detail is included (e.g.: compare items 3 and 4).
What we mean by fully describe the system is a rather vague, unless one introduces some mathematics. Then one can make it precise by stating that it means that: The resulting mathematical
model is well posed (what well posed means is defined below). One cannot tell if a phase space is
appropriate without simultaneously having the mathematical model that goes with it. This may
seem a little circular and not very satisfactory, but the making of a mathematical model for a
science or engineering problem is not a plug into recipe and turn the crank process. It is hard,
1

Below we define what fully describe means more precisely. For now, we leave this at the intuitive level.

Rosales, MIT (Math.).

Various lecture notes for 18385.

and requires originality and a well oiled human brain to work. Think of how long it took to arrive
at Newtonian Mechanics, or of how hard was the phase space switch when formulating Quantum
Mechanics.
A mathematical problem is well posed if it has a unique solution that depends continuously on any
parameters present in the problem. Examples:
5. Initial value problem for a linear homogeneous and constant coefficients ode system.2 That
is: ~y = A ~y , with ~y (0) = ~y0 where ~y is vector valued and A is a constant square matrix.
The (unique) solution to this problem is ~y = eA t ~y0 , which clearly depends continuously (in
fact, analytically) on all the parameters in the problem: the entries of A and ~y0 .
6. Consider the boundary value problem u0 0 + m u = 0 for 0 < x < 1, with u(0) = u(1) = 0.
Does this have a solution u? When a solution exists, is it unique? Does the solution, if any,
depend continuously on m?
Could this be a model for a question of the type: what is the shape of a string hanging from
two supports, one at each end?, where the shape of the string is given by u = u(x)?
Note also that one could have a model in which the question is: What are the values of
m for which there is a solution? (i.e.: an eigenvalue problem), which leads to different
considerations.
7. Neumann problem for Poisson in some bounded region R2 . That is: T = f in , with
n T = 0 along .
7.1 Problem as steady state heat distribution for an insulated thin plate, heated/cooled by
some source.
Z
7.2 Argue, physically, that there cannot be a steady state unless:
f dV = 0 (the net heat

input vanishes).
7.3 Use Gauss theorem to mathematically derive the condition in item 7.2.
This problem is well posed, provided that one restricts the sources by

f dV = 0.

Note that a problem of this type arises for the pressure, at each time step, in some Zincompressible Navier Stokes solvers. Then numerical errors guarantee that the condition f dV = 0

is never quite satisfied. A way around this problem is to look for a T that is as close to a
solution as possible (in some appropriate sense), which then yields a well posed problem.
q

8. The initial value problem x = 2 |x|, with x(0) = 0. This has at least two solutions:3 x = 0
and x = t2 . Thus, it is an ill-posed problem.
2
3

Later we will state a theorem that generalizes this to very general initial value problems for ode systems.
In fact, it has infinitely many.

Rosales, MIT (Math.).

Various lecture notes for 18385.

Consider the situation of a cylindrical bucket filled with water up to a height h = h(t), and
being emptied through a small hole at the bottom. Assume that the bucket is empty at time
t = 0, and write the equation for x = x(t) = h(t). This gives rise to an equation similar
to the one above. This should not be a surprise, since it should be obvious that we cannot
determine h(t) for t < 0 unless the time at which the bucket finished emptying is known.
Of course, being well posed is not enough. The model must give results and predictions that
match experiments and observations, with some reasonable4 degree of accuracy. But it is needed.
An ill posed (i.e.: not well posed) model is useless!
Remark 1.1.1 Well posed versus chaotic. In connection to chaos many of you may have heard
statements such as stepping on a butterfly in China may produce a tornado in Kansas. We will
consider later the extent to which this can be taken seriously. For now, however, notice that this
implies a situation where very small changes can produce huge outcomes! Does this contradict the
continuous dependence on parameters requirement for a well posed problem? Does this mean
that a problem in which chaos occurs is not well posed?
The answers is a most emphatic NO! When dealing with an initial value problem, well posed
means that: As the size of a perturbation to the initial conditions vanishes, the size of the resulting
perturbation to the solution also vanishes at any fixed time. The solution can have arbitrarily large
changes as t . Chaos is tied up to fast growth of the errors with time, not with errors going
from infinitesimal to finite in arbitrarily short times (which is what lack of continuity means).
When a model is discretized and programed into a computer, the following things can happen:
9. The original model is ill posed, but the computer program behaves well. This, of course,
means that whatever the computer is computing is not related to the physical reasoning
that led to the model! This is, clearly, not desirable. Hence, it is important to understand
the model, and not rely solely on a computer implementation.
10. The original model is OK, but the computer implementation behaves badly. This means: bad
discretization. Numerical analysis aims to address (avoid) this problem. Important, but not
the focus of this class.
11. The original model is OK, and the computer implementation behaves well. But they have
little to do with each other. Again: issue dealt with by numerical analysis.
Since it is rather impossible to do numerical analysis for a system that is poorly understood,
understanding is needed in all cases. This class is aimed at providing tools for understanding.5
4

This depends on the intended use.


Of course, one can get understanding from numerical computations, and use this to correct/improve the calculations, and so on. These two aspects are not independent.
5

Rosales, MIT (Math.).


1.1.1

Various lecture notes for 18385.

Classifications of models.
[vln385:LN12:L01:CLA]

Models/problems can be classified into:


Static.
Example: what is the shape of a hanging cord?
Dynamic.
Example: vibrations of a string.
In this course we concentrate on dynamics. The time evolution in a dynamic problem can be:
Random.
Example: coin tossing.
Deterministic.
Example: orbital mechanics.
In this course we concentrate on deterministic problems. The time evolution can be:
Discrete.
Example: generations in a biological system.
Continuum.
Example: car moving along a road.
For a continuum dynamical system, the solutions can be interpreted as curves in phase space. A
main objective of this course is to describe the topology of these curves: how does the phase
portrait look like? That is: we are not seeking quantitative information, but mainly qualitative
information. This approach was proposed by Poincare when, towards the end of the 19-th century,
it became clear that exact solutions were possible for only a very limited number of problems. In
general, the best one could hope for (analytically) was qualitative information.6
Further classifications:
Linear.
Define. Example: harmonic oscillator.
Non-linear.
Define. Example: Navier-Stokes equations.
Because linear systems are vulnerable to a divide-and-conquer strategy, they are much better
understood than non-linear systems. In fact, notice that non-linear systems are defined by not
being linear, in other words: but what they are not. This (very poor) definition is an indication
of how little we understand them, in general though there are some types of non-linear systems
for which we know more (e.g.: completely integrable Hamiltonian systems).
Finite dimensional.
Examples: ode.
Infinite dimensional. Examples: pde.
In this course we will concentrate on finite and low [1, 2, 3] dimensional systems, governed by ode.
Why only this? Because we understand 1-D and 2-D systems fairly thoroughly, and we have some
understanding of 3-D (though not complete). Beyond that, most of what we know results from
situations where a higher dimensional system displays behavior that can be understood in terms
of low dimensional concepts. This is not intended to trivialize the techniques involved in analyzing
6

Of course, with computers, we can now hope both for quantitative as well as qualitative information. However,
this does neither make the problems trivial, nor eliminates the need for deep and intelligent thinking. When (and
if) AI truly arrives, this may change. But we are not there yet.

Rosales, MIT (Math.).

Various lecture notes for 18385.

such systems, which are covered in other courses, but we must draw the line somewhere.
Check the Dynamical view of the world section by Strogatz, with the frontier in the dimension
versus nonlinearity diagram. Obviously the diagram is incomplete, and many things are missing,
but it illustrates well where the boundary between known and unknown roughly is.
1.1.2

Canonical forms.
[vln385:LN12:L01:CAN]

Continuum, finite dimensional, deterministic, dynamical systems governed by ode can be written
in the canonical form
d~y
= F~ (~y ),
(1.1.1)
dt
where ~y and F~ are d-vector valued, and F~ is some function. Then d is the dimension of the
system. Explained in the lecture:
How time dependent coefficients can be eliminated by adding t as an extra unknown, with
the extra equation t = 1.
How higher order derivatives can be eliminated by adding derivatives as extra unknowns.
How, after the above, one must be able to solve for ~y (at least in principle) and write the
equation as above: A time evolution ode problem where the time derivative is not uniquely
determined by the present cannot be well posed.
How, for a constrained system (e.g.: pendulum) the form above will apply (at least locally)
once an appropriate (maybe local) coordinate system is selected incorporating the constraints.
Generally, this applies to systems where the phase space P is some finite dimensional manifold.
Similarly, finite dimensional, deterministic, dynamical discrete systems can be written in the form
~yn+1 = F~ (~yn ),

(1.1.2)

where ~yn P and F~ is some function from P to P. If P is a d-dimensional manifold, then d is the
dimension of the system. Explained in the lecture:
How to reduce systems of the form ~xn+1 = F~ (~xn , ~xn1 ) to the form in (1.1.2) by letting
~yn = (~xn , ~xn1 ). Similarly for systems of the form ~xn+1 = F~ (~xn , ~xn1 , ~xn2 ), etc.
How to reduce systems of the form ~xn+1 = F~n (~xn ) to the form in (1.1.2) by introducing n as
an extra unknown.
Remark 1.1.2 Delay-difference and integro-differential equations. Some mathematical models
in science and engineering involve delay-difference and integro-differential equations. In general
such equations (even when they involve a finite number of unknowns which are functions of a single
variable: time) should not be considered as finite dimensional.

Rosales, MIT (Math.).

1.2

Various lecture notes for 18385.

Lecture # 02, Tue. Sep. 11.


[vln385:LN12:L02]

Consider an initial value problem of the form


~ y , ~y ) = 0,
G(~

with ~y (0) = ~y0 .

(1.2.1)

As pointed out in 1.1.2, for this problem to make sense, we need to be able to solve for ~y in terms
of ~y , and write the problem in the form (1.1.1) else there will either be no solution, or too many.
The inverse and implicit function theorems deal with situations under which systems of nonlinear
algebraic equations have solutions that behave properly. These are:7
~ x) be a system of d functions in d variables. Assume
1. Inverse function theorem. Let ~y = G(~
~ is continuously differentiable. Let ~y0 = G(~
~ x0 ). Then a unique inverse function ~x = F~ (~y )
G
with ~x0 = F~ (~y0 ) is defined in a neighborhood of ~y0 , provided that the linearized problem
~y = A ~x is invertible (that is, A = gradient of G at x0 is an invertible matrix).

~ ~ (~y )).
Note 1: Inverse function means that ~y = G(F
Note 2: The linearized problem is obtained by substituting ~y = ~y0 + ~y and ~x = ~x0 + ~x into
~ x), and retaining only the linear terms.
~y = G(~
~ u, ~y ) = 0 in d unknowns ~u,
2. Implicit function theorem. Consider a system of d equations G(~
~ is continuously differentiable. Let G(~
~ u0 , ~y0 ) = 0. Then the
with n parameters ~y where G
system has a unique solution ~u = F~ (~y ) with ~u0 = F~ (~y0 ) defined in a neighborhood of
~y0 , provided that the linearized system A ~u + B ~y = 0 has a unique solution (that is, A =
gradient of G with respect to ~u at (~u0 , ~y0 ) is an invertible matrix).

These theorems are also important for bifurcation theory, which will be an important part of this
course. Another important theorem is the following:
3. Existence and uniqueness for the I.V. problem for ode. Consider the initial value problem:
d~y
= F~ (~y ),
dt

~y (0) = ~y0 ,

where F~ is Lipschitz continuous.

(1.2.2)

This problem has a unique solution, defined on some interval 0 < t < T . The solution depends
continuously on ~y0 , as well as on any parameters on which F~ depends continuously. Furthermore: If F~ is differentiable, the solution has derivatives with respect to ~y0 . If a parameter
dependence of F~ is differentiable, so is the solution, etc ... up to analytic.

No analogous theorem for infinite dimensions: (i) ode theory is fairly complete, but there
is no complete pde theory at all. (ii) Robust black-box ode solvers exist, not so for pde.
7

Proofs can be found in analysis textbooks.

Rosales, MIT (Math.).

Various lecture notes for 18385.

Examples, etc.:
4. Lipschitz continuous means that kF~ (~y1 ) F~ (~y2 )k K k~y1 ~y2 k for some constant K.
If F~ is differentiable, with a bounded derivative, then it is Lipschitz continuous.

Explained in the lectures: Definition of differentiable for vector valued functions.


q

5. Consider the initial value problem in item 8 in 1.1: x = 2 |x|, with x(0) = 0. Clearly
F (x) = 2

|x| is not Lipschitz continuous near zero, and sure enough uniqueness fails.8

6. Example of a differentiable function, with derivative not bounded near a point (hence not
Lipschitz):
f (x) = |x|a sin(1/x), where 1 < a < 2.
(1.2.3)
6 0, and f 0 (0) = 0.
Then f 0 (x) = |x|a2 cos(1/x) + a |x|a1 sin(1/|x|) for x =
Another example is: f (x) = |x|a sin(1/x2 ), where 1 < a < 3.
In models involving an ode where F~ is not Lipschitz continuous, extra care must be taken. In
particular: additional constraints/hypothesis may be needed to have a good model. For example,
consider the ode with a discontinuous right hand side
x = 2 + sign(x).

(1.2.4)

One can think of this as the limit  0 of


x = g(x/),

(1.2.5)

where g = g(z) is some nice smooth function with limits at infinity, where g() = 3 and
g() = 1. For example: g(z) = 2 + tanh(z) + 5 sech(z).
Unfortunately, there are two possible limits! One when g(z) > 0 for all (z), and another when
g(z0 ) = 0 for some z0 . Thus (1.2.4) has more than one possible interpretation! If you just write
(1.2.4), and say nothing else, then you have an equation whose solutions are not properly defined.
1.2.1

The linearization metatheorem.


[vln385:LN12:L02:MTT]

Here we state a rule-of-thumb, which is useful in identifying statements that are probably true. In
particular, when you can trust the results from a linearization, and when you cannot. The rule is:
Imagine some system of equations, for which you have one solution. Imagine also that you can
write a linearized system that solutions infinitesimally close to the one you know must satisfy.9
8
9

It can be shown that, for existence, continuity is enough. It is the uniqueness that requires Lipschitz continuity.
It is not always possible to linearize. Examples where this happens occur, in applications.

Rosales, MIT (Math.).

Various lecture notes for 18385.

Assume now that the linear system has some property, and that this property is structurally stable.10
Then the property is also true for the for the full system.
Examples:
7. Inverse and Implicit function theorems. Note that their formulation, in items 1 and 2, is
exactly in the language of the metatheorem.
8. Consider now an ode problem as in (1.1.1)
d~y
= F~ (~y ),
dt
and some arbitrary solution ~y = ~z(t). Then, in order to be able to linearize the equations near
~z we need F~ to be differentiable. Further, for the resulting linear system to be well behaved,
it seems reasonable to require that the derivative of F~ be (at least) bounded. But then F~ is
Lipschitz continuous and (theorem in item 3) the nonlinear system is also be well behaved.
Notice that the theorem in item 3 is stronger than the metatheorem, in the sense that it
applies even in cases where a linearization is not possible.
On the other hand, note that: properties that are not structurally stable for the linearized system
are prime candidates for failure in the full system. You must always try to investigate the effects
of small nonlinear corrections in such situations. Of course, this is much harder than linearizing,
and the subject of perturbation theory. We will cover a few tools from perturbation theory.

THE END

10

Structurally stable means that the property remains true when the linear system is perturbed slightly.

MIT OpenCourseWare
http://ocw.mit.edu

18.385J / 2.036J Nonlinear Dynamics and Chaos


Fall 2014

For information about citing these materials or our Terms of Use, visit: http://ocw.mit.edu/terms.

Tricky Asymptotics Fixed Point Notes.


18.385, MIT.


Rodolfo R. Rosales.

October 31, 2000.

Contents
1 Introduction.

2 Qualitative analysis.

3 Quantitative analysis, and failure for n = 2.

4 Resolution of the diculty in the case n = 2.

5 Exact solution of the orbit equation.

14

6 Commented Bibliography.

15

List of Figures

1.1 Phase plane portrait for the Dipole Fixed Point system (n = 1.) . . . . . . . . . . .

3.1 Phase plane portrait for the Dipole Fixed Point system (n = 5.) . . . . . . . . . . .

10

Abstract
In this notes we analyze an example of a linearly degenerate critical point, illustrating some
of the standard techniques one must use when dealing with nonlinear systems near a critical
point.

For a particular value of a parameter, these techniques fail and we show how to get

around them.

For ODE's the situations where standard approximations fail are reasonably

well understood, but this is not the case for more general systems. Thus we do the exposition
here trying to emphasize generic ideas and techniques, useful beyond the context of ODE's.

 MIT,

Department of Mathematics,Cambridge, MA 02139.

Tricky asymptotics xed point.

1 Introduction.

Notes: 18.385, MIT.

Rosales, Fall 2000.

Here we consider some subtle issues that arise while analyzing the behavior of the orbits near the
(single, thus isolated) critical point at the origin of the Dipole Fixed Point system (see problem
6.1.9 in Strogatz book)

dx 2
= xy ;
dt n

and

dy
= y2
dt

x2 ;

(1.1)

where 0 < n  2 is a constant. Our objective is to illustrate how one can analyze the behavior
of the orbits near this linearly degenerate critical point and arrive at a qualitatively1 correct
description of the phase portrait. We will use for this \standard" asymptotic analysis techniques.
The case n = 2 is of particular interest, because then the standard techniques fail, and some
extra tricks are needed to make things work.
Just so we know what we are dealing with, a computer made phase portrait for the system2

(case n = 1) is shown in gure 1.1. Other values of 0 < n  2 give qualitatively similar pictures.
However, for n > 2 there is a qualitative change in the picture. We will not deal with the case n > 2
here, but the analysis will show how it is that things change then. The threshold between the two
behaviors is precisely the tricky case where \standard" asymptotic analysis techniques do not work.

2 Qualitative analysis.
We begin by searching for invariant curves, symmetries, nullclines, and general \orbit shape" properties for the system in (1.1).

A.

Symmetries.

The equations in (1.1) are invariant under the transformations:3

! x
A1 and A2 show that we need only study the behav! y and t ! t
ior of the equation in the quadrant x  0, y  0.
A3. x ! ax, y ! at, and t ! t=a, for any constant a > 0.
A1. x
A2. y

1 With quantitative extra information.


2 The

analysis will, however, proceed in a form that is independent of the information shown in this picture.

3 Notice

that these types of invariances occur as a rule when analyzing the \leading order" behavior near degenerate

critical points; because such systems tend to have homogeneous simple structures.

Tricky asymptotics xed point.

Notes: 18.385, MIT.

Rosales, Fall 2000.


2

Dipole Fixed Point: xt = 2xy/n, yt = y - x , n = 1.


2

-1

-2
-2

-1

x
Figure 1.1: Phase plane portrait for the Dipole Fixed Point system (1.1) for n = 1. The qualitative
details of the portrait do not change in the range 0 < n  2. However, for n > 2 di erences arise.

The last set of symmetries (A3) shows that we need only compute a few orbits, since once we

have one orbit, we can get others by expanding/contracting it by arbitrary factors


a > 0. Note that we say \a few" here, not \one"! This is because the expansion/contractions
of a single orbit need not ll up the whole phase space, but just some fraction of it. A
particularly extreme example of this can be seen in gure 1.1, where the orbit given by y > 0
and x  0 simply gives back itself upon expansion. On the other hand, we will show that
any of the orbits on x > 0 (or x < 0) gives all the orbits on x > 0 (respectively, x < 0) upon
expansion/contraction.4 Actually: this is, precisely, the property that is lost for n > 2!
Note: (A2) shows that this system is reversible. On the other hand, because there are open
4 It

even gives the special orbits on the

y-axis by taking a = 1, and the critical point by taking a = 0.

Tricky asymptotics xed point.

Notes: 18.385, MIT.

Rosales, Fall 2000.

sets of orbits that are attracted by the critical point (we will show this later), the system is not
conservative. In fact, this is an example of a reversible, non-conservative system with a minimum
number of critical points.

B.

Simple invariant curves.

The y -axis (x  0) is an invariant line. Along it the ow is

in the direction of increasing y , with vanishing derivative at the origin only. This invariant
line is clearly seen in gure 1.1.
For n > 2, two further (simple) invariant lines are: y =  p

pn

n 2

x.

Whenever a one parameter family of symmetries exist (such as (A3)), you should look for
invariant curves that are invariant under the whole family. In this case, this means looking
for straight lines (which is what we just did.)

C.

D.

Nullclines.

The nullclines are given by

C1. The x-axis (y  0), where x_ = 0 (and, for x 6= 0, y_ < 0.)


C2. The y -axis (x  0), where x_ = 0 (and, for y 6= 0, y_ > 0.)
C3. The lines y = x, where y_ = 0. In the rst quadrant we also have x_ > 0 here.
Orbit shape properties.

In the rst quadrant (from (A) above, it is enough to study

this x > 0 and y > 0 quadrant only), consider the equation for the orbits

dy n(y 2 x2 ) n y
=
=
dx
2xy
2 x
A simple computation then shows that:

x
:
y

(2.1)

d2 y
n 1 x dy n y 1
=
+
+
2
dx
2 x y 2 dx 2 x2 y


n 
2
2
2
2
=
(2
n
)
y
+
nx
y
+
x
< 0:
4x2 y 3

(2.2)

This shows that the orbits are (strictly) concave in this quadrant. Note, however, that
the inequality breaks down for n > 2. Then the orbits are concave for (n
convex for (n

2)y 2 > nx2 .

2)y 2 < nx2 and

Tricky asymptotics xed point.

Notes: 18.385, MIT.

Rosales, Fall 2000.

All this information can now be put together, to obtain a rst approximation to what the

phase portrait must look like, as follows:


I.

Region 0

< y < x (y <


_

x>

and _

0.)

The orbits enter this region (horizontally)

across the nullcline y = x, bend down, and must eventually exit the region (vertically) across
the nullcline y = 0. It should be clear that, once we show that one orbit exhibiting this behavior occurs, then all the others will be expansion/contractions of this one and, in particular, of
each other (see (A3).)
The only point that must be clari ed here is why we say above that the orbit \must eventually
exit the region"? Why are we excluding the possibility that y will decrease, and x will increase,
but in such a fashion that the orbit diverges to in nity, without ever making it to the x-axis?
The answer to this is very simple: this would require the orbit to have an in ection point,
which it cannot have.5

II.

Region 0

< x < y (y >


_

x>

and _

0.)

Considering the ow backwards in time, we

see that all the orbits that exit this region (horizontally, entering region I) across the nullcline

y = x, must originate at the critical point.


However: do all the orbits that originate at the critical point, exit this region across the

nullcline y = x? Or is it possible for such an orbit to reach in nity without ever leaving this
region? | in fact, this is precisely what happens when n > 2, when all the orbits in the region

n 2 y > n x do this. Figure 1.1 seems to indicate that this is not the case, but: how can
be sure that a very thin pencil of orbits hugging the y -axis does not exist?
In section 3 we will show that all the orbits leave the critical point with in nite

slope (i.e.: vertically). Consider now any orbit that exits this region through the nullcline
y = x, and (we know) starts vertically at the critical point. We also know that all the expansions/contractions of this orbit must also be orbits (see (A)), and it should be clear that
these will ll up this region completely (the fact that the orbit starts vertically is crucial for
this.) But then there is no space left for the alternative type of orbits suggested in the prior
paragraph, thus there are none. This clari es the point in the prior paragraph.
5 See (D)

| notice that the orbits are always concave in this region, for

all

values of

n > 0.

Tricky asymptotics xed point.


III.

Conclusion.

Notes: 18.385, MIT.

Rosales, Fall 2000.

With this information, and using the symmetries in (A), we can draw a

qualitatively correct phase plane portrait, which will look as the one shown in gure 1.1. It
should be clear from this gure that:
The index of the critical point is

= 2.

3 Quantitative analysis, and failure for n = 2.


Our aim in this section is to get some quantitative information about the orbits near the critical
point. In particular, exactly how they approach or leave it.
Our approach below is \semi-rigorous", in the sense that we try to justify all the steps as
best as possible, without going to \extremes" (whatever this means). 100% mathematical rigor in
calculations like the ones that follow is possible in simple examples like the one we are doing | and
not even very hard | but quickly becomes prohibitive as the complexity of the problems increases.
But the type of techniques and way of thinking that we follow below remain useful well beyond the
point where full mathematical rigor is currently achievable. Thus, provided one is willing to pay
the price of not having the \absolute" certainty that full mathematical rigor gives, large gains can
be made | while maintaining a \reasonable" level of certainty. This point of view is pretty close
to the one adopted by Strogatz in his book.
We begin by showing the result announced (and used) towards the end of section 2, namely: that

all the orbits leave/approach the critical point vertically. As before,


we restrict out attention to the rst quadrant, and assume x; y > 0.
a. All the orbits must have a tangent limit direction as they approach the origin. This follows
dy
easily from the concavity of the orbits (see (D)): as t ! 1, the slope
increases monodx
tonically. Thus, it must have a well de ned limit (which may be
to show that this limit is 1.)

1; in fact, the aim here is

b. Suppose that there is an orbit that does not approach the critical point vertically. Then, the
result in item (a) shows that we should be able to write
y  x ; for 0 < x  x ;

(3.1)

Tricky asymptotics xed point.

Notes: 18.385, MIT.

then yields (upon taking the limit x ! 0)

n
2

1

() (n

dy
. Substitution of this into equation (2.1)
dx

where 1  < 1 is a constant,6 in fact = xlim


!0

Rosales, Fall 2000.

2) 2 = n ;

(3.2)

which has no solution for 0 < n  2! It follows that an orbit approaching the critical point at
a nite slope cannot occur | which is precisely what we wanted to show.
We now become more ambitious and ask the question: How exactly do the orbits leave the

critical point? | that is to say: What is the leading order behavior in their shape
for 0 < x << 1? As we will show later (see remark 3.2), the answer to this question is useful in
calculating the rate (in time) at which the solutions approach the critical point.
To answer this last question we proceed as follows: We know that the orbits have in nite slope near
the critical point, thus we can write

y  x for 0 < x  1 :

(3.3)

Using this, we should be able to replace equation (2.1) by the approximation

dy
dx
This yields

 2nxyy

ny
:
2x

y  xn=2 ;

(3.4)

(3.5)

where is a constant. This last step is not rigorous, by a long shot, and we must be a bit careful
before accepting it. Equation (3.4) is correct (the neglected terms are smaller than the ones kept),
but it is not clear that (upon integration) the neglected terms will not end up having a signi cant
contribution to the solution of the equation.
Thus before we accept equation (3.5) we must make some basic checks (these sort of
checks are important, you must always try to do as much as it is reasonable and you can do along
these lines), such as:
6 We

know that

 1 because the orbit must leave the critical point staying above the line y = x.

Tricky asymptotics xed point.


c.

Notes: 18.385, MIT.

Consistency with known facts.

Rosales, Fall 2000.

For example:

c1. For 0 < n < 2, (3.5) is consistent with (3.3).


c2. For n > 2, (3.5) is not consistent with (3.3). However, our proof that the orbits approach
the critical point vertically (which is what (3.3) is based on) does not apply for n > 2.
In fact, for n > 2, (3.2) provides a very de nite (neither in nite nor zero) direction of
approach | which happens to agree with the invariant lines mentioned in (B) earlier.
So, there is no contradiction (see remark 3.1 below for a brief description of what the
situation is when n > 2.)
c3. For n = 2, (3.5) is not consistent with (3.3). Since our proof that the orbits approach the
critical point vertically (which implies (3.3)) does apply for n = 2, we have a problem

here, a rather tricky one, which we will address in section 4 below.


d.

Self-consistency

(plug in the proposed approximation into the full equation and check

that the neglected terms are indeed small). In this case the neglected term in the equation is

nx
, which has size (using (3.5))
2y
nx
= O(x(2
2y

n )=2

);

while

dy ny
=
= O ( x (n
dx 2x

2)=2

):

n)=2 > (n 2)=2,


which is true only for n < 2. Thus (3.5) is self-consistent only for n < 2.
For the retained terms to be smaller than the neglected terms, we need (2

e.

Estimate the error.

That is, write the solution as

y = xn=2 + y1 ;
and assume y1  xn=2 . Then use this to get an approximate equation for y1 , solve it, and
check that, indeed: y1  xn=2 .

In the case 0 < n < 2 (the only one worth doing this for, since the other cases have already
failed the two prior tests) one can do not only this, but repeat the process over and over again,
obtaining at each stage higher order asymptotic approximations to the solution. That is, an
asymptotic series of the form

y = xn=2 + y1 + y2 + y3 + : : : ;
where yn+1  yn , can be systematically computed.

(3.6)

Tricky asymptotics xed point.

Notes: 18.385, MIT.

Rosales, Fall 2000.

Remark 3.1 What happens when n > 2.


The same methods that work for 0 < n < 2 can be used to study this case (but a bit more work
is needed). The main di erence in the phase portrait occurs because all the orbits (except for the
p
p
special ones along the y -axis) approach the critical point along the lines n 2 y =  n x.
p
p
For n 2 jy j <  n jxj, the orbits look rather similar to the orbits in the case 0 < n < 2, that
is to say: closed loops starting and ending at the critical point, except that they approach the critical
p
p
point along the lines n 2 y =  n x, not the y -axis.
p
p
For n 2 jy j >  n jxj, the orbits approach the critical point at one end (along the lines
p
p
n 2 y =  n x) and in nity at the other (ending parallel to the y -axis there). In between their
slopes vary steadily (no in ection points) from one limit to the other.
Figure 3.1 shows a typical phase plane portrait for the n > 2 case. From the gure it should be
clear that we still have for the index: I = 2.
Remark 3.2 Rate of approach to the critical point (0 < n < 2.)
Substituting (3.5) into (1.1), we obtain (near the critical point, where both x and y are small)
dx 2 (n+2)=2
dy

x
;
and
 y2 ;
dt
n
dt
where (in the second equation) we simply used the fact that y  x. Thus
1
x=O
( t)2=n

and

y=O

 

as

t!

1:

4 Resolution of the diculty in the case n = 2.


Again we restrict out attention to the rst quadrant, and assume x; y > 0.
The results of section 3 are quite contradictory, when it comes to the case when n = 2. On the
one hand, we showed that (3.3) must apply. But, on the other hand, when we implemented the
consequences of this result (in (3.4)) we arrived at the contradictory result in (3.5). As we pointed
out, the step from (3.4) to (3.5), is not foolproof and need not work. On the other hand, it usually
does, and when it does not, things can get very subtle.7 We will show next a simple approach that
works in xing some problems like the one we have.
7 In

fact, there are some open research problems that have to do with failures of this type, albeit in contexts quite

a bit more complicated than this one.

Tricky asymptotics xed point.

Notes: 18.385, MIT.

Rosales, Fall 2000.


2

10

Dipole Fixed Point: xt = 2xy/n, yt = y - x , n = 5.


2

-1

-2
-2

-1

x
Figure 3.1: Phase plane portrait for the Dipole Fixed Point system (1.1) for n = 5. The qualitative
details of the portrait do not change in the range 2 < n, but di er from those that apply in the range
0 < n < 2 (see gure 1.1.)
What happens for n = 2 must be, in same sense, a limit of the behavior for n < 2, as n ! 2.
Now, look at (3.5) in this limit: it is clear that the behavior must become closer and closer to that
of a straight line (since the exponent approaches 1), at least locally (i.e.: near any xed value of

x). On the other hand, it would be incorrect to assume that this implies that the orbits become
straight in this limit, because this ignores that fact that will depend on n too. In fact, we know
that the limit behavior is not a straight line, but this argument shows that is must be very, very
close to one. Thus we propose to seek solutions of the form

y = x ;

(4.1)

Tricky asymptotics xed point.

Notes: 18.385, MIT.

Rosales, Fall 2000.

11

where = (x) is not a constant, but behaves very much like one as x ! 0. By this we mean that,
when we calculate the derivative

dy
d
= + x;
dx
dx

(4.2)

d
x ; as x ! 0 :
dx

(4.3)

we can neglect the second term. That is

We also expect that ! 1 as x ! 0; since we know that the orbits must approach the critical
point vertically.
Notice that this proposal provides a very clean explanation of how it is that the step

from (3.3) to (3.5), via (3.4), fails (and provides a way out): In writing (3.4) some small
terms are neglected, and what is left is (when writing the solution in the form (4.1)) is . Comparing
this with (4.2), we see that the neglected terms are, precisely, those that make non-constant. Thus,
by neglecting them we end predicting that is a constant,8 which leads to all the contradictions
pointed out in section 3.
What we need to do, therefore, is calculate the leading order correction9 to the right hand side
in (3.4), and equate it to the second term in (4.2). This will then give an equation for

d
, which
dx

we must then solve. If the solution is then consistent with the assumption above in (4.3), we will
have our answer and the mystery will be solved.10
We now implement the process described in the prior paragraph. The leading order correction
to the right hand side in (3.4) is (recall n = 2 now)
correction =

x
1
=
;
y

(4.4)

which is small, since is large for 0 < x  1. Thus the equation for is:

d
1
=
dx

=)

= c 2 ln(x) ;

(4.5)

where c is a constant. It is easy to see that this is consistent with (4.3).


=

8 That

is,

9 That

is to say: plug (4.1) into equation (2.1) and then expand, using the fact that

10 Note

in (3.5).

is large.

that this answer must be sub ject to the same type of basic checks we went through in

section 3, before we accepted (3.5) in the case 0

< n < 2.

(c), (d),

and

(e)

of

Tricky asymptotics xed point.

Notes: 18.385, MIT.

Rosales, Fall 2000.

12

Remark
4.1 It turns out that this problem is so simple that the \leading order" correction | i.e.:
1 
above in (4.4) | is everything! Thus (4.1 { 4.5) in fact provides not just an approximation

near the critical point, but an exact solution! It follows that we do not need to check for any
\consistencies" to make sure that the \approximation" can be trusted (in the manner of (c), (d),
and (e) of section 3.)
Of course, in more complicated problems this will (generally) not happen, and expressions like
the one in (4.1) | with given by (4.5) | will end up being just the rst term in an asymptotic
approximation for the orbit shape.


At this point you may wonder:

what exactly is the \method" proposed here?

Well, as usual with these kind of things, there is no precise recipe that can be given | just as there
is no precise recipe that can be given to explain the \standard" methods. However, just as in the
standard methods one can give a vague | and rather short | list of things to do (e.g.: balance
terms and look for pairs that may dominate, therefore simplifying the problem11 ) we provide below a
list of hints as to what one can do when faced with problems like the one we treat in this section. In
the end, though, each problem is its own thing and (at least with our present level of understanding)
the only way to learn how to do these things \well" is by painfully acquired experience.

When faced with a problem of this type, you may try this:
1. See if you can add a parameter to the equations (say: n), in such a way that the dicult
problem corresponds to some critical value n = nc , and you can do the problem for n 6= nc .
In the example here nc = 2.
2. Look at the behavior of the solution for the \easier" problems as n ! nc . This limit will,
almost certainly, be singular. What you should then do is try to extract a functional form
(by looking at these limits) with appropriate properties.12 The aim is to \guess" what the
\right" form to try for the solution is, by looking at the behavior of the solutions of the nearby
problems on each side of nc (these ought to \sandwich" the right behavior between them.)
11 Books

in asymptotic expansions deal with these and other ideas at length; see (for example) Bender, C. M., and

Orszag, S. A. (1978)

12 Sorry

Advanced Mathematical Methods for Scientists and Engineers

if this sounds very vague; it is very vague, but it is the best I can do!

(McGraw-Hill, New York.)

Tricky asymptotics xed point.

Notes: 18.385, MIT.

Rosales, Fall 2000.

13

In the example studied here we had ( is a constant):

y  x1  ;
y

p  x ;

where  =
where  =

nc

n nc
2

when n < nc = 2
and

when n > nc = 2 :

In the rst case the limit behavior is x, but it is a very non-uniform limit near x = 0 (see
what happens with the derivatives.) In the second case there is not even a limit.
The solutions for both cases, however, have the common form x, where the bad behavior is
restricted to . Thus we picked this common form, and assumed properties for \intermediate" between the behaviors on each side: a constant, but not quite one, and going to in nity

as x ! 0.

3. Alternatively, look at the solution13 that fails for n = nc . This solution will satisfy an approximate form of the equations (where some small terms have been neglected), but will be
inconsistent with the assumptions made in arriving to it | e.g.: the small terms end up not
being as small as assumed. The failure must occur because the neglected small terms have
some important e ect. Therefore, try the following: assume a form of the solution equal to
the one that fails, but allow any free parameters in this solution to be \slow" functions, rather
than constants (this means: when taking derivatives, the terms involving derivatives of the
parameters will be higher order.14 ) Then use this \slow" dependence to eliminate the leading
order terms in the errors to the approximations that lead to the failed solution in the rst
place. If you are lucky, and clever enough, this might x the problem.
In the example studied here, the failure occurs for n = 2, when equation (3.4) becomes

dy y
= ; with solution y = x ( a constant.)
dx x
This solution is inconsistent with the assumption y  x used in deriving (3.4). Thus we took
this form, but made the free constant parameter in the solution ( ) a slow function of x, with
13 Given

by \standard" techniques.

14 These

functions should also have properties (e.g.: large, small, ...

in some limit) that make the assumed form

consistent with the approximations that lead to the equations they solve.

Tricky asymptotics xed point.

Notes: 18.385, MIT.

Rosales, Fall 2000.

14

the additional property  1 as x ! 0 (so that y  x still applies.) This then works, in this
case so well that it gives an exact solution.
The three hints outlined above will work straightforwardly for relatively simple problems, both in
ODE's and PDE's. Beyond that . . .

5 Exact solution of the orbit equation.


Equation (2.1) is simple enough that one can solve it exactly (for all values of n.) We can then
use this exact solution to verify that everything done earlier (using approximate arguments) is
absolutely correct. This is not a luxury one can a ord too often; generally exact solutions are not
available and rigorous arguments are either too expensive or impossible | thus, the only tools one
is left with are numerical computations, approximate analysis, and experimental observations.15
Let us now solve (2.1). Multiply both sides of the equation by 2y and integrate. This yields a linear
equation in y 2 , namely:

dy 2
dx

n 2
y = nx :
x

Now multiply the equation by x n , and integrate again, to obtain (assume x > 0):

dy 2x
dx

= nx1

From this the following solutions follow:

Case 0

<n<

2.

y 2 = 2Rxn

x2 ;

for 0  x 

2R(2

n) 2 n
;

(5.1)

where R > 0 is a constant. For n = 1 these are circles of radius R, centered at (x; y ) = (R; 0).

Case

= 2.

y 2 = (2 ln(x0 ) 2 ln(x)) x2 ;

for 0  x  x0 ;

(5.2)

where x0 > 0 is a constant.


15 For

2-D problems all sorts of theoretician luxuries are available. But real problems are seldom this simple.

Tricky asymptotics xed point.

Case

n>

Notes: 18.385, MIT.

Rosales, Fall 2000.

15

2.

y 2 = Cxn +

n 2

for 0  x 

x2 ;

(n

2)C

n 2;

(5.3)

where C > 0 is a constant (these are the orbits giving closed loops in gure 3.1), or

y 2 = Cxn +
where C

n 2

x2 ;

for 0  x ;

(5.4)

 0 is a constant (these are the orbits that diverge to in nity in the sectors around

the y -axis in gure 3.1.)

6 Commented Bibliography.
Below I list a few books that I think might be of use to you.

1. Cole, J. D. (1968). Perturbation Methods in Applied Mathematics, Blaisdell, Waltham, Mass.


Very nice and concise book (unfortunately, out of print.) It introduces the fundamental
concepts in asymptotic methods, using examples from applications ( uid dynamics, mostly.)
It aims at realistic scienti c problems, so it deals mostly with PDE's (not ODE's).

2. Bender, C. M., and Orszag, S. A. (1978). Advanced Mathematical Methods for Scientists and
Engineers, McGraw-Hill, New York.
This book has an extensive treatment of many of the ideas in asymptotic (and other) methods,
with many comparisons between the asymptotic approximations and numerical solutions. It
introduces the methods using simple examples, so it deals (mostly) with ODE's.

3. Coddington, E. A., and Levinson, N. (1955). Theory of Ordinary Di erential Equations,


McGraw-Hill, New York.
A rigorous treatment of the theory of ODE's, and a classic for this. This book proves everything, but it does so with minimum use of jargon. It has several chapters dedicated to
asymptotic properties of ODE's, a complete treatment of the Poincare Bendixson theorem,
and many other things. If you want hard core proofs, without excuses or unnecessary jargon,
this is the place to go. Of course, it is a bit old, and a lot of the new theory is not here |

Tricky asymptotics xed point.

Notes: 18.385, MIT.

Rosales, Fall 2000.

16

but you cannot really appreciate (or understand) any proof in the newer theory without this
background.

4. Ince, E. L. (1926). Ordinary Di erential Equations, Longmans, Green, London.


There is also a Dover edition!
Old, perhaps, but very good. A hard core exposition of the classical theory of ODE's.

THE END.

MIT OpenCourseWare
http://ocw.mit.edu

18.385J / 2.036J Nonlinear Dynamics and Chaos


Fall 2014

For information about citing these materials or our Terms of Use, visit: http://ocw.mit.edu/terms.

Problem Set Number 1, 18.385j/2.036j


MIT (Fall 2014)
Rodolfo R. Rosales (MIT, Math. Dept., Cambridge, MA 02139)
September 9, 2014
Due Fri., September 19, 2014.

Get equation from phase line portrait problem #07.

Statement: Get equation from phase line portrait problem #07.


Consider the ode on the line

dx
(1.0.1)
= f (x),
dt
where f is some function which is (at least) Lipschitz continuous. Assume that (1.0.1) has at least two
critical points, and let x1 < x2 be two consecutive critical points (there is no other critical point between
them). Assume now that both x1 and x2 are stable. Is this possible? Does a function f = f (x) yielding
this exist?
If the answer is no, prove it.
If the answer is yes, prove it by giving an example.

18.385 MIT, (Rosales)

Get equation from phase line portrait problem #08.

Get equation from phase line portrait problem #08.

Statement: Get equation from phase line portrait problem #08.


Consider the ode on the line

dx
= f (x),
(2.0.1)
dt
where f is some function which is (at least) Lipschitz continuous. Assume that (2.0.1) has infinitely many
critical points < . . . xn < xn+1 < . . . . Assume that all the critical points are semi-stable.1 Is this
possible? Does a function f = f (x) yielding this exist?
If the answer is no, prove it.
If the answer is yes, prove it by giving an example.

Problem 02.02.10 - Strogatz (Fixed points).

Statement for problem 02.02.10.


(Fixed points). For each of (A)(E) below, find an equation x = f (x) with the stated properties, or if there
are no examples, explain why not. In all cases assume that f (x) is a smooth function.
A. Every real number is a fixed point.
B. Every integer is a fixed point, and there are no others.
C. There are precisely three fixed points, and all of them are stable.
D. There are no fixed points.
E. There are precisely 100 fixed points.

Problem 02.03.04 - Strogatz (The Alle effect).

Statement for problem 02.03.04.


(The Allee effect). For certain species of organisms, the effective growth rate N /N is highest at intermediate N . This is called the Allee effect.2 For example, imagine that it is too hard to find mates when N is
very small, and there is too much competition for food and other resources when N is large.
1
2

A critical point is semi-stable if the solutions diverge from the critical point on one side, and converge on the other.
Edelstein-Keshet, L. (1988) Mathematical Models in Biology (Random House, NY).

18.385 MIT, (Rosales)

Get equation from phase line portrait problem #08.

a. Show that N /N = r a(N b)2 provides an example of the Allee effect, if r, a, and b satisfy certain
constraints, to be determined.
b. Find all the fixed points of the system, and classify their stability.
c. Sketch the solution N (t) for different initial conditions.
d. Compare the solutions N (t) with those found for the logistic equation. What are the qualitative and
or quantitative differences, if any?
Hint: for parts c and d write the equation in terms of the a-dimensional variables x = N/b and = (a b2 ) t.
The equation will then have a single parameter the growth rate for x 1. Find and show that it
satisfies a constraint > c . Investigate then the behavior of the solutions of biological interest for various
values of . In particular: what happens for close to c ?

Problem 02.05.05 - Strogatz


(Non-uniqueness example).

Statement for problem 02.05.05.


(A general example of non-uniqueness). Consider the initial value problem x = |x|p/q , x(0) = 0, where
p and q are positive integers with no common factors.
a. Show that there are an infinite number of solutions if p < q.
b. Show that there is a unique solution if p > q.
Hint: Use Lipschitz continuity to show uniqueness.

Problem 02.05.06 - Strogatz (The leaky bucket).

Statement for problem 02.05.06.


(The leaky bucket). The following example3 shows that in some physical situations, non-uniqueness is
natural and obvious, not pathological.
Consider a water bucket with a hole in the bottom. If you see a water bucket with a puddle beneath it, can
you figure out when the bucket was full? No, of course not! It could have finished emptying4 a minute ago,
3

Hubbard, J. H., and West, B. H. (1991) Differential Equations: A Dynamical Systems Approach, Part I (Springer, New
York).
4
Note that, in this problem, evaporation effects are neglected.

18.385 MIT, (Rosales)

Get equation from phase line portrait problem #08.

ten minutes ago, or whatever. The solution to the corresponding differential equation must be non-unique
when integrated backwards in time.
Here is a crude model for the situation. Let h(t) = height of the water remaining in the bucket at time
t; a = area of the hole; A = cross-sectional area of the bucket (assumed constant); v(t) = velocity of the
water passing through the hole.
a. Show that a v(t) = A h . What physical law are you invoking? Warning: since h < 0, this presumes
that we assign a negative value to the velocity v. This is a weird choice, implicit in the problem
statement, but acceptable.
b. To derive an additional equation, use conservation of energy. First, find the change in potential energy
in the system, assuming that the height of the water in the bucket decreases by an amount h, and
that the water has density . Then find the kinetic energy transported out of the bucket by the
escaping water. Finally, assuming all the potential energy is converted into kinetic energy, derive the
equation v 2 = 2 g h g = gravity acceleration.

ap
c. Combining a and b, show that h = C h, where C =
2 g.
A
d. Given h(0) = 0 (bucket empty at t = 0), show that the solution for h(t) is non-unique in backwards
time, i.e., for t < 0.

Problem 03.02.06 - Strogatz (Eliminate the cubic term).

Statement for problem 03.02.06.


(Eliminating the cubic term). Consider the system
dX
= RX X 2 + aX 3 + O(X 4 ) ,
dt

(7.0.1)

where R 6= 0. We want to find a new variable x such that the system transforms into
dx
= Rx x2 + O(x4 ) .
dt

(7.0.2)

This would be a big improvement, since the cubic term has been eliminated and the error term has been
bumped to fourth order. In fact, the procedure to do this (sketched below) can be generalized to higher
orders.5 This generalization is the subject matter of problem 03.02.07.
Let x = X + bX 3 + O(X 4 ), where b is chosen later to eliminate the cubic term in the differential equation
for x. This is called a near-identity transformation, since x and X are practically equal: they differ by
a cubic term.6 Now we need to rewrite the system in terms of x; this calculation requires a few steps.
5
6

That is, one can successively eliminate all the higher order terms: O(x3 ), O(x4 ), . . . , etc.
We have skipped the quadratic term X 2 , because it is not needed you should check this later.

18.385 MIT, (Rosales)

Get equation from phase line portrait problem #08.

1. Show that the near-identity transformation can be inverted to yield X = x + cx3 + O(x4 ),
for c.

and solve

2. Write x = X + 3bX 2 X + O(X 4 ), and substitute for X and X on the right hand side, so that everything depends only on x. Multiply the resulting series expansions and collect terms, to obtain
x = Rx x2 + kx3 + O(x4 ),

where k depends on a, b, and R.

3. Now the moment of triumph: choose b so that k = 0.


4. Is it really necessary to make the assumption that R =
6 0? Explain.

Problem 03.04.05 - Strogatz


(Find and classify bifurcations).

Statement for problem 03.04.05.


For equation (8.0.1) below, find the values of r at which a bifurcation occurs, and classify them as saddlenode, transcritical, supercritical pitchfork, or subcritical pitchfork. Finally, sketch the bifurcation diagram
of fixed points x versus r.
dx
= r 3x2 .
(8.0.1)
dt

Baby normal forms

Do problems #2 and #3 in the Bifurcations: baby normal forms notes in the lecture notes. Note that, in
problem #3, what you are expected to do is to answer the question at the very end in large boldface.
THE END.

MIT OpenCourseWare
http://ocw.mit.edu

18.385J / 2.036J Nonlinear Dynamics and Chaos


Fall 2014

For information about citing these materials or our Terms of Use, visit: http://ocw.mit.edu/terms.

Problem Set Number 2, 18.385j/2.036j


MIT (Fall 2014)
Rodolfo R. Rosales (MIT, Math. Dept.,Cambridge, MA 02139)
September 20, 2014
Due Mon., September 29, 2014.

Inverse function problem #01.

Statement: Inverse function problem #01.


Consider the following equation
y = x + sin(x) = f (x),

(1.0.1)

where, in particular, f (0) = 0 and f 0 (0) = 2 =


6 0. The inverse function theorem guarantees that: there is a
neighborhood of x = 0 where f has a unique inverse, x = X(y), such that X(0) = 0. Furthermore, since f
is an analytic function, X is an analytic function. This means that X has a Taylor series
X=

xn y n ,

n=0

which converges for || small enough. Find x1 , x3 , x5 , and xn for all even n.

(1.0.2)

18.385 MIT, (Rosales)

Find and classify bifurcations problem #01.

Find and classify bifurcations problem #01.

Statement: Find and classify bifurcations problem #01.


For equation (2.0.1) below, find the values of r at which a bifurcation occurs, and classify them as saddlenode, transcritical, supercritical pitchfork, or subcritical pitchfork. Finally, sketch the bifurcation diagram
of fixed points x versus r.
dx
x2
=r
.
(2.0.1)
dt
1 + x2

Irreversible switch using a saddle node and a transcritical bifurcation.

Statement: Irreversible switch using a saddle node and a transcritical bifurcation.


Imagine a system1 with a controlling parameter r, and with (at most) two distinct stable equilibrium states:
x1 = x1 (r) and x2 = x2 (r). In particular, such that infinity is unstable that is: for every solution x = x(t)
there exists a constant M > 0 such that |x| < M for t large enough. Furthermore:
A. There is a value r = rs = switch value such that: for r > rs both states exist and are stable so that
the system can be in either one of them.
B. For r < rs only the state x1 exists and it is stable.
C. Both x1 (r) and x2 (r) are continuous functions of r (though, maybe, not smooth), and |x1 (r) x2 (r)|
is bounded away from zero.
Such a system, if started in the state x2 for r > rs , remains in x2 for as long as r varies (slowly enough)
in the range r > rs . Once r crosses below the threshold rs , the system switches to x1 , and remains there
for all values of r. A switch back to x2 is not produced by slow variations in r. The condition in item C
is important, for otherwise small perturbations could produce an accidental switch if x1 and x2 get very
close.
Remark 3.0.1 A standard (reversible) switch [e.g.: a thermostat], operates using hysteresis. For such
systems there are two switching values r1 < r2 , with only x2 stable for r > r2 , only x1 stable for r < r1 , and
both states stable for r1 r r2 . Then the system jumps from x2 to x1 as r is lowered below r1 , and back
to x2 as r is raised above r2 .

Construct an irreversible switch, using a 1-D system of the form


dx
= f (x, r),
dt
1

A switch.

(3.0.1)

18.385 MIT, (Rosales)

Toy model for shell buckling.

with the behavior caused by two bifurcations: a trans-critical and a saddle node (no other
bifurcations should occur!) Then draw the bifurcation diagram.
Hint: It is very easy to construct an explicit example in which f in equation (3.0.1) is a cubic polynomial
in x, and it is linear in the parameter r.
Remark 3.0.2 (Switch uniqueness). Even for a 1-D system such as the one in (3.0.1), there is an infinite
number of possible bifurcation diagrams that yield a switch, with various types of bifurcations involved.2
However, if the restriction that there should be only two bifurcations (one saddle-node and one transcritical)
is imposed, then there are only two possible topologies for the switch bifurcation diagram. This problem asks
you to produce an example of one such switch.

Toy model for shell buckling.

Statement: Toy model for shell buckling.


Hold a ping-pong ball between your thumb and index fingers and squeeze it. If you do not apply enough
force, the ball will deform slightly with a purely elastic response. But, if you push hard enough, the ball will
buckle and you will make a (permanent) dent on it and the ball will be ruined. This is the phenomena
of (thin) shell buckling.
Shell buckling is a very rich phenomena,3 way beyond the scope of this course. Here we will study an
extremely simplified (1-D) version of this phenomena (the emphasis here being on toy model) where all
the geometrical richness of the original setting is gone, and only the buckling bifurcation remains.

m
k

support

rod
support

Figure 0.1: Toy model for shell buckling. A bead of mass m (black square) can slide along a rigid vertical
rod (in red). The bead is connected by two equal springs (in blue), with spring constant k, to two supports
placed symmetrically on each side of the rod. See the text for further details.
A sketch depicting the model is shown in figure 0.1. Further assumptions and notation are:
2

This is the subject of another problem: Irreversible switches; classification.


Lots of interesting and important questions arise. For example: What is the shape of the dent that forms? The dents edges
have sharp corners: why these corners form, and how do they propagate as further pressure is applied?
3

18.385 MIT, (Rosales)

Bifurcations in the circle problem #04.

1. Idealize the bead as a point mass.


2. Let x be the vertical distance, along the rod, of the bead from the horizontal line joining the spring
supports. Let x > 0 if the bead is above the supports and x < 0 if below.
3. Let h > 0 be the distance of the spring supports from the rod, and let L > 0 be the springs equilibrium
length. Assume L > h, so that the springs are under compression for x = 0.
4. Hooks law applies to the springs. Thus they exert a force of magnitude F = k (` L), where ` is the
spring length, along the spring axis, pushing if ` < L, and pulling if ` > L.
5. When the bead slides along the rod, the motion is opposed by a friction force of magnitude b x , where
b > 0 is a constant.
6. Because the rod is rigid, we need to consider only the vertical components of the various forces that
act on the bead. These forces are: (i) Gravity, of magnitude m g, pointing down. (ii) The forces by
the springs. (iii) Friction along the rod.
PROBLEM TASKS:
A. Derive an ode for the bead position, and write it in appropriate a-dimensional variables.4
B. Assume that friction is large, so that inertia can be neglected. Exactly which a-dimensional number
has to be small for friction to be large?
C. Analyze the bifurcations that occur for the equation resulting from item B, as the bead mass changes
in this toy model, increasing the bead mass plays the role of squeezing harder on the ping-pong ball.
What type of bifurcation(s) occur?
Hint: It is a bad idea to try to do this by attempting to solve for the critical points and bifurcation thresholds
analytically. A qualitative, graphical, analysis is the best way to go.
D. The picture in figure 0.1 corresponds, in this toy model, to the ping-pong ball in a more-or-less spherical
shape. What is the buckled state?
E. What a-dimensional parameter controls when bifurcations happen?
Assume that the ratio = L/h > 1 is kept fixed.

(4.0.1)

Bifurcations in the circle problem #04.

Statement: Bifurcations in the circle problem #04.


For equation (5.0.1) find the values of r at which a bifurcation occurs, and classify them as saddle-node,
transcritical, supercritical pitchfork, or subcritical pitchfork. Finally, sketch the bifurcation diagram for the
4

Suggestion: to a-dimensionalize use h for length and b/(2 k) for time.

18.385 MIT, (Rosales)

Bifurcations in the circle problem #04.

fixed points versus r, including the flow direction and the stability of the various branches of solutions (solid
lines for stable branches and dashed ones for unstable ones).
d
= (r sin()) sin(),
dt

(5.0.1)

where is an angle (in radians). Note that the bifurcation diagram which is periodic in should be
for a 2 range in , and a range of r that includes all the bifurcations.

Problem 03.04.08 - Strogatz


(Find and classify bifurcations).

Statement for problem 03.04.08.


For the following equation, find the values of r at which bifurcations occur, and classify those as saddle
node, transcritical or pitchfork (supercritical or subcritical). Finally, sketch the bifurcation diagram of fixed
points, x versus r.
dx
x
= rx
.
(6.0.1)
dt
1 + x2
Extra question: Notice that something strange happens for r = 0 in the bifurcation diagram. Is this a
bifurcation? If so, which type? Does the principle of conservation of stability apply? Hint: look at the
equation satisfied by y = 1/x.

Problem 03.04.11 - Strogatz


(An interesting bifurcation diagram).

Statement for problem 03.04.11.


(An interesting bifurcation diagram). Consider the system

A.
B.
C.
D.
E.
F.

dx
= rx sin(x) .
(7.0.1)
dt
For the case r = 0, find and classify the fixed points, and sketch the vector field.
Show that, when r > 1, there is only one fixed point. What kind of fixed point is it?
As r decreases from to 0, classify all the bifurcations that occur.
For 0 < r  1, find an approximate formula for the values of r at which bifurcations occur.
Now classify all the bifurcations that occur as r decreases from 0 to .
Plot the bifurcation diagram for < r < , and indicate the stability of the various branches of
fixed points.

18.385 MIT, (Rosales)

Bifurcations in the circle problem #04.

Problem 03.04.14 - Strogatz (Subcritical Pitchfork).

Statement for problem 03.04.14.


(Subcritical Pitchfork). Consider the system
dx
= rx + x3 x5 ,
dt

(8.0.1)

which exhibits a subcritical pitchfork bifurcation.


a) Find algebraic expressions for all the fixed points as r varies.
b) Sketch the vector fields as r varies. Be sure to indicate all the fixed points and their stability.
c) Calculate rc , the parameter value at which the nonzero fixed points are born in a saddle-node bifurcation.

Problem 03.06.06 - Strogatz (Patterns in fluids).

Statement for problem 03.06.06.


(Patterns in fluids). G. Ahlers (1989)5 gives a fascinating review of experiments on one-dimensional
patterns in fluid systems. In many cases, the patterns first emerge via supercritical or subcritical pitchfork
bifurcations from a spatially uniform state. Near the bifurcation, the dynamics of the amplitude of the
patterns are given approximately by

dA
dt

= A gA3

in the supercritical case,

(9.0.1)

dA
dt

= A gA3 kA5

in the subcritical case.

(9.0.2)

or

Here A = A(t) is the amplitude of the pattern, > 0 is a typical time scale, and  is a small dimensionless parameter that measures the distance from the bifurcation. The parameter g is positive in the supercritical
case, whereas g < 0 and k > 0 in the subcritical case. (In this context, the equation A = A gA3
is often called the Landau equation.)
a) Dubois and Berge (1978)6 studied the supercritical bifurcation that arises in RayleighBenard convection, and showed experimentally that the steady state amplitude depends on  according to the power
law A  , where = 0.50 0.01. What does the Landau equation predict?
5

Ahlers, G. (1989) Experiments on bifurcations and one-dimensional patterns in nonlinear systems far from equilibrium. In
D. L. Stein, ed. Lectures in the Science of Complexity (Addison-Wesley, Reading, MA).
6
Dubois, M., and Berge, P. (1978) Experimental study of the velocity field in RayleighBenard convection. J. Fluid Mech.
85, 641.

18.385 MIT, (Rosales)

Bifurcations in the circle problem #04.

b) The equation A = A gA3 kA5 is said to undergo a transcritical7 bifurcation when g = 0; this
case is the borderline between supercritical and subcritical bifurcation. Find the relation between A
and  when g = 0.
c) In experiments on TaylorCouette vortex flow, Aitta et al. (1985)8 were able to change the parameter
g continuously from positive to negative by varying the aspect ratio of their experimental set-up.
Assuming that the equation is modified to
dA
= h + A gA3 kA5 ,
dt

(9.0.3)

where h > 0 is a slight imperfection, sketch the bifurcation diagram of A versus  in the three cases:
g > 0, g = 0, and g < 0. Then look up at the actual data in Aitta et al. (1985, figure 2) or see Ahlers
(1989, figure 15).
d) In the experiments of part (c), the amplitude A(t) was found to evolve toward a steady state in the
manner shown in figure 2 of the book (page 88) redrawn from Ahlers (1989), figure 18. The results
are for the imperfect subcritical case g < 0, h =
6 0. In the experiments, the parameter  was switched
at t = 0 from a negative value to a positive value f (in the figure f increases from the bottom to the
top.)
Explain intuitively why the curves have this strange shape. Why do the curves for large f go almost
straight up to their steady state, whereas the curves for small f rise to a plateau before increasing
sharply to their final level? (Hint: Graph A versus A for different f .)

10

Problem 04.01.01 - Strogatz (Define a flow in circle).

Statement for problem 04.01.01.


For which real values of a does the equation
d
= sin(a)
dt
give a well defined vector field on the circle?

THE END.

WARNING: This is a rather unfortunate choice of name! Do not to confuse this situation with the transcritical
bifurcation introduced in section 3.2 of the book. They are not the same thing!
8
Aitta, A., Ahlers, G., and Cannell, D. S. (1985) Transcritical phenomena in rotating TaylorCouette flow. Phys. Rev. Lett.
54, 673.
7

MIT OpenCourseWare
http://ocw.mit.edu

18.385J / 2.036J Nonlinear Dynamics and Chaos


Fall 2014

For information about citing these materials or our Terms of Use, visit: http://ocw.mit.edu/terms.

Problem Set Number 3, 18.385j/2.036j


MIT (Fall 2014)
Rodolfo R. Rosales (MIT, Math. Dept., Cambridge, MA 02139)
September 30, 2014
Due Fri., October 10, 2014.

Problem 140915 (First order phase transition)

Statement for problem 140915


In equilibrium statistical mechanics, one says that a system undergoes a first order phase transition
when the systems thermodynamic equilibrium undergoes a discontinuity (finite jump) as some parameter
crosses a critical value. The freezing of water into ice is an example of a first-order phase transition.
In mathematical terms: let V is the energy potential associated with the system. Then thermodynamic
equilibrium corresponds to the state at which V is at a minimum (if such a minimum exists).1 If the

Note that local, but not absolute, minimums of V correspond to meta-stable equilibrium states. The system can stay in these
states provided it is not perturbed too much.

2
potential depends on some parameter r, and at some value r = rc there is an exchange of minimums (some
local minimum becomes the new absolute minimum, while the old absolute minimum becomes a local
minimum), then a first order phase transition occurs at rc .
Here we will study a simple, 1-D model, for a first order phase transition. Consider a small particle moving
in a 1-D potential force field (in a heavily dissipative environment), while being kicked one way and the
other by the action of thermal molecular motion. In dimensionless form the equation is
x = F (x) +
where

db
,
dt

(1.1)

3
5
1. x is the dissipation and F (x) = dV
dx = r x + x x is the force produced by the potential.

2. b = b(t) is Brownian motion, with a jump parameter size > 0 (see hints)
that is small.

db
dt

is white noise. Assume

Part a. Consider the potential V for the system (1.1), and calculate rc . Here rc is defined by the condition
that V has three equally deep wells, i.e. the values of V at the three local minima are equal.2 Show that rc
corresponds to a first order phase transition.
Part b. Note that the potential is left-right symmetric. What happens with the system state, at equilibrium,
as r crosses rc ?
Part c/challenge. The pdf (probability distribution function) = (x, t) for the position of the particle,
is defined by


1
Probability |x(t) s| < ds = (s, t) ds.
(1.2)
2
c.1 Derive an equation for the time evolution of .
c.2 Find the time independent, equilibrium pdf, 0 , by solving the equation derived in (c.1).
c.3 It can be shown that 0 as t . What does 0 tell you about the behavior of the system, as
r crosses rc ?
Part d. Does the 1-D, deterministic, dynamical system
x = F (x) = r x + x3 x5

(1.3)

have a bifurcation at r = rc ?
Hint for part (c.1)
The pdf describes the probability of finding the particle somewhere in space, at any given time. A related function
is the probability flux (x, t), defined by
(x, t) dt = Probability that the particle crosses x left to right, during the time interval [t, t + dt].
Unlike , need not be positive < 0 means that the particle is moving right to left. In fact,
2

(1.4)

dx = 0.

For this value of r, there is equal probability of finding the system in the state corresponding to any of the three minima.

3
From the definition of , it should be clear that
Z

t (x, t) dx =
a

d
dt
|

(x, t) dx = (a, t) (b, t) =


a
{z
}
conservation of probability

x (x, t) dx

(1.5)

for any a < b. Assuming that t and x are continuous, this leads to the equation
t + x = 0.

(1.6)

Hence, in order to do part (c.1), you need to write in terms of , and then use (1.6).
The flux is the sum of two parts. The flux produced by the deterministic term F in (1.1), plus the flux produced
by the random Brownian motion fluctuations. The deterministic flux is easy to write, as it is the flux produced by
motion at speed F what is the flux of some density, when the things the density characterizes move at some known
speed?
To write the flux associated with Brownian motion, you need to know what Brownian motion is. There are many ways
to define Brownian motion. A very simple one (albeit mathematically rather sloppy) is this:

In a time interval dt, the particle jumps a distance dt,


(1.7)
to the right or left with probability 12 in each direction.
By considering the probability of the particle being in the intervals [x

dt, x] and [x, x + dt], at some time

t, and the above definition, you can compute the flux across x in a time interval dt. Expanding, and neglecting all
quantities with orders higher than dt, should then give you written in terms of .

Hint for part (c.2)


To find 0 you will have to solve a second order ode. It is easy to integrate this ode once, reducing the problem to that
of solving a first order linear ode. Because the problem is a second order ode, the solutions involve two free constants.
R
These free constants are picked uniquely by that has to be integrable, non-negative, and satisfy 0 (x) dx = 1.

Problem 140916 (Model problem on singular limits)

Statement for problem 140916


Consider the linear differential equation


d2 x dx
+
+ x + sin(t) = 0,
dt2
dt

where

0 <   1,

(2.1)

subject to the initial conditions x(0) = 1 and x (0) = 0.


a. Solve the problem analytically, for any 0 <  < 1/4.
b. Show that, for 0 <   1, there are two widely separated time scales in the solution, and estimate
them in terms of . Hint: expand all the constants in the solution in powers of .

c. What do you conclude about the validity of replacing (2.1) by its singular limit x + x + sin(t) = 0?
Hint: inspect the behavior of the leading order in of the solution that you just obtained, for t  .
d. Give a mechanical example where this equation arises. Then find the dimensionless combination of
parameters corresponding to , and state the physical meaning of the limit 0 <   1.
e. Graph x(t) and x (t) for 0.15. Compare with the solution to the singular limit.

Problem 140924 (Excitable systems)

Statement for problem 140924


(Excitable systems). Consider the situation when a neuron is stimulated. For a small enough stimulus, not
much happens: the neuron increases its membrane potential slightly, and then relaxes back to its rest state.
Beyond a critical threshold, the neuron fires, and produces a large voltage spike before returning to rest.
Interestingly, the spikes size is almost independent of the stimulus size anything above the threshold
produces essentially the same response. Similar phenomena occur for other types of cells and some chemical
reactions.3 These systems are called excitable. An excitable system is characterized by the properties:
1. It has a unique, globally attracting rest state.
2. A stimulus above some threshold sends the system orbit on an O(1) excursion through phase space,
before it returns to the rest state.
A very simple excitable system (one of the simplest possible) is
where is slightly larger than 1 (0 < 1  1).

d
= 1 + sin ,
dt

(3.1)

a. Show that this system satisfies the properties above. Identify the rest state and the threshold.
b. Let V (t) = cos (t). Plot V for various initial conditions V is the neurons membrane potential
analog, and the initial conditions correspond to different perturbations from the rest state.4
c. Is there another range of values for which make (3.1) an excitable system?

Winfree, A., T. (1980) The Geometry of Biological Time (Springer, New York).
Rinzel, J., and Ermentrout, G. B. (1989) Analysis of neural excitability and oscillations. In C. Koch and I. Sergev, eds.
Methods in Neuronal Modeling: From Synapses to Networks. (MIT Press, Cambridge, MA).
Murray, J. (1989) Mathematical Biology (Springer, New York).
4
Of course, equation (3.1) is much too simple to take it seriously as a model for a neuron.

Problem 05.01.02 - Strogatz (Asymptotic behavior as t )

Statement for problem 05.01.02


Consider the system x = ax, y = y, where a < 1. Show that all trajectories become parallel to the ydirection as t , and parallel to the x-direction as t .
dy
Hint: Examine the slope
= y/x .
dx
Strictly speaking, not all trajectories satisfy these two statements. What are the exceptions?

Problem 140922 (Attracting and Liapunov stable)

Statement for problem 140922


Recall the definitions for the various types of stability that concern critical points:
Let x be a fixed point of the system x = f (x). Then:
1. x is attracting if there is a > 0 such that limt = x whenever kx(0) x k < . That is: any
trajectory that starts within of x eventually converges to x . Note that trajectories that start
nearby x need not stay close in the short run, but must approach x in the long run.
2. x is Liapunov stable if for each  > 0, there is a > 0 such that kx(t) x k <  for t > 0, whenever
kx(0) x k < . Thus, trajectories that start within of x stay within  of x for all t > 0.
In contrast with attracting, Liapunov stability requires nearby trajectories to remain close for all t > 0.

3. x is asymptotically stable if it is both attracting and Liapunov stable.


4. x is repeller if there are  > 0 and > 0 such that if 0 < kx(0) x k < , after some critical time
kx(t) x k >  applies (i.e., for t > tc ). Repellers are a special kind of unstable critical points.
For each of the following systems, decide whether the origin is attracting but not Liapunov stable, Liapunov
stable, asymptotically stable, repeller, or unstable but not a repeller.
a) x = 2 y

and

y = 3 x.

b) x = y cos(x2 + y 2 )

and

y = x cos(x2 + y 2 ).

c) x = x

and

y = |y| y.

d) x = 2 x y

and

y = y 2 x2 .

e) x = x 2 y x2 4 y 3

and

y = y + x3 + 2 x y 2 .

f) x = y

and

y = x.

Hint: what happens along x = 0?

g) Finally, consider the critical point (x, y) = (1, 0), for the system
x = (1 r2 ) x (1 xr ) y and y = (1 r2 ) y + (1 xr ) x,
(5.1)
p
defined in the punctured plane r = x2 + y 2 > 0. Hint: write the equations in polar coordinates.

6
Additional hints. In some cases you can get the answer by finding a function J = J (x, y) with a local
minimum at the origin such that ddtJ > 0 along trajectories or maybe one such ddtJ < 0, or maybe one
such ddtJ = 0. In other cases look for special trajectories that either leave, or approach, the origin.

Problem 06.01.07 - Strogatz (Nullclines versus stable manifolds)

Statement for problem 06.01.07


(Nullclines versus stable manifolds). There is a confusing aspect of Example 6.1.1 in the book,5 dealing
with the system
dx
dy
= x + ey ,
and
= y.
(6.1)
dt
dt
The nullcline x = 0 in Figure 6.1.3 has a similar shape and location as the stable manifold of the saddle,
shown in Figure 6.1.4. But they are not the same curve! To clarify the relation between the two curves, plot
both of them on the same phase portrait. You have two options here. Either:
1. Use a computer to do the plot, generating the stable manifold by numerically solving the equation; or
2. Find an explicit formula for the stable manifold, and then do a sketch of the phase portrait.
Hint. Write the equation for

dy
,
dx

and then solve it.

A sketch without an analytical justification is not an acceptable answer!

Problem 06.01.09 - Strogatz (Computer generated phase portrait)

Statement for problem 06.01.09


Plot a computer generated phase plane portrait for the Dipole fixed point system
dx
= 2xy
dt

and

dy
= y 2 x2 .
dt

(7.1)

Problem 06.01.10 - Strogatz (Computer generated phase portrait)

Statement for problem 06.01.10


First, plot a computer generated phase plane portrait for the two-eyed monster
dx
= y + y2
dt
5

and

First edition: pp. 147-148. Second edition: pp. 148-149.

dy
1
1
6
= x + y x y + y2.
dt
2
5
5

(8.1)

18.385 MIT, (Rosales)

Index for a critical point with zero determinant.

In particular, make a plot that covers the region 5 x 3 and 3 y 2.


Next find the critical points and classify them. Does what you observe in the plot match what the theory
predicts? Explain any discrepancies.
Hint: Explore carefully what happens close to the critical points.

Problem 06.01.13 - Strogatz (Draw a phase portrait)

Statement for problem 06.01.13


Draw a phase portrait that has exactly three closed orbits and one fixed point.

10

Index for a critical point with zero determinant.

Statement: Index for a critical point with zero determinant.


Consider a phase plane system
x = f (x, y)

and y = g(x, y),

(10.1)

where f and g are smooth functions of all of its arguments. Assume that:
1. The origin O is an isolated critical point. That is f (0, 0) = g(0, 0) = 0, and there are no solutions to
f (x, y) = g(x, y) = 0 with 0 < x2 + y 2 <  for some .
2. Let A be the 2 2 matrix corresponding to the linearization near O, with = tr(A) and = det(A).
Suppose that = 0 and > 0 so that one eigenvalue of A vanishes, and the other equals .
This is a structurally unstable situation, in particular: the index for O is not determined at all by the
linearized equations. Construct examples of the above situation where:
A. I = index(O) =

1.

B. I = index(O) = 1.
C. I = index(O) =

0.

Sketch the phase plane diagrams for the systems that you construct.
Hints. Consider the linear system Y = A Y , and then add a nonlinear correction which:
For part A. Makes O into a (nonlinear) node.
For part B. Makes O into a (nonlinear) saddle.
For part C. Makes O into a (nonlinear) saddle on one side, and a (nonlinear) node on the other.

THE END.

MIT OpenCourseWare
http://ocw.mit.edu

18.385J / 2.036J Nonlinear Dynamics and Chaos


Fall 2014

For information about citing these materials or our Terms of Use, visit: http://ocw.mit.edu/terms.

Problem Set Number 4, 18.385j/2.036j


MIT (Fall 2014)
Rodolfo R. Rosales (MIT, Math. Dept.,Cambridge, MA 02139)
October 9, 2014
Due Fri., October 17, 2014.

Problem 06.08.09 - Strogatz (Counter-rotating limit cycles)

Statement for problem 06.08.09


A smooth vector field on the phase plane is known to have exactly two closed trajectories, one of which lies
inside the other. The inner circle runs counterclockwise, and the outer runs clockwise.
True or False: there must be at least one fixed point in the region between the cycles.
If true, prove it. If false, provide a simple counterexample.
Hint: Beware of gut feeling instinctive answers. There is a good chance that your intuition is wrong!

Index theory - interpolating from saddles to nodes #2

Statement: Index theory - interpolating from saddles to nodes #2


Consider a one parameter family of phase plane systems
x = f (x, y, r)

and y = g(x, y, r),

(2.1)

where f and g are smooth functions of all of its arguments including the parameter r. Assume that:

2
1. The origin is a critical point for all values of r. That is f (0, 0, r) = g(0, 0, r) = 0.
2. For r = 0 the origin is an isolated critical point. In fact, a saddle.
3. For r = 1 the origin is an isolated critical point. In fact, a node.
Show that there is at least one value 0 < R < 1, such that: for r = R the origin is not isolated critical
point.
Hint. Let I = I(r) be the index of the critical point at the origin for (2.1), for any r for which it is defined.
Also note that I(0) = 1 and I(1) = 1. Use now the properties of the index.

A single critical point between two limit cycles #1

Statement: A single critical point between two limit cycles #1


Consider a phase plane system
x = f (x, y)

and y = g(x, y),

(3.2)

where f and g are smooth functions of all of its arguments. Assume that:
1. The system has isolated critical points only.
2. The system has exactly two limit cycles, 1 and 2 , with 2 enclosing 1 .
3. Between 1 and 2 there is a single critical point C.
Given the above:
A. Calculate the index I for C.
B. If A is the matrix of the linearized system near C, show that = det(A) = 0.
C. Construct an example of this situation, and sketch its phase plane portrait. Hint h2 below leads you
to the construction of a system of this form that will have a cycle graph (produced by an homoclinic
orbit connecting C to itself). Sketch a phase plane portrait of the neighborhood of C. How would
you classify this critical point?
Hints:
h1. The index of a critical point is 1 if > 0. This follows because then the critical point is either a node, a spiral
point, satisfies1 = 2 /4 (linearly degenerate node), or has = 0 (linearized center). In the first two cases the
answer is obvious. For the other two cases an arbitrarily small perturbation can be used to transform the critical
point into a spiral point, and then the continuity of the index yields the desired result.
1

Here is the trace of the linearization matrix at the critical point, while is the determinant.

h2. Consider systems of the form x = a x b y and y = b x + a y, which are equivalent to r = r a and = b, where r
and are the polar coordinates (show this!) To do part C select a = G(x2 + y 2 ) and b = b(x, y) appropriately.
For example: take b such that = b > 0 everywhere, except for a single point C between the circles r = 1 and
r = 3. Then choose a so that limit cycles occur at r = 1 and r = 3, and so that C is a critical point. The
reason for selecting a = G(r2 ) as a function of r2 , and not r, is to avoid possible singular behavior at r = 0, where
p
r = r(x, y) = x2 + y 2 is not differentiable.

Problem 06.01.09 - Strogatz (Computer generated phase portrait)

Statement for problem 06.01.09


Plot a computer generated phase plane portrait for the Dipole fixed point system
dx
= 2xy
dt

and

dy
= y 2 x2 .
dt

THE END.

(4.1)

MIT OpenCourseWare
http://ocw.mit.edu

18.385J / 2.036J Nonlinear Dynamics and Chaos


Fall 2014

For information about citing these materials or our Terms of Use, visit: http://ocw.mit.edu/terms.

Problem Set Number 5, 18.385j/2.036j


MIT (Fall 2014)
Rodolfo R. Rosales (MIT, Math. Dept.,Cambridge, MA 02139)
October 17, 2014
Due Fri., October 24, 2014.

Large limit for Li


enard system #03

Statement: Large limit for Li


enard system #03
x
+ f 0 (x) x + g(x) = 0,

(1.1)

d
(x + f (x)) + g(x) = 0.
dt

(1.2)

A Lienard equation has the form


for some functions f and g. Here > 0 is a parameter.
This can be re-written in the form
Introduce y =

x + f (x), to get the system


x = (y f (x))

In this problem we will consider the case


with  1.

1
and y = g(x).

f (x) = x +

1 3
1 5
x
x
3
60

Analyze the large limit for this system. In particular:


1. Are there any limit cycles? Are they stable, unstable, semi-stable?
2. Are there any critical points? Are they attractors, repellers?
3. Does the system have any global attractor?
4. Sketch the phase plane portrait.

(1.3)
and

g(x) = x,

(1.4)

Saddles and Conservative Systems

Statement: Saddles and Conservative Systems


Saddles for conservative systems are special, with the rate of expansion and contraction along the two
principal directions equal. The purpose of this problem is to show this.
Consider a phase plane autonomous system
dx
= f (x, y)
dt

and

dy
= g(x, y),
dt

(2.1)

where both f and g have continuous partial derivatives. We will now assume:
A. The origin x = y = 0 is an isolated critical point; in fact: a saddle, with eigenvalues for the linearized
problem 1 > 0 > 2 .
B. The system is conservative near the origin. That is: there is a function E = E(x, y) (defined in a
neighborhood of the origin) which is a constant on the orbits. Furthermore, assume that E is twice
continuously differentiable, and that E 6= 0 (except at the origin).
C. The origin is a true saddle for E that is det(D 2 E) < 0 at the origin.
Show that: 1 = 2 which is equivalent to fx (0, 0) + gy (0, 0) = 0 (why?).
Remark 2.0.1 What does C mean? Since E is twice differentiable we can write
E = E0 +


1
1
a x2 + b x y + c y 2 + o (x2 + y 2 )1.5 ,
2
2

(2.2)

where E0 , a, b, and c are constants. Then det(D2 E) < 0 a c b2 < 0 which means that the quadratic form
Q = Q(x, y) = 12 a x2 + b x y + 12 c y 2 above in (2.2) is non-degenerate and non-definite. Thus:
1. The level lines for Q govern the local behavior of the level lines for E, and
2. Q has a saddle at the origin, so its level lines yield a saddle.
Hence the surface z = E(x, y) has a saddle at the origin, dominated by the quadratic terms in its Taylor expansion.
Example where this is not true: Consider the system x = x and y = 2 y, which has a saddle at the origin, and the
conserved quantity E = y x2 . But the surface z = E(x, y) does not have a saddle at the origin.1 Another example
is provided by the system x = x and y = 3 y, also with a saddle at the origin and a conserved quantity: E = y x3 . In
this case the surface z = E(x, y) has a saddle at the origin, but it is a very degenerate saddle.2
Remark 2.0.2 Note concerning E. From equation (2.2) is follows that
!
!
!
ax + by
a
b
x
E =
+ o (x2 + y 2 ) =
+ o (x2 + y 2 ).
bx + cy
b c
y

(2.3)

Thus, because the matrix of coefficients is non-degenerate, E =


6 0 near (but not at) the origin.
1

By saddle we mean that the level lines E = 0 split the plane into four regions, and in each region E grows (decays) as we move
away from the origin, with decay and growth alternating.
2
You would not like sitting on a horse saddle made following this design!

3
Hint 2.0.1 Because E is conserved:
Z = Z(x, y) = f Ex + g Ey 0.

Explain why this.

(2.4)

On the other hand, because f and g are differentiable (and the origin is a critical point), we can write (for
some constants , , , and ):
f = x + y + o (x2 + y 2 )

and

g = x + y + o (x2 + y 2 ).

(2.5)

Substitute these expansions and (2.3), into (2.4) and conclude that 0 = + = fx (0, 0) + gy (0, 0). This
then implies 1 = 2 (why?), which is precisely what you are being asked to show.
Remark 2.0.3 General saddles. Modulo a nonsingular linear transformation of the dependent variables x and y, and
a change in the time scale (possibly including a time reversal), the equations for a linear system with a saddle can
x = x and y = y, where 1.
(2.6)
always be written in the form
Conserved quantities for this last system must have the form E = f (|x| y), where f = f () is some arbitrary
function. But then
Ex = sign(x) x(1) y f 0 (|x| y) and Ey = x f 0 (|x| y),
(2.7)
so that E vanishes identically for x = 0, unless = 1, which corresponds to 1 = 2 .

Uniqueness of the van der Pol oscillator limit cycle

Statement: Uniqueness of the van der Pol oscillator limit cycle


Consider the van der Pol equation
d2 x
dx
+ (x2 1)
+ x = 0,
2
dt
dt
where 6= 0 is a constant. Show that (3.1) has a unique limit cycle.

(3.1)

The existence of, at least one, non-trivial 3 periodic orbit was shown in class. Hence, you need only show
that there cannot be more than one such orbit.
It follows (Poincare-Bendixon theorem) that the limit cycle is a global attractor (repeller) for > 0 ( < 0,
resp.). Only one orbit (the critical point) does not approach the limit cycle as either t .
Hints.
h1. Index theory tells us that periodic orbits must enclose the (single) critical point x = x = 0 in the phase plane
(x, y) where y = x . Hence, if the number of periodic orbits exceeds one, they will be nested. Any two
successive ones will then delimit an annular region , with the origin in the center hole.
h2. The change t t reverses the sign of in (3.1). Hence, without loss of generality, assume < 0, and write
dx
dy
the equation as
= y and
= x  (1 x2 ) y, where  = > 0.
(3.2)
dt
dt
Then the origin is an attractor (node or spiral, depending on the size of ), and the limit cycle is unstable.
3

Note that critical points are (trivial) periodic orbits.

h3. Study the behavior of the function E =

1
2


x2 + y 2 along the solutions to (3.2), and show that:

If (x, y) = (X(t), Y (t)) is a non-trivial periodic solution, then X 2 + Y 2 > 1 for all t.
Thus the center hole for the item h.1 annular region includes the unit disk x2 + y 2 1.


(3.3)

To prove (3.3) you need to examine the sign of E in the unit disk. Be careful to provide a complete proof for
(3.3), since along the segment y = 0 and 1 x 1 (where E = 0) a special argument is needed.
p
h3. Find a function g = g(r), defined and smooth for r = x2 + y 2 > 1, such that
h
i
I = div (x g, y g)
x and y as given by (3.2)
(3.4)
is non-negative for r > 1, and vanishes for x = 1 only. Note that g could have singularities when r 1 we
do not care about what happens there.
Sub-hint. The expression for I in (3.4) involves g and its derivative g 0 . The trick is to pick g 0 in terms of g and
r so that the desired property applies. This will give you an ode for g, which you can solve.
h4. Generalize Dulacs criterion to prove that there can be only one periodic orbit:
(1) Assume that there is more than one periodic orbit, and consider the region between any two
consecutive ones, as defined in item h.1.
(2) Integrate I, defined in (3.4), over . Use Gauss theorem to reduce the integral to an integral
over the boundary of , which is made up by the two periodic orbits.
This should yield a contradiction.
It follows that the assumption in (1) must be false: there cannot be more than one periodic orbit.

THE END.

MIT OpenCourseWare
http://ocw.mit.edu

18.385J / 2.036J Nonlinear Dynamics and Chaos


Fall 2014

For information about citing these materials or our Terms of Use, visit: http://ocw.mit.edu/terms.

Problem Set Number 6, 18.385j/2.036j


MIT (Fall 2014)
Rodolfo R. Rosales (MIT, Math. Dept., Cambridge, MA 02139)
October 22, 2014
Due Fri., October 31, 2014.

Problem 07.05.07 - Strogatz (Cell cycle)

Statement for problem 07.05.07


Tyson 1 proposed an elegant model of the cell division cycle, based on interactions between the proteins
cdc2 and cyclin. He showed that the models mathematical essence is contained in the following set of
dimensionless equations
du
dv
= b (v u) ( + u2 ) u and
= c u,
(1.1)
dt
dt
where u is proportional to the concentration of the active form of a cdc2-cyclin complex, and v is proportional
to the total cyclin concentration (monomers and dimers). The parameters b  1 and 0 <  1 are fixed
and satisfy 8 b < 1, while c is adjustable.
A. Sketch the nullclines.
B. Assume that 0 <  = b  1. Then show that the system exhibits relaxation oscillations when
c1 < c < c2 , for some constants c1 and c2 that you should determine approximately, using the fact
that  is small exact expressions are also possible, and not too hard to get.
C. Show that the system is excitable if c is slightly less than c1 . Namely, the system has a globally
attracting fixed point, but some (small, but not infinitesimal) disturbances can send the system on a
long excursion through phase space before returning to the fixed point.
1

J. J. Tyson, 1991, Modeling the cell division cycle: cdc2 and cyclin interactions, Proc. Nat. Ac. Sc. USA, 88, 7328.

2
Remark 1.0.1 The first quadrant is not invariant for this system, since v < 0 for u > c. Thus solutions with u > c,
and v too small, escape the first quadrant. Hence (1.1) is un-biological for u > c, and v too small.

In your answer, concentrate only on the solutions for which u 0 and v 0 applies.

Justify the bead on a wire reduction

Statement: Justify the bead on a wire reduction


In Strogatz book, bead on a wire problems are often modeled by equations of the form
x = f (x).

(2.1)

A more complete description of these systems typically has the form


m

d2 X
dX
+b
=Ff
2
dT
dT

1
L


X ,

(2.2)

where (m, b, F , L) are physical constants,2 and f has no dimensions (T is time, with dimensions).
1. Introduce appropriate variables with no dimensions, and re-write (2.2) in terms of them.
Then use phase plane analysis to fully justify a condition under which (2.1) applies.
2. Estimate the size of the error in (2.1). That is, we can write x = f (x) + E, where E is the
error. Find a leading order estimation for E.

Multiple scales and limit cycles #01

Statement: Multiple scales and limit cycles #01


Consider the equation

d2 x
dx
1
 cos x
+ sin(  x) = 0,
2
dt
dt


where

0 <   1.

(3.1)

Use a multiple scales analysis to calculate the frequency, stability and amplitude of any limit cycle (the
frequency up to the first correction beyond linear and the amplitude up to leading order).

THE END.
2

For example: mass, damping coefficient, typical applied force, and typical length scale. But they can also be related to the
capacitance, resistance, inductance and applied potential in a circuit. Or some other physics.

MIT OpenCourseWare
http://ocw.mit.edu

18.385J / 2.036J Nonlinear Dynamics and Chaos


Fall 2014

For information about citing these materials or our Terms of Use, visit: http://ocw.mit.edu/terms.

Problem Set Number 7, 18.385j/2.036j


MIT (Fall 2014)
Rodolfo R. Rosales (MIT, Math. Dept., Cambridge, MA 02139)
October 27, 2014
Due Fri., November 7, 2014.

Problem 141002 - Computer test of two timing

Statement for problem 141002


Consider the equation
d2 x
dx
+  x2
+ x = 0,
2
dt
dt

(1.1)

where 0 <   1.
a. Derive the averaged equations.
b. Given the initial conditions x(0) = a and x (0) = 0, solve the averaged equations and thereby find
an approximate formula for x = x(t, ).
c. Solve equation (1.1 numerically for a = 1,  = 2, and 0 t 50. Plot the results in the same graph
as your part b answer. Note how good the agreement is, even though  is not small. In fact, most of
the error is due to a (constant) phase shift. After correcting for it, the agreement is impressive (show
this).
Challenge: can you explain why the agreement is so good?

Problem 08.02.06 - Strogatz (Hopf bifurcation using a computer)

Statement for problem 08.02.06


For the following system
dx
= x + y x3
dt

and

dy
= x + y + 2y 3 ,
dt

(2.1)

2
a Hopf bifurcation occurs at the origin when = 0. Using a computer, plot the phase portrait and determine
whether the bifurcation is subcritical or supercritical. For small values of , verify that the limit cycle is
nearly circular. Then measure the period and radius of the limit cycle, and show that the radius R scales
with as predicted by theory.

Problem 08.07.x1 - Forced overdamped linear system

Statement for problem 08.07.x1


Strogatz problem 8.7.3 mentions an overdamped linear oscillator (or an RC-circuit) forced by a square
wave, yet the equation considered is
x + x = F (t),
(3.1)
while in fact it should be
x
+ x + x = F (t),

(3.2)

where 0 <   1. One could argue that, because  is small, the term  x
can be neglected. Yet the problem
is concerned with the long term behavior of the system, where even small effects can become important (if
they accumulate).
The purpose of this problem is to show that, while neglecting the term  x
can cause small changes, it does
not alter the qualitative conclusions of the problem. Show that (3.2) is equivalent to TWO systems of
the form (3.1)
x1 + r2 x1 = F (t) and x2 + r1 x2 = F (t),
(3.3)
for appropriate choices of the constants rj > 0 and functions xj .
Hint. Write the equation in 2-vector form and project the solution along the eigendirections.

A linear center that is a nonlinear spiral.

Statement: A linear center that is a nonlinear spiral


Consider the system

x = y x3 and y = x.

(4.1)

For this system linearization predicts a center at the origin. However, by transforming the system into polar
coordinates, it is easy to see that the origin is actually a nonlinear spiral. Task #1: do this.
Task #2. Implement a two-time expansion for solutions near the critical point,1 and calculate the rate of
approach of the solutions towards the critical point as t . Is it exponential or algebraic? What is the
rate (if exponential) exponent (if algebraic)?

THE END.
1

The small parameter here is the radius  of the neighborhood considered.

MIT OpenCourseWare
http://ocw.mit.edu

18.385J / 2.036J Nonlinear Dynamics and Chaos


Fall 2014

For information about citing these materials or our Terms of Use, visit: http://ocw.mit.edu/terms.

Problem Set Number 8, 18.385j/2.036j


MIT (Fall 2014)
Rodolfo R. Rosales (MIT, Math. Dept., Cambridge, MA 02139)
November 5, 2014
Due Fri., November 14, 2014.

Problems 08.07.05/06/07 - Strogatz


(Another driven overdamped system)

Statement for problems 08.07.05/06/07


A. By considering an appropriate Poincare map, prove that the system
d
+ sin = sin t
dt

(1.1)

has at least two periodic solutions. Can you say anything about their stability?
B. Give a mechanical interpretation for equation (1.1).
C. Plot a computer generated phase portrait for the system in (1.2). Check that the answer agrees with
the results in part A.
Hint. Regard the system as a vector field on a cylinder:
dt
=1
dt

d
= sin t sin .
dt

and

(1.2)

Sketch the nullclines and thereby infer the shape of certain key trajectories that can be used to bound the
periodic solutions. For instance, sketch the trajectory through P = (t, ) = 12 (1, 1).

Problem 141003 - Newtons Method in the complex plane

Statement for problem 141003


Suppose that you want to solve an equation, g(x) = 0. Then you can use Newtons method, which is as follows:
Assume that you have a reasonable guess, x0 , for the value of a root. Then the sequence xn+1 = f (xn ),
n 0, where
f (x) = x gg(x)
(2.1)
0 (x) ,
converges (very fast) to the root.
Remark 2.0.1 (The idea). Assume an approximate solution g(xa ) 0. Write xb = xa + x to improve it, where x
a)
is small. Then 0 = g(xa + x) g(xa ) + g 0 (xa ) x x gg(x
0 (x ) , and (2.1) follows.
a
Of course, if x0 is not close to a root, the method may not converge. Even if it converges, it may converge to a root that
is far away from x0 , not necessarily the closest root. In this problem we investigate the behavior of Newtons method in
the complex plane, for arbitrary starting points.

Consider iterations of the map in the complex plane generated by Newtons method for the roots of z 3 1 = 0.


That is
2
1
zn+1 = f (zn ) =
+
zn , n 0,
(2.2)
3 3 zn3
where 0 < |z0 | < is arbitrary. Note that

1
1
1 = 1, 2 = ei 2 /3 = (1 + i 3), and 3 = ei 4 /3 = (1 i 3),
(2.3)
2
2
are the roots of z 3 = 1.
1
Your tasks: Write a computer program to calculate the orbits {zn }
n=0 . Then, for every initial point z0 ,
draw a colored dot at the position of z0 , where the colors are picked as follows:
zn 1 , cyan.
zn 2 , magenta.
zn 3 , yellow.
No convergence, black.
(2.4)
What do you see? Do blow ups of the limit regions between zones.

Hint. Deciding that the sequence converges is easy: once zn gets close enough to one of the roots, then the very
design of Newtons method guarantees convergence. Thus, given a z0 , compute zN for some large N , and check if
|zN j | < for one of the roots and some small tolerance which does not have to be very small, in fact
= 0.25 is good enough. You can get pretty good pictures with N = 50 iterations on a 150 150 grid. A larger N is
needed when refining near the boundary between zones.
Hint. If you use MatLab, do not plot points. Instead, plot regions, where the color of each pixel is decided by z0
use the command image(x, y, C) to plot. Why? Because using points leaves a lot of unpainted space in the figure,
and gives much larger file sizes.

Coupled oscillators #01

Statement: Coupled oscillators #01


In this problem we present an example of the process described in 4.1, and consider the coupling of two
oscillators with a stable, and strongly attracting, limit cycle each. The oscillators are very simple, with trivial
1

Numerically this means: choose a sufficiently fine grid in a rectangle, and pick every point in the grid. For example, select the
square 2 < x < 2 and 2 < y < 2, where z0 = x + i y.

18.385 MIT, (Rosales)

Coupled oscillators, phase locking, oscillator death, etc.

equations in polar coordinates. This simplifies the analysis enormously, but the principles illustrated here
are valid for the coupling of more generic oscillators.
Consider the following equations for two coupled oscillators
xj
yj

= j yj + j (Rj2 x2j yj2 ) xj + Fj (x1 , y1 , x2 , y2 ),


=

j xj +

j (Rj2

x2j

yj2 ) yj

(3.1)

+ Gj (x1 , y1 , x2 , y2 ),

(3.2)

where j = 1 or j = 2, and
(a) j > 0, Rj > 0 and j > 0, are constants, with j  1,
(b) Fj and Gj are some functions these are the coupling terms.
Using the fact that j  1, write
q reduced equations for the two phases 1 and 2 , defined by xj = rj cos j

and yj = rj sin j , where rj =

x2j + yj2 . In particular, consider the following cases

1. What form do the reduced equations take when the Fj and Gj are only functions of the variables
= x1 x2 + y1 y2 and = y1 x2 x1 y2 .
2. What form do the reduced equations take when G1 = G2 = 0, F1 =
2
R
x where and are constants.
R1 1

R1
R2

x2 , and F2 =

Hint. Write the equations in polar coordinates.2 Then consider what happens in a neighborhood of the limit
cycles for the two oscillators when de-coupled i.e.: rj not too far from Rj . In this context, argue 3 that
the dependence on the radial variables can be made trivial.

Notes: coupled oscillators, phase locking, etc.

These are notes with facts useful for the problems. They are not a problem.

4.1

On phases and frequencies

Consider a system made by two coupled oscillators, where each of the oscillators (when not coupled) has a
stable attracting limit cycle. Let the limit cycle solutions for the two oscillators be given by ~
x1 = F~1 (1 t)
~j are
and ~
x2 = F~2 (2 t), where ~
x1 and ~
x2 are the vectors of variables for each of the two systems, the F
periodic functions of period 2 , and the j are constants (related to the limit cycle periods by j = 2 /Tj ).
In the un-coupled system, the two limit cycle orbits make up a stable attracting invariant torus for the
evolution. Assume now that either the coupling is weak, or that the two limit cycles are strongly stable.
Then the stable attracting invariant torus survives for the coupled system.4 The solutions (on this torus)
can be (approximately) represented by
~x1 F~1 (1 )

and ~x2 F~2 (2 ),

(4.1)

Recall that rj rj = xj x j + yj y j and rj2 j = xj y j yj x j .


Use arguments similar to the one introduced to describe relaxation oscillations, e.g.: for the van der Pol equation. Another
example occurs when justifying that inertial terms can be neglected in the limit of a large viscosity.
4
With a (slightly) changed shape and position.
2

18.385 MIT, (Rosales)

Coupled oscillators, phase locking, oscillator death, etc.

where 1 = 1 (t) and 2 = 2 (t) satisfy some equations, of the general form
1 = 1 + K1 (1 , 2 )

and 2 = 2 + K2 (1 , 2 ).

(4.2)

Here K1 and K2 are the projections of the coupling terms along the oscillator limit cycles. For example,
take K1 (1 , 2 ) = sin 1 cos 2 and K2 (1 , 2 ) = sin 2 cos 1 . Another example is the one in 8.6 of Strogatz
book (Nonlinear Dynamics and Chaos), where a model system with
K1 (1 , 2 ) = 1 sin(1 2 ) and K2 (1 , 2 ) = 2 sin(1 2 )
is introduced, with constants 1 , 2 > 0. Note that:
1. In (4.2), K1 and K2 must be 2 -periodic functions of 1 and 2 .
2. The phase space for (4.2) is the invariant torus T , on which 1 and 2 are the angles. We can also
think of T as a 2 2 square with its opposite sides identified. On T a solution is periodic if and
only if 1 (t + T ) = 1 (t) + 2 n and 2 (t + T ) = 2 (t) + 2 m , where T > 0 is the period, and both n
and m are integers.
3. In the Coupled oscillators # 01 problem an example of the process leading to (4.2) is presented.
4. The j s are the oscillator phases. One can also define oscillator frequencies, even when the j s
do not have the form j = j t, with j constant. The idea is that, near any time t0 we can write
j = j (t0 ) + j (t0 ) (t t0 ) + . . ., identifying j (t0 ) as the local frequency. Hence, we define the oscillator frequencies by
j = j . These frequencies are, of course, generally not constants.
5. The notion of phases can survive even if the limit cycles cease to exist (i.e.: oscillator death). For
example: if the equations for 1 and 2 have an attracting critical point. We will see examples where
this happens in the problems, e.g.: Bifurcations in the torus # 01.

4.2

Phase locking and oscillator death

The coupling of two oscillators, each with a stable attracting limit cycle, can produce many behaviors. Two
of particular interest are
1. Often, if the frequencies are close enough, the system phase locks. This means that a stable periodic
solution arises, in which both oscillators run at some composite frequency, with their phase difference
kept constant. The composite frequency need not be constant. In fact, it may periodically oscillate
about a constant average value.
2. However, the coupling may also suppress the oscillations, with the resulting system having a stable
steady state. This even if none of the component oscillators has a stable steady state. This is oscillator
death. It can happen not only for coupled pairs of oscillators, but also for chains of oscillators with
coupling to the nearest neighbors.
On the other hand, we note that it is also possible to produce an oscillating system, with a stable oscillation,
by coupling non-oscillating systems (e.g., the coupling of excitable systems can do this).

THE END.

MIT OpenCourseWare
http://ocw.mit.edu

18.385J / 2.036J Nonlinear Dynamics and Chaos


Fall 2014

For information about citing these materials or our Terms of Use, visit: http://ocw.mit.edu/terms.

Problem Set Number 9, 18.385j/2.036j


MIT (Fall 2014)
Rodolfo R. Rosales (MIT, Math. Dept.,Cambridge, MA 02139)
November 11, 2014
Due Fri., November 21, 2014.

Simple Poincar
e Map for a limit cycle #02

Statement: Simple Poincar


e Map for a limit cycle #02
Consider the following autonomous phase plane system
dx
dt
dy
dt

(x2

(x2

y4)

y4)

x3

y+

1
4

x3

x2 y

x2 y

x y2
x y2

4 y3

y3

where > 0.

(1.1)

This system has a periodic solution (show this), which can be written in the form
x = 2 cos , y = sin ,

where

d
= 2 (x2 + y 4 ) = 2 (1 + cos2 )2 .
dt

(1.2)

This solution produces an orbit going through the point x = 0, y = 1 in the phase plane. The orbit is an
ellipse, as (1.2) shows.1
Construct (either numerically 2 or analytically) a Poincar
e map near this orbit, and use it to show that
the orbit is a stable limit cycle. Define the Poincare map z u = P (z) as follows:

1
2

Note that is a strictly increasing function of time.


If you do it numerically, keep as a variable and check your answers for several values say: = 0.1, 0.5, 1, 2, 5.

2
For every sufficiently small z, let x = X(t, z) and y = Y (t, z) be the solution of (1.1) defined by
X(0, z) = 0 and Y (0, z) = 1 + z.
For this solution the polar angle in the phase plane is an increasing function of time, starting at
= 12 for t = 0. Thus, there is a time t = tz at which the
solution reaches = 52 (note that tz is a function of z).
Then take u = Y (tz , z) 1.
Hint. Because tz is a function of z, unknown a priori, the definition of the Poincare map above is a bit
awkward to implement. To avoid having to calculate tz for each solution, it is a good idea to use a parameter
other than time to describe the orbits. For example, if the equations are written in terms of a parameter
such as the polar angle namely dx
= F (x, y) and dy
= G(x, y), then the Poincare map is easier
d
d
1
5
to describe, as varies from = 2 to = 2 in every one of the orbits needed to compute u = P (z ).
Note that this is just a for example, using the polar angle is not the best choice. You may want to scale
the variables first, so that the limit circle is a circle, not an ellipse.

Bifurcations in the torus #01

Statement: Bifurcations in the torus #01


Bifurcations in the torus, phase-locking, and oscillator death. This problem is based on a paper on systems
of neural oscillators by G. B. Ermentrout and N. Kopell,3 where they illustrate the notion of oscillator death
(see 3) with the following model
1 = 1 + sin 1 cos 2

and 2 = 2 + sin 2 cos 1 ,

(2.1)

where 1 , 2 > 0. Here 1 and 2 are to be interpreted as the phases of two coupled stable and attracting
limit cycle oscillators, which are assumed to survive the coupling, so that the notion of their individual
phases remains see 3.
a. Classify all the different behaviors that the solutions to (2.1) have, as the parameters vary in the
positive quadrant of the [1 , 2 ]plane. Do a diagram in this quadrant, indicating the regions that
correspond to each behavior.
The final answer should look something like this: (i) In such and such region the solutions are attracted to
a limit cycle [Note that this is phase locking]. (ii) In such and such region the solutions are attracted
to a stable node [Note that this is oscillator death]. (iii) In such and such region the solutions are
quasi-periodic with two periods [Phase locking fails]. (iv) . . .
Plus a drawing of the regions . . . will all the statements properly justified.
b. Draw the bifurcation curves in the [1 , 2 ]plane. Describe each bifurcation.
3

Oscillator death in systems of coupled neural oscillators. SIAM J. Appl. Math. 50:125 (1990).

3
Hints. I did not find an elegant way to analyze the system geometrically. The hints below lead you to an
approach that is (mostly) analytical, but allows a systematic and thorough investigation.
h1. Consider the equations satisfied by = 1 + 2 and = 1 2 .
h2. You may find the following result useful
Let > 1. Then the solutions to the equation
can be written in the form
where > 0 is a constant, X is 2 -periodic,
X(0) = 0, and t0 is an arbitrary constant.

= + sin
= (t t0 ) + X( (t t0 )),

Furthermore: is an increasing function of , with lim = 0 and lim = .


1

All this follows from the results in 2.1, upon using a change of variables that transforms = + sin
into (2.2). In particular, note that the in 2.1 (call it
) is related to the one here by =
,
with = 1/.

2.1

Notes on first order equation with a periodic right hand side

These are notes with facts useful for this problem. They are not a problem.
Consider the equation
= 1 sin ,

where 0 < < 1.

(2.2)

Since 1 > 0, is monotone increasing. The statements below apply.


1. There is a constant 0 < < 1, and a function = () periodic of period 2 such that any
solution to (2.2) has the form
= (t t0 ) + ( (t t0 )),

(2.3)

where t0 is a constant and (0) = 0.


2. Note that sin() is periodic in
t, of period T = 2 , with

M = average(sin ) =

> 0,

(2.4)

where M is defined by the first equality M = M () only, since depends on only.


3. Let be the solution to (2.2) defined by (0) = 0 i.e.: set t0 = 0 in (2.3). Then
Z t
1
( t) =
(sin( (s)) M ) ds = ( t),

(2.5)

where is defined by the first equality.


4. Assume that 0 <  1. Then a Poincare-Lindstedt expansion yields
= t (1 cos( t)) + O(2 )
It follows that T = 2 + 2 + O(4 ) and M =

and = 1

1
+ O(3 ).
2

1 2
+ O(4 ).
2

(2.6)

18.385 MIT, (Rosales)

Coupled oscillators, phase locking, oscillator death, etc.

5. Assume that 0 < 1  1. Then

= O( 1 )

(2.7)

In case you are curious as to how to show the above applies, here are some hints.
a. Define T > 0 as the (unique) time at which (T ) = 2 why is the solution unique?
b. Show that (t + T ) = 2 + (t) sub-hint: both sides are solutions!
c. Define by ( t) = (t) t, with = 2 /T , and show that is periodic of period 2 .
d. Write the general solution in terms of .


e. Show that T = O 1/ 1 as 1 sub-hint: critical slowing-down.
f. To show that < 1, use (2.2) and separation of variables to write T as an integral over from 0 to
2 . Then show T > 2
g. To show (2.4), take the average of (2.2).
h. To obtain the second equality in (2.5), substitute = t + ( t) into (2.2), and obtain a formula
for sin( ) in terms of .

Notes: coupled oscillators, phase locking, etc.

These are notes with facts useful for the problems. They are not a problem.

3.1

On phases and frequencies

Consider a system made by two coupled oscillators, where each of the oscillators (when not coupled) has a
stable attracting limit cycle. Let the limit cycle solutions for the two oscillators be given by ~
x1 = F~1 (1 t)
~j are
x1 and ~
x2 are the vectors of variables for each of the two systems, the F
and ~
x2 = F~2 (2 t), where ~
periodic functions of period 2 , and the j are constants (related to the limit cycle periods by j = 2 /Tj ).
In the un-coupled system, the two limit cycle orbits make up a stable attracting invariant torus for the
evolution. Assume now that either the coupling is weak, or that the two limit cycles are strongly stable.
Then the stable attracting invariant torus survives for the coupled system.4 The solutions (on this torus)
can be (approximately) represented by
~x1 F~1 (1 )

and ~x2 F~2 (2 ),

(3.1)

where 1 = 1 (t) and 2 = 2 (t) satisfy some equations, of the general form
1 = 1 + K1 (1 , 2 )
4

With a (slightly) changed shape and position.

and 2 = 2 + K2 (1 , 2 ).

(3.2)

18.385 MIT, (Rosales)

Coupled oscillators, phase locking, oscillator death, etc.

Here K1 and K2 are the projections of the coupling terms along the oscillator limit cycles. For example,
take K1 (1 , 2 ) = sin 1 cos 2 and K2 (1 , 2 ) = sin 2 cos 1 . Another example is the one in 8.6 of Strogatz
book (Nonlinear Dynamics and Chaos), where a model system with
K1 (1 , 2 ) = 1 sin(1 2 ) and K2 (1 , 2 ) = 2 sin(1 2 )
is introduced, with constants 1 , 2 > 0. Note that:
1. In (3.2), K1 and K2 must be 2 -periodic functions of 1 and 2 .
2. The phase space for (3.2) is the invariant torus T , on which 1 and 2 are the angles. We can also
think of T as a 2 2 square with its opposite sides identified. On T a solution is periodic if and
only if 1 (t + T ) = 1 (t) + 2 n and 2 (t + T ) = 2 (t) + 2 m , where T > 0 is the period, and both n
and m are integers.
3. In the Coupled oscillators # 01 problem an example of the process leading to (3.2) is presented.
4. The j s are the oscillator phases. One can also define oscillator frequencies, even when the j s
do not have the form j = j t, with j constant. The idea is that, near any time t0 we can write
j = j (t0 ) + j (t0 ) (t t0 ) + . . ., identifying j (t0 ) as the local frequency. Hence, we define the oscillator frequencies by
j = j . These frequencies are, of course, generally not constants.
5. The notion of phases can survive even if the limit cycles cease to exist (i.e.: oscillator death). For
example: if the equations for 1 and 2 have an attracting critical point. We will see examples where
this happens in the problems, e.g.: Bifurcations in the torus # 01.

3.2

Phase locking and oscillator death

The coupling of two oscillators, each with a stable attracting limit cycle, can produce many behaviors. Two
of particular interest are
1. Often, if the frequencies are close enough, the system phase locks. This means that a stable periodic
solution arises, in which both oscillators run at some composite frequency, with their phase difference
kept constant. The composite frequency need not be constant. In fact, it may periodically oscillate
about a constant average value.
2. However, the coupling may also suppress the oscillations, with the resulting system having a stable
steady state. This even if none of the component oscillators has a stable steady state. This is oscillator
death. It can happen not only for coupled pairs of oscillators, but also for chains of oscillators with
coupling to the nearest neighbors.
On the other hand, we note that it is also possible to produce an oscillating system, with a stable oscillation,
by coupling non-oscillating systems (e.g., the coupling of excitable systems can do this).

THE END.

MIT OpenCourseWare
http://ocw.mit.edu

18.385J / 2.036J Nonlinear Dynamics and Chaos


Fall 2014

For information about citing these materials or our Terms of Use, visit: http://ocw.mit.edu/terms.

Problem Set Number 10, 18.385j/2.036j


MIT (Fall 2014)
Rodolfo R. Rosales (MIT, Math. Dept.,Cambridge, MA 02139)
November 19, 2014
Due Fri., December 05, 2014.

Problem 09.06.02 - Strogatz. Pecora and Carrolls approach

Statement for problem 09.06.02


Pecora and Carrolls approach for signal transmission/reception using the Lorenz system. In the pioneering work of Pecora and Carroll 1 one of the receiver variables is simply set equal to the corresponding
transmitter variable. For instance, if x(t) is used as the transmitter drive signal, then the receiver equations
are

xr (t) x(t),

dyr
= r x(t) yr x(t) zr ,
(1.1)
dt

dzr

= x(t) yr b zr ,
dt
where the first equation is not a differential equation.2 Their numerical simulations, and a heuristic argument, suggested that yr (t) y(t) and zr (t) z(t) as t , even if there were differences in the initial
conditions.
Here are the steps for simple proof of the result stated above, due to He and Vaidya.3
A. Show that the error dynamics are governed by:

ex (t) 0,

dey
= ey x(t) ez ,
dt

dez

= x(t) ey b ez ,
dt

(1.2)

Pecora, L. M., and Carroll, T. L., Synchronization in chaotic systems. Phys. Rev. Lett. 64:821, (1990).
This equation replaces the first equation xr = (yr xr ) in a Lorenz system for (xr , yr , zr ). Then x is used to replace xr in
the other two equations. The Lorenz system constants are , r, b.
3
He, R., and Vaidya, P. G., Analysis and synthesis of synchronous periodic and chaotic systems. Phys. Rev. A, 46:7387
(1992).
2

2
where ex = x xr , ey = y yr , and ez = z zr .
B. Show that V = (ey )2 + (ez )2 is a Liapunov function.
C. What do you conclude?

Hill equation problem #04 (with damping)

Statement: Hill equation problem #04 (with damping)


Let S = S() be a periodic (of period 2 ) function i.e.: S( + 2 ) = S(). Consider now the damped
Hill equation problem

x
+ 2 x + k2 + a2 S( t) x = 0,
(2.1)
where , k, a, > 0 are constants note that the coefficients period is T =

Problem tasks:
1. Write the equations in the standard form X = A( t) X, where A is a 2 2 matrix with period 2
and X is a two-vector.
2. Write the Floquet multipliers j in terms of = 12 Tr(R), where R is the Floquet matrix.
Hint. = det(R) can be computed explicitly.

3. Write the stability/instability condition in terms of .


4. Find the function 0 = lima0 . Then use it to identify the places, if any, where an instability may
occur for 0 < a  1. That is, the values k = k such that, for 0 < a  1, instabilities can arise for k
near k only.
Hint. For a small instabilities only arise near ks where a Floquet multiplier satisfies |j | = 1 for a = 0.

5. Plot 0 versus k/, with / fixed, in a graph that includes the neutral stability curves. Use the
range 0 k/ 5.1 and take / = 0.06, 0.20, 0.50.
The neutral stability curves are lines in the -k plane such that: a Floquet multiplier satisfies |j | = 1 when/where
the graph of intersects the curve. You should know these curves from item 3.
Hint: when solving item 4 you should find that 0 is a function of k/ and / only, while the neutral stability
boundary depends on / only.

6. Plot 0 versus k/, with a function of k, in a graph that includes the neutral stability curves. Use
2
the range 0 k/ 5.1 and take = 0.06 k, 0.12 k, 0.10 k .

THE END.

MIT OpenCourseWare
http://ocw.mit.edu

18.385J / 2.036J Nonlinear Dynamics and Chaos


Fall 2014

For information about citing these materials or our Terms of Use, visit: http://ocw.mit.edu/terms.

  
 

"!$#&%!(')'*+,%.-/01324-,!(#&5)6.6%7%!(,,#98&:#;8<!(0;81#&5=>@?BADCFEGAIH9JGCLK,5)#I#&K0;818&8<0;5M!('
,N#&6N0&5=-/#8<K,(#&O2P,Q!(%7R8S!3'/6N%7%T!(,#J*(T,NT8&@UWV<X&Y(Z[XO\N6N8&0&#]!(,$%!38<0;5=6N#^R8&@US_Y(`babX
8&K,%@^(!3,c8&edSfQXWV;Y(ZgXhF5)8&Kc8&K,%@J*CLK#&]!(0&!(')'#&K,(0;8*K,5=>Kbig'=W\N'R6%7%T!3,,#^j-/6!(,#&k?BADCFEGAIH
6N,#;8<!(R8<'mlch40;nb#ohF5)8&K%!38<0;5=6N#Jopq-/'=5)N\(8&KQ!38l(hF5)'='')5=n(8<K]r/jh40o8<KQ!s8*8&K,5=#o#&(2t8gh]!(0;]>5m\#N^
8&77')5=,!(0L!(')>-0<!u-Rlv!7#;0;5=#F(2*#&K0;8w5=#;8<0;,6N8&5=#x
UWV<X&Y(ZgXwy
UWVSX&Y(ZgXwz
US_Y(`babXwy
{u|b}~Zf/}~yz
yNlRR
z:yTxmj
y$jD
uykz




CLK,wh](0&:Nl(I#;8<!(,,#2P08<K,I5),R8&5)8gle%!38<0;5mJCLK,9#&,-,%!38&0&5m7z$yTxmjDr,5=6Sn+#k8]6N'=,%7 (J
CLK,5=,#8<0;,6N8&5=By$je0;#&W8<#98&K,TLNR8<0;lv8&:JkCLK6(%%!(,ByLz%.')8<5)r,'=5)#]8&K,
%!38<0;5=6N#Oy!(, zoJOA9')'o8&K,#;.6%7%!(,,#O!(0&0&r/!s8<5) 09'=5)#;8O-N'=hJIw0;.5=#O!(,!3%r,')
(2*5=R\(0;8&5=,>7!u%!38<0;5m$!(,@#&'m\5),>e!e'=5=!(0k#lb#;8<N%Bx
UWVSX&Y(ZgXc
UWV<X&Y(Z[X
t`Q(XWVZ4
ds}~(X1Ov
#RFWlRR LuxmSR
5)R\GP1
@190
u
v L

CLK,O%T!s8<0&5u(2q!('='Q(,#hk!(#]!(,,N8&Nl(RRW^!(,TF5)#5)8<#D8&K,5=0;T6('=,%7qJCLK$5)+\P1Dr,0&b,,6N#
8&K,5=R\N0&#&%T!s8<0&5P,0&%!(')')l5=v,6N5=%!('=#N2P0L2P0<!36N8<5),#L,#;OdsV{7Y(ZV&Y(Z;J]CLK,c#;lb#;8&%OB
5)#I#&'m\N-+l 5=R\q1w^qhFK,5=6SK5)#O8<Ku#&'=hh]!l(JCLK,u-,!(6Sn+#&'M!3#&K6%7%!(,Ic,#;#
!(,#;#&5M!3:'=5)%5)Q!38&5=e5)2o5)#k#&+Q!30&c!(,v,W\0F6N%r8<N#k8<K,15)+\(0;#&%T!s8<0&5J*K,N@8&K,10&5)>KR8
#;5=,9O+Q!3'=#48&K,I8<K5=0&6'),%7$(2^b8<K,9#&')8<5)@%.,#8L-/e7v)=Jwjq_QX9ZV;Y(`gfQdXOW3{7Wds}b
{7YsRXSYUSds}~|{u`v(XSUWZgdsVWJCLK,N$Gvr,5=6Sn+#88<K,F8<K5=0&76('=,%7e(2q^!(h4OKQ!\(9OvJ
ON0&c!(0;!.2PWh6(%%7R8<#NJCLK,6%7%NR8F#;lb%.-/'G5=#w$x
CLKc#lb%.-('=#F$!(B!(0;7( IXWVSXW`QZx?BADCFEGA9H5=#L6!(#&ig#;,#;5)8<5m\3J
Clbr7_QXW} fW}mYsS_B2P0O!T,N#&60;5=r8&5=B(2DK,h8<,#&8<K,.-,!(6Sn+#&'M!3#&K#;lb%.-/'JwCLKh](0&"_QXW} f
6!(@-I2P')'=h4v-Rlv!T?@ADCFEA9H#lb%.-('G0L6%7%!(,vQ!(%710w?viQ'=1Q!3%3J

w(8<3xCLK,I6N%7%T!(TQ!(%7w5=#]r,rN0]6!(#&I5)T8<K9,N#&60;5=r8&5=T>5)\($-RlK,')rq^-8k%.#;8L-/
)' h]N0L6!(#;5)B!36N8<,!('q,#;(JA9,v8<K,-Q!36Snb#;'M!(#;KBI95=#L5)0&NR8whFK 5)#L,(8F#&+Q!30&(J
Cou,5)#&r,'=!lT!(')'*,5=>5m8<#8glr/kdsV{7Y(ZD}=ds`ba(JDCLK,1,0;%T!3'bdsV{7Y(ZS_QdsVZ]>5m\N#ku,5=>(5)8<#]!32t8<N0
8&K,,6N5=%!('J
A#;%75=6N'=v!s2t8<0w!u6(%%!(,v!\(5=,#F,5)#&r,'=!lT(2o8<K,0&N#&,'m8J
,#R,hF5=')',(8F5=#&r'M!l$8<Kuu5),R8<5m8gl:%!38<0;5mJ
w#&18<Kcri[!(0;0&h6N,0&#;0L8<e0&W8<,0;@8&er,0&W\5),#F6(%%!(,,#NJ
o 11O
Qo G4;1]4
z$ Ts KQ!(#Fc0;hhF5)8&K:8<K,0;c6%7r(,R8<#!vcu%!38<0;5m,
. <,s KQ!(#L8&K,0&N0;jhF#F#;rQ!(0&!38<N -Rl:#;%75=6('=,#c!e7"%!38<0;5m,
. Ts 0Fuz ZV;Y(`gfQdXSczB8&7r0&b,,618&K,#<!(%7
x >N,0&!38<N#w8&K,0&h\(6N8&0 e$jGhF5)8<Kv,5)8F#;8&r,#
zx x 8S!3n(#w#;8&r,#w(2 8&e>5)\(1zv~
o 11O
g Qo QD
~ &Te3 KQ!(#L8gh40;hF#.!(')h]!l#w!u#&N%5)6'):-/N8gh4 0&hF#S
~
!(')#&er,0&b,,6N#F8<K,%!38<0;5mT-,8w5=#LK,!(0&,N0L8<u8glbr
$e3
~ <e3m5=#k8&K,eZV;Y(`gfQdXc(2*JDCLK+,#ww5=#Lw5)B?BADCFEGAIH
o GQ7[Go
o P:r0&b,,6N#w8&K,,5M!3>Q!('%T!s8<0&5ThF5m8<K:\N6N8&0O@5)8<#L5M!(>(Q!('
b* P>5)\(#F8<K,c#;lb%7%W8<0;5=6$USds`WZY(`QZ (Yabds`/Y(}o%T!38&0&5$hF5)8&K :!3#wQ0;#;8O0&h!(BQ0;#;8O6'i
%
b* <e>5)\(#k8<Kc6N,#8S!(R8;i,5M!3>Q!('%T!s8<0&57hF5)8<K !3#kQ0&#8F6'),%v!3,v$!(#kQ0;#;8F0;jh
G:>5m\#w!( %T!s8<0&5$32#

  





R j qv>5m\N#O!3 %!38<0;5mT(2 0;#


/ G:>5m\#L8&K, 5),R8&5)8gl$%!38&0&5m
G:>5)\(#L!3 B %!38&0&5mehF5m8<K$0&!(,,(%N+8&0&5)#k-/N8gh4N .!3,e,5)2P0;%,5=#8<0&5)-,8&5=
q@>5m\N#F!( %!38&0&5mhF5)8&K:,0;%T!(')')le,5)#;8&0&5=-8<N@NR8<0&5)#cP%!(ve!(,:\3!(0&5=!(,6j
 q R j  G  G>5m\ %T!38&0&5)6#
R P1 R s R P1 / R 1 O>5)\(1%T!38&0&5)6#L32*8&K,#<!(%71#&KQ!3r!(#L
o  I o] ]g4* ,
. D 0&N#&W8<#F8<K, ]R8&0;l:8<eNb,!('
. xD 0&N#&W8<#F8<K,18<K5=0&v0;jh8<7+Q!('G
.xm D 0&N#&W8<#F8<K,#&N6 6('=,%7:8<e+Q!('
CLK, c6'):#;lb%.-/'D xQ#8S!(,#F2P(0cY(}t}L!3'='6N '=%,#L(0F!('='q0;hF#S
u; sxD.& x]6SKQ!(,>N#90;jhF# !(,Bu(2*
o GQ 1G
@
,
. &
0&W8<,0;,#L8<K, ; bkR8<0lv(2*8&K,%T!s8<0&5$P#&6!('M!(0wc%!38&0&5m,
. &x
0&W8<,0;,#L8<K, 8&KB0;h(2*!(#F0&h\6W8<0<
.xm b
0&W8<,0;,#L8<K, 8<K@6'),%7:(2!(#F6'),%$\(6W8<0<
. xR,Sux 0&W8<,0;,#F0&hF#w2P 0;% 8<e7!(,@6('=,%7,#k2P0&(% .8< !(#Fe.%T!s8<0&5,
.& (x 0&W8<,0;,#F0&hF# !(B7!(,B!(')'6('=,%7,#c!3# %!38<0;5m,
.x
0&W8<,0;,#F,'=>76N'=%$2P0;%N:2P0&%8<K,6('=,%7,#L(2* %T!s8<0&5,
P1 #;N8&#9!(')'N+8&0&5)#w-N'=h8&K,%T!35=:,5=!(>Q!3'8< N0& r,rN0L8<0&5=!(,>'M!(0<
P1 #;N8<#O!(')'qR8<0;5=N#O!3-\18&K,%T!(5):,5M!3>Q!('/8< N0& '=h40k8&0&5M!3,>,'=!(0S
,1Q1;[G,4  k]Q4
k >5m\N#F8&K,%T!s8<0&5$r0&b,,6N8w9P5)2o6!(@%.,'m8<5=r')lu
L >5m\N#F8&K,R8<0lRig-RlRigR8&0;l@r,0&b,,6W85)2o#;5 Iw#&5 NRu&
5)+\qI >5m\N#w k5m25)#L#&+Q!(0;!3,B5)R\08<5)-,'=
r,5)R\GP1 >5m\N#F8&K,r,#&N,,5)R\0;#&(2
I >5m\N#w5)R\IoL5m25)+\qIkW5=#8<#xOSYbUSW}mYsS_v5)#L'=W2t8L,5)\b5)#&5=(
:1 >5m\N#F8&K,#&')8<5)$8<7O:95m25)R\GP1kW5=#8<#
Ne_QXW} fW}mYsS_$hFK,B5=#F!e0&6W8S!(>,'M!30L%T!38&0&5


 









  
















 



 












"!









 

#!

&

('

. 0/

%$

*)

+-,

%$

. 1/

.

.

1/

2/





3 4

3

"8

:<;

?>

1 Q  ,[GDF g
R Iv5)#k8<K,bXWZgXWVW{ut`/Y(`QZI5m2*5)#F!e#;+Q!(0&c%!38<0;5m,
Iv5)#k8<K,eV;Y(`P+,%.-/0F(2*r,5)\((8<#O5=%7,#;5=:(20&h#;rQ!(6N!3,B326N'=,%7:#&r,!(6
R I:5)#k8<K,rQ!35=0k(2+,%.-/0&#
P1$5=#k8&K,eZV;YbUSX#;,%(2*,5M!3>Q!('R8<0;5=#O#&,% (2*5)>R\3!('),#
P1T5=#F!e%!38<0;5mhFK,#& v u6N'=,%7,#F!(0;!(@0;8&K,>,!('-Q!3#&5=#L2P0k8&K,+,'=')#&rQ!(6N1(2
1$5)#F!7%!38<0;5mhFK,#; u6N'=%,#F!30&!(@0;8&K,>(Q!('-,!(#&5)#F2P(0k8<K,6('=,%7:#&rQ!36c(2
91[o
yWlbP+WSyT jOe60&!38<N#9!u7e')%7 R8S!(0l$N'=5=%75=,!38<5)T%!38<0;5m
y:#;,-8&0<!(6W8<#Ou8&5=%7#L0;jh(22P0;%0;jh J
<60;!38&#F8<K, c!(>%NR8<@%!38<0;5mhF5)8<K@I!3#wb8<0<!e6('=,%7
yWlbRW y$; 3x]60&!38<N#9!ur/0;%.8S!s8<5=(:%T!38&0&5
w(8<I8&KQ!38F8<0;5=GP1G8<0&5)'15M!(>P,5M!3>I&k+Q!(')#F
1g (; c W, D4 G,4IN;$4] [
L 4 1 1k>5m\N#F8&K,0&N%T!38&0&5)6#khF5m8<K
P P1]5=#F!u\N6N8&0F6R8S!35=,5),>u8<K,5)>R\3!('),#L(2*
Sy I4>5m\#k!.,5M!(>(Q!('/5)>R\3!('=9%!38<0;5mTy!(,:N5=>NR\6W8<0L%!38&0&5m hF5)8&K$
Dy7JDp25=#L(8w,5M!(>(Q!(')5 !3-,'=P8&.2PWh5)>R\(6N8&0&#<F8&K, 5)#L,(8w5=R\N0;8&5=-,')(J
e 9/ P1D>5)\(#L!( 0;8&K,>(Q!('%T!s8<0&5 !(, 8<0;5M!(,>(,'M!(0 hF5m8<K
QQ
c
?viQ'=N#]!30&L8<84Q'=N#,,5),>hF5m8<K@J %hFK5=6SKv?BADCFEGA9H
,#;#]2P0D2P,6N8<5),#!(,#&6N0&5=r8<#JDA#&6N0&5)r8
5)#F!7#;+,6.(26%7%!(,,#khFK,5)6SK %!l:-N68&32t8<^!3,B6!(@-cr,'M!36@5=@!(@%uiQ')1#&
8&K,6%7%T!3,,#k,e,38OK,!\(18<e-0;N8glbrNqJ]?BADCFEGAIH #O%7#F!(0&1!(%r'=#F(2o8<K#&#;60&5)r8<#NJ
AOW!(%7r,'=75=#8<KT,N%B6!(')'= _Qds|+XWJB?B(#;8.32w?BADCFEGAIH #2P,6N8<5),#!(0&!(6N8&Q!(')')l%uiQ')#^
!(@6!(@-1\b5=Wh]N -Rl$hF0&5m8<5),>:ZSfQX BhFK,N0& 5=#k8&K,Q!(%7(2*8&K,12P,,6W8<5=(qJ
@

A&

"8

CD$

B8

BE

F

3 <G
 4



IH

MH

J LK

O

ZY

\[

D7RQTS

U'

D'

O

WV

O

]Y^[

a`

b`

Zc

d

]e 

f

`

`

f

6c

g

"!l7RQTS

nm

nm

no3o3o

Mo3o3o

6

hd

icjd

CoBhF0;5)8<el(,0hF #;60;5=r8&#02P,,6W8<5=(,#^l KQ!\T8&B6N0&!s8<$!@,Nh8&Wb8,'=ehF5)8<K"!(RlQ!(%7


l(B')5=n3(^Qr,0;j\b5),@5)8FN,,#FhF5)8<KJ%B^#&T?BADCFEGAIHhF5)'='q0;6N>,5 5)8JDC*8wQ')#F6!(@-/c6N0&!38<Nq^
N,5)8&B!(, #&!\BhF5)8<K !(Rl:8<8O5)8<(0^Q'=5)n($XW{7YbUSW^ ^Q0tJ]A#&6N0&5=r8OQ')5=#F#&5)%r,'ml:!e'=5)#;8F(2
?BADCFEGA9H 6N%7%T!(,#JoK,e8<K,k,'=LQ!(%7k5)#8glbrNT!388<K,O?BADCFEGAIHr,0;%7r8^8&K,L6R8&R8<#(2
8&K,9Q')OhF5=')'-I68<J Q0]!(:%eiQ'=F8<-1!c2P,,6N8&5=T5)8k%,#;8L#8S!(08khF5)8&KT8<K9h](0&.|b`UWZds`
2P')'=h4:-Rl:8<K,8<r,8F\3!(0;5M!(-,')#k5)v-,0<!(6Sn3N8&#^Q8<K,12P,6N8<5)@Q!(%7(^Q!(v8&K,5=,r8F\(!30&5M!3-,'=N#J
91[o
|b`UWZds` w{u|}~Z
V wV;Y(`
OR4
!\(8&K,!(-\("6N%%!(,#:5=R8< !8<Wb8@Q'=,!(%N%,')8J % CLK,8<K,5)#$2P,R8<5=(hF5)'='O8<!(n("!
%!38<0;5mv!(, 0&N8&,0& ,')l:8&K,c%T!38&0&5vr,0&b,,6W81eJFCLK,\3!(0;5M!(-,')eV75)#O(890;N8&,0&,N-N6!(#&.5m8
h]!(#I,3895),6'),,!(#I!(B8&r,89\3!(0;5M!(-'=(JFCLK,6%7%T!3,,#O!(0&c2P'=')jh4B-Rl ]#&8<KQ!38w8<KNl@hF5='='
,38u-Tr,0&5)+8&8&8<K,v?@ADCFEA9HhF5=,,hN\(0l8<5)%78<K,Wl!(0;$W6N8<NqJ p8.5=#.,#;N2P,'4hFK,
,!('=5),>.hF5)8&Kv'M!(0;>I%!38<0;5=6N#Jw0&5)#O!3,(8<K0LW!(%7r,')(x
|b`UWZds` V Df/V<dSfQXWVZXS
q_b=|b`UWZds` `/sZt_QXcV&Y(` ]XWmabXW`QsY(}~|XSY(`/:XWmabXW`Q(XSUWZ[dsV<.d[
{ ` LW sX
 1{ w`
1XWma
V wV;Y(`
XW}X
(=gf 4VV<dsV eq_QX{7Y(ZV ${u|RWZkWX9 |,Y(V<X
XW`/
w0&c8<K,c2P,,6W8<5=( 8S!(n3#98&K,%!38&0&5m !(#O5=,r8I!3,,'mlv0&W8<,0;,#98gh4$%T!s8<0&5)6N#9!( 8<K,0<!(n
!(#T8<r8JCLK, 5=#$#&!3#:!6%7%7R8JCLK2P,,6N8&5= 6SK,6Sn+#:8<#;5m218<K,5),r,8T%T!ji
8&0&5m 5=#9#&+Q!(0;e!(,8<K,NQ,,#I8&K,u0<!(n/^GN5=>NR\(!3'=,N#1!(,5)>R\(6N8&0&#(2!:%!38<0;5m 1JqClr5=,>
f/V<dSfQXWVZXS T,')l0&N8&,0&#c8&K,7,0&#;88&r,8^ G^8&K,e%T!s8<0&5(2]5=>(R\N6N8<(0&#JT("%.#;8c8glr/
V Df/V<dSfQXWVWZXS 8<e>N8O!(')'8&K,0&N8<r,8&#J
p

?qsr

^t

uvxwzy

zy

{|~}

{|~}3

M

u3" wzy

{|^}

u zwzy

{|^}3

gyy

u3?wzy
zy

C{|^}3

{|~}3

{ q

< }3

|

|

{|^}

u3?" wzy

{|^}

o1;  D ]4I
CLK,6(%%!(, o P @8&'=')#@?BADCFEGAIH8< 0&N60;N\(0lb8&K,5=> ,5)8<K, ?BADCFEGAIH
hF5),,h^!(#<!\8<K,0&#;,')8&#5)"8&K,8&Wb8Q'),!(%N Q'= )JvClbr,5=,> oP (0 o
8&>>')#]8&K,0&6N0&5=,>,J I'):,5M!(0l,'=#F6!(v-1\b5=Wh]NB,#;5=,>7!.8<b8F,5m8<0N^,0Lr,0;5=R8<N@#&5=>@} f/V
5)@,5mJDpg ?BADCFEGAIH9^Q8<KNl:6!(B-/1\b5=Nh4@,#;5=,>e8<K, GD }=Xc6%7%T!3,qJ
GO; 4]IDD[ 1[o  G
CLK,u6N%7%T!( *P #&!\#18&K,u6%7%!(,,#9l(8glr/!(#Ih4')'D!(#c?BADCFEGA9H #8<r,8^G-,85m8
,b#1(81#<!\(e8&K,u6(+8&R8(2l(,0I\3!(0&5=!(-,'=N#I!(%!38&0&5=6N#NJcCLK#&u\3!(0;5M!(-,')#O6!(-/.'=5)#;8<N-Rl
8&K,T6N%%!(, o hFK5=6SK!(')#&@'=5)#;8<#8<KT#&5 N#(2k8&K,T%!38&0&5=6N#NJ:CLK,$6(%%!(,
hF5)'='#<!\$8&K,:%T!s8<0&5)6N#.!(, !(')'\3!(0;5M!(-'=#'=5)#;8<N-Rl 8&K, o 6(%%!(,5)R8<8<KTQ'=TQ!(%7
J ?BADCFEGAIH'=!(-N'=#18<K#&$,'=#chF5)8&K! J%T!s88&,#;5=5),#;8&!( (21J% hFK5=6SK !(0;T#&6N0&5)r8<#0
2P,6N8<5),#NJ ={7Y(Z]Q'=N#L6!(@-/c0;!(@-Rl@?BADCFEGAIH!38w!u'M!38&0k8&5=%7I-Rl$8glbr,5=,> Q J

"!

#8

s'

L h\

<

J 

3

L

"!

h&

"8

nm

3

 

K


K


o3o3o

ho3o3o

o3o3o

D  
CLK,w#&5)%r'=#86%7%!(,e5=# *Q t ,*hFK5=6SK7,#&N#48gh4\N6N8&0&#v!(, .(28<K,w#<!(%7F'=N,>(8<KJCLK,
r/5=R8&#cP g t4hF5)'='q-/1r,')(8&8&@!(,@6,6W8<B-Rlv#&')5=:'=5),#NJ
p2,c\6W8<04:5=#>5)\(q^,?BADCFEGAIH!3#&#&%N#]8<KQ!s84* g &JDCLK *Q ,KQ!3#]+Q!3'#;rQ!(65),>
:8&K,1,i[!s5=#NxD8<K,r/5=R8<#F!30&7 ; g;WJ
CLK,I8glbr/!3,:6')0]32o8&K,1'=5),9-/N8gh4NBr/5)+8&#k6!3@-/16SKQ!(>v-Rl:!8&K,5=0;v!(0&>%N+8JDCLK,
,W2!(,'m8LhF5)8<Kv,!(0;>,%7R8F5=#F!e#&')5=:-,'=!(6Sn$')5=, ,JO#;e_QXW} fTf/}=dsZ]2P0L%!(Rl:r8&5=,#N^h45),,5mi
6!38<,'ml:!.2PNhx
?BADCFEGA9Hx *Q t / x~Lr'=(8&#L5= 0;vhF5)8&K2P0Lr(5=R8<#F!(B,38&8<NB')5=,
?BADCFEGA9H,x *Q t / ]5=#w!uQ!(#;K,B'=5),1!(, *Q t ]5)#F!7,38&8<Nv'=5=
@6!(v%75)8]8<K,'=5),#F!3,vr,'=(8F(,')l8<K,,5)#&60;N8&cr/5)+8&#L5=v,5mN0&R8Fh]!lb#Nx
*Q t / j~L>(5)\N#F65=0;6')#J 98&K,0Fr8&5=#F!(0&9/0kgD/0L)j
Q0B8gh4>0<!3r,K,#8&K,#&!(%!s#,#; PQ P / J wNr,'M!36 *Q -Rl P 0
 P 0  P 8&"6SK,!(,>@,@07-(8&K !jN#8&"')>R!(0;5)8<K%5)6T#&6!('=3JCLK,B6%7%T!3,

z2

< T

.

.  .

0

H

H

 

 

 

& t*hF5)'=',#;6!(')k8<K,F>0&!(r,Ke8<'=5)L5=u8<Kw0;6W8S!(,>('=F ^
QJCo8<5)8&'=]8&K,
>0&!(r,K:0L'=!(-/'8&K,I,i[!j5)#k0]8&K, +ig!s5=#N^r,8L8<K,1,N#&5)0&v'=!(-N'5):b(8<N#O!3#L5=$8<K#&,!3%r,')#Nx
ZtZ}=XT K,5)>KR8O32#&!38<N'='=5m8< }mYRWXW}L 8&5=%7I5=v#;6(,,# /}mYRWXW}] K5=>(K+8F5)v%W8<0;#
CLK,.6N%7%T!( n(Nr,#18<K,6,0;0&R81>(0<!(r,K!(#9l(r,'=(89!T,Wh>0<!(rKqJ Fr/!38&5=,> o/
hF5)'='6N'=!(0k8<K,#;60&NqJCor,0;5=R8^0L#<!\c8<K,>0&!(r,K,5)6#khF5),,h5)B!.Q')(^,#;u_QXW} f$f/Vt`QZk0L,#;
r,0&5)R8ki Dr,0&5)R8<0;Q!(%7 r,0&5)R8kig Q')Q!3%

"b

MIT OpenCourseWare
http://ocw.mit.edu

18.385J / 2.036J Nonlinear Dynamics and Chaos


Fall 2014

For information about citing these materials or our Terms of Use, visit: http://ocw.mit.edu/terms.

You might also like